Download - QUESTION BANK - edumentor.co.inedumentor.co.in/Downloads/2018/Extra Content... · of India's oldest and most important uranium mine, ... Punjab banned e-cigarettes last year and Maharashtra

Transcript
Page 1: QUESTION BANK - edumentor.co.inedumentor.co.in/Downloads/2018/Extra Content... · of India's oldest and most important uranium mine, ... Punjab banned e-cigarettes last year and Maharashtra

IPM - IIM

BANK

QUESTION

for

Page 2: QUESTION BANK - edumentor.co.inedumentor.co.in/Downloads/2018/Extra Content... · of India's oldest and most important uranium mine, ... Punjab banned e-cigarettes last year and Maharashtra

IPM - IIM

Reasoning

Critical

Page 3: QUESTION BANK - edumentor.co.inedumentor.co.in/Downloads/2018/Extra Content... · of India's oldest and most important uranium mine, ... Punjab banned e-cigarettes last year and Maharashtra

Critical Reasoning

Inferences (An inference is something which is not directly stated but can be inferred from the given facts.) 1. A number of CEOs are preparing to make fresh investments and are also stepping up hiring. Which of the following inferences can best be drawn from the above statements? a) 6–8% GDP growth is likely over the next three years.b) A higher GDP growth is likely over the next few years.c) The new regime has failed to meet expectations.d) The companies will soon be burdened with loans.e) None of these

2. Statement: A Washington-based news organisation released areport that claims radioactive and toxic waste has been leaking out of India's oldest and most important uranium mine, affecting people, livestock, rivers, forests and agricultural produce in the area. Which of the following can be inferred from the given information? (a) The government has closed the mines. (b) Government of India has taken adequate measures to prevent toxic leaks from the site. (c) People living in the area have evacuated the toxic place. (d) Mining has exposed workers and villagers to radiation, heavy metals and other carcinogens, including arsenic. (e) Adequate measures have not been taken to prevent toxic leaks from the site.

3. Statement: The first tranche of sovereign gold bonds thatclosed on November 20 received an encouraging response despite falling gold prices, suggesting that the scheme may be successful in reducing imports of the metal. Which of the following can be inferred from the given statement? (An inference is something that is not directly stated but can be inferred from the given facts.) (A) Investment in gold is considered safe by the people. (B) Investment in gold is considered to give good returns. (C) Country imports of gold will fall. (D) The positive response to the gold scheme has elicited response from across the country. (E) The scheme will encourage bulk depositors such as Hindu undivided families and institutions to participate. (a) Only (A) and (C) (b) Only (B) and (D) (c) Only (A) and (B) (d) Only (D) and (E) (e) Only (E) and (A)

4. Statement: In a bid to secure airports the civil aviation ministry has decided to cut down the number of airport entry passes (AEPs) being provided to the staff of Union Ministers and MPs. Which of the following can be inferred from the given information? (a) No follower of any minister will be issued pass at the airport. (b) The airport will be less crowded. (c) Ministers and MPs will be to travel alone at the airports (d) The sale of visitor tickets will be stopped at all the airports. (e) Airport security will be strengthened.

5. Statement: Solar power tariff in India touched a record low ofRs. 4.63 per unit, accelerating India’s $ 160-billion clean energy

drive and casting a shadow on fossil fuel plants that pollute air and sometime charge a higher rate. Which of the following can be inferred from the given information? (An inference is something that is not directly stated but can be inferred from the given information). (a) Lot of private companies are investing in solar power projects in India. (b) The government is focusing on clean energy. (c) The level of Carbon Dioxide (CO2) in India is rising. (d) The government is providing for setting up solar power plants. (e) None of the above

6. Statement: Google has received India’s permission to start testing Project Loon, under which the company will beam the internet from balloons in the sky, in an effort to provide affordable internet access. Which of the following can be inferred from the given information? (An inference is something that is not directly stated but can be inferred from the given information). (a) Project Loon will boost up the internet activities in rural areas. (b) The privacy of the end user of Internet will be lost. (c) The project will give an opportunity to watch our country and capture sensitive information, making India more vulnerable. (d) The company will be able to save lots of money through this project. (e) None of the above

7. Statement: Persistent weakness in global demand and the lower value of oil products led India’s merchandise exports to fall for the sixth straight month in May, while a decline in gold imports helped the trade deficit narrow to a three-month low. Which of the following inference drawn is probably false according to the above statement? (a) There will be softening of oil products in the global market (b) Rupee will appreciate against the dollar (c) There will be an increase in the collection of indirect taxes in the month of April and May (d) There will be an increase in the domestic demand of gold and jewellery. (e) Only (b) and (d)

8. Statement: Spitting on streets could cost people dear as the Maharashtra state cabinet approved an innovative anti-spitting law that combines monetary punishment with mandatory community service. Which of the following can be inferred from the above statement (An inference is something that is not directly stated but can inferred from the given information). (a) The law is likely to act as a precursor to banning chewing tobacco in public (b)Anti-spitting law would bring down transmission of all airborne diseases (c) People will be restrained form spitting out tabacco in public places (d) Maharashtra’s roads will look clean and tidy (e) Only (b) and (c)

Page 4: QUESTION BANK - edumentor.co.inedumentor.co.in/Downloads/2018/Extra Content... · of India's oldest and most important uranium mine, ... Punjab banned e-cigarettes last year and Maharashtra

Strong weak argument 9. Statement: In a move to strengthen public transport in the Capital, ahead of the implementation of the odd-even vehicle scheme, Delhi government has decided to issue permits to 10,000 new auto rickshaws by the end of December. Which of the following statements would strengthen the decision taken by the Delhi government? (a) New auto rickshaws will cause congestion on the road. (b) Pollution in the city will not be controlled. (c) Most of the people will use public transport in the city. (d) Delhi has the lowest pollution from auto rickshaws among metros. (e) People do not use public transport for lack of last-mile connectivity.

10. Statement: In its first bi-monthly policy review, RBI hasdecided to cut the repo rate by 25 basis points and take measures to ease the liquidity constraints in the banking system. Which of the following negates the above steps taken by the RBI? (a) Loans and EMI will become cheaper for retail loans. (b) Banks will pay higher interest rate of FD and RD. (c) Banks will have to deposit more money with the RBI. (d) Rupee will strengthen against other currencies. (e) None of the above

11. Statement: To keep railway premises and trains free from dirt and filth, Railways has introduced clean train scheme (CTS) at major stations of East Central Railway (ECR). Which of the following will be a fallout of the above statement? (a) Travellers will find neat and clean coaches while travelling. (b) Only a few stations under ECR will be neat and clean. (c) People can find clean and tidy platforms falling under the jurisdiction of ECR. (d) Only Premium trains and Rajdhani will be benefited from this scheme. (e) Only (b) and (d)

12. Statement: The Central Government of Country X has decided to prune the number of Centrally sponsored schemes to 30 from 50, a move that will allow it to allocate bigger sums to ‘core of the core’ schemes and deflect criticism that it has neglected some critical areas. Which of the following statements would strengthen the decision taken by Country X? (a) The government of Country X is not able to properly manage the funds which have been allocated to its states. (b) The government of Country X found that lots of projects are not benefiting the poor people. (c) The main focus of the government of Country X should be on social protection and social inclusion which require more funds. (d) The main focus of the government of Country X is on eradicating poverty and strengthening the education system in the country. (e) The decision will help the states of Country X to spend more on social security programmes.

13. The government is considering a ban on the sale of electroniccigarettes, a vapour device that delivers a nicotine hit minus the smoke. Which of the following statements substantiates the fact expressed in the above statement?

a) India does not have laws that deal directly with the sale of e-cigarettes. b) Unlike conventional cigarettes, e-cigarettes release vapor — and not smoke — when a nicotine-laced liquid is heated. c) An expert panel has found it could lead to addiction, particularlyamong the youth. d) Punjab banned e-cigarettes last year and Maharashtra is considering a similar move. e) None of these

14. Statement: The Ministry of Urban Development has launched a Smart National Common Mobility Card (NCMC) model to enable seamless travel by different metros and other transportation systems across the country besides being used for shopping as well. Which of the following statements need not be true according to the given statement? (a) The card will help people travel hassle-free across the country (b) Only one type of card will be used across the country (c) People travelling across the country can save time and money (d) Shopping and travelling will become easier for the people (e) Only (b) and (d)

15. Statement: It has been reported in a recent study that intake of moderate quantity of milk chocolate reduces the risk of suffering from central nervous system-related illnesses. Which of the following would weaken the findings of the study reported in the above statement? (a) People generally prefer to eat chocolate when they are young. (b) Majority of those not diagnosed with diseases related to central nervous system have stayed away from eating chocolates in their lives. (c) Chocolates contain certain elements which strengthen the functions of the central nervous system. (d) Majority of those suffering from central nervous system related diseases are middle-aged. (e) Many of those who suffer from diabetes also suffer from other major ailments.

Conclusion: 16. The project of the road construction (work) has crossed its first deadline as far as pre-monsoon road work are concerned. In the major city the road works are given great emphasis and these are the places where road work has been completed. Which of the following can be concluded from the statements given above? (a) To start the work of the road one has to go through a lot of tiresome paperwork before starting the repair work which delays the whole work of the road. (b) It takes several hours travelling via these roads. (c) The work of the road is going on (d) They will start the road works well in advance (e) None of the above

17. Statement: The Railways has earmarked two berths -one lower and one middle - in sleeper classes under the handicapped quota for physically challenged people travelling on concession. Which of the following can be concluded from the above statement? (a) Physically handicapped people can have hassle-free journey in trains.

Critical Reasoning - IPM

Page 5: QUESTION BANK - edumentor.co.inedumentor.co.in/Downloads/2018/Extra Content... · of India's oldest and most important uranium mine, ... Punjab banned e-cigarettes last year and Maharashtra

(b) Handicapped people need some privilege in trains. (c) Handicapped people will now not need any attendant while travelling in the trains. (d) A good initiative by the railway for the handicapped people. (e) Only a) and d)

18. Statement: After providing LPG in easy-to-carry 5-kg cylinders,the government launched 2-kg bottles at local kirana stores and introduction of online booking of new connections for subsidised cooking fuel. Which of the following can be concluded from the above statement? (a) Online booking will end hassles of customers running to gas agencies for getting a new LPG connection. (b) The 2-kg cylinder will cater to the LPG requirements for all sections of society, including economically weaker families (c) The online booking will help the customers get new cylinders delivered quickly (d) The scheme will be particularly beneficial for the rural people and poor who cannot afford to pay the price of a 14.2-kg or even 5-kg cylinder (e) All of the above

19. Statement: “To stop train pull chain” notice inside railwaycoaches is soon set a become history. The railway ministry has decided to do away with these chains. Which of the following can be concluded form the above statements? (a) The mobile phone number of the driver and assistant driver will be displayed in coaches, so that passengers can call in case of an emergency (b) Any one finding to stop the train without valid reason will be fined (c) Trains will run on time from one place to another. (d) Passengers will reach their homes on time (e) Only (a) and (d)

Assumptions (An assumption is something that is not directly stated but is supposed or taken for granted.)

20. Statement: The UN World Food Programme warned of severe shortages of food and fuel in Nepal caused by the ongoing major disruptions in imports across its southern border with India, saying that the land-locked country could face extreme hardship if prices of basic food staples continue to soar. Which of the following has been assumed in the given statement? (a) Pulses and cooking oil have increased by more than 30 per cent since August and more than 50 per cent since last year and in remote areas. (b) Nepal totally depends on their imports with India. (c) Disruptions at the border have led to increase the cost of some basic staple food. (d) Free flow of food items across the border was to ensure proper availability of food in Nepal. (e) None of the above

21. The India-Japan civilian nuclear deal may not be sealed during Prime Minister Narendra Modi’s trip to Japan.

Which of the following assumption is implicit in the above statement? (An assumption is something supposed or taken for granted.) a) Discussions are on to bridge the core differences between the two sides over the proposed treaty. b) Japan is insisting on tougher safeguards and “no nuclear test” clause. c) There are some concerns about the deal from either side.d) Discussions were stalled following Fukushima nuclear accident in 2011. e) None of these

22. A massive agitation is brewing in State X against the proposed widening of the national highway. Which of the following represents an assumption implicit in the given statement? (An assumption is something that is not directly stated but is supposed or taken for granted.) (a) The agitation is carried out by local villagers who would be worst affected by widening or the highway. (b) Many other states have much wider highways as compared to State X. (c) A national highway already exists in State X. (d) Commuters are greatly inconvenienced due to frequent accidents owing to the narrow highway. (e) For a road to be declared a highway, a certain width is essential.

23. ‘The first ever low-cost tablet with a three-hour battery which works only with Wi-Fi for accessing the Internet will be a game changer for India’s rural population who have been on the wrong side of the digital divide in the country.’ – Stated Company X, the manufacturers of the tabled on their role in providing access to Internet throughout India. Which of the following can be assumed/inferred from the given statement? (An assumption is something that is not directly stated but is supposed or taken for granted and an inference is something which is not directly stated but can be inferred from the given facts.) (a) Wi-Fi connections are already available in rural parts of India. (b) Other tablets did not have the features such as Wi-Fi connectivity. (c) Improvement in accessibility to Internet would help education sector the most. (d) A mere three-hour battery would be grossly insufficient to maximize its benefits. (e) The tablet would not benefit urban population in India as much as the rural population.

24. Small brands are now looking beyond local grocery stores and are trying up with supermarkets such as Big Bazaar to pull their business out of troubled waters. Which of the statement can be assumed from the facts/information given in the statement? (A) A smaller brand manufacturing a certain product of quality comparable with that of a bigger brand, makes much more profit from the local grocery stores than from the supermarkets. (B) As the supermarkets have been set up only in bigger cities at present, this step would fail to deliver results in the smaller cities. (C) Supermarkets help the smaller brands break into newer markets without investing substantially in distribution.

Critical Reasoning - IPM

Page 6: QUESTION BANK - edumentor.co.inedumentor.co.in/Downloads/2018/Extra Content... · of India's oldest and most important uranium mine, ... Punjab banned e-cigarettes last year and Maharashtra

(D) Supermarkets charge the smaller brands 10% higher than the amount charged to the bigger brands. (E) Being outnumbered by the bigger brands, visibility of the smaller brands at local grocery stores is much lower as compared to the supermarkets. (F) Smaller brands are currently making substantial losses in their business.

Cause Effect 25. Statement: Human activity is leading to the rapid draining ofabout one-third of the planet’s largest underground water reserves and it is unclear how much fluid remains in them. Which of the following can be the probable reason(s) for the decline of water reserve? (a) Decline of population all around the world (b) Unauthorised digging by private builders (c) Lack of policymakers in the country (d) Excessive a forestation and inappropriate use of natural resources (e) Only (a) and (d)

26. Statement: The water situation is grim as the total storage in 91 main reservoirs across the country has gone below the last 10-year average, says the report of the Central Water Commission (CWC). Which of the following may be the most probable reason for the fall in the level in reservoirs? (a) Less rainfall across the country (b) More use of water by the irrigation department (c) Depletion of ground level in the country (d) Less plantation of tree in the country (e) Wastage of water by the people of the country

27. Statement: The government is planning to put a cap on premature withdrawal of provident fund (PF) money. The employees’ Provident Fund organization (EPFO) has proposed that an employee be allowed to withdraw only 75% off the overall kitty, instead of 100% as permitted earlier. Which of the following may be an effect of the above decision taken by the government? (a) Premature withdrawal of such savings should be discouraged to ensure secure future for the employees (b) People who tend to withdraw PF money between jobs or those planning to use it for either buying a house for paying medical bills will not be able to withdraw the money (c) People will be able to withdraw more money at the time of retirement (d) People will invest in other flexible schemes in the market (e) None of the above

28. Statement: The government has decided to develop 25regional airports in the country. Which of the following will be an effect of the steps taken by the Government of India? (a) Regional connectivity in the country will be boost up. (b) Air travel for the common man will become cheaper. (c) Infrastructure development will be boost up in the country. (d) Jobs in the aviation sector will increase. (e) None of the above

29. Statement: Government-run Air India has launched a limited-period super sale scheme, offering customers all-inclusive ticket prices starting as low as Rs. 1,499 on its domestic network. Which of the following will not be an effect of this scheme? (a) More people will able to travel by flight with the launch of this scheme. (b) Other airlines will come down. (c) Air-India’s profit will come down. (d) Fewer seats will be left vacant in Air India with the launch of this scheme. (e) None of the above

30. The temple at the religious site wears a deserted look with the number of devotees trickling down. Which of the following can be a possible cause of the above effect? (a) A structural engineer had visited the temple a month back and had declared the structure unsafe. (b) The temple is facing a drastic depletion of its funds which had accumulated over the years due to offerings made by devotees. (c) The local corporation decided to donate a huge amount of money to the temple for its renovation. (d) The village housing the religious site has qualified priests to perform religious ceremonies. (e) A famous actor recently visited the temple and paid his respects to the deity.

31. Statements: The Income Tax authorities carried out raids atthree different business houses in the city last week. Which of the following can be a possible effect of the above statement? (a) The three business houses are regular defaulters in payment of their income tax. (b) The Income Tax department had received a tip off about the illegal activities going on in the three business houses. (c) The Government decided to look into the matter and has appointed an inquiry committee. (d) Other business houses took immediate action to clear off all their income tax dues in order to avoid raid on their establishments. (e) The authorities intend to conduct raids in several other business houses in the vicinity.

32. As a step to regulate private hospitals, the state health department is framing rules to ensure all such hospitals are registered with it. Which of the following can be a possible cause of the above statement? (a) The department realised the private hospitals charge much less for treatment as compared to government hospitals. (b) Government run hospitals do not maintain the same standards as private hospitals. (c) The department realised that several hospitals were rejecting cases starting lack of infrastructure. (d) Apart from the number of doctors, nurses and beds, the kind of procedure a hospital can carry out based on its infrastructure will also be registered and detailed. (e) Private hospitals nor registering with the department shall be forced to do so and will have to pay hefty penalties.

33. Statement: HSBC Holdings Plc is shutting its private banking unit in India, marking the exit of another foreign bank from the cut-

Critical Reasoning - IPM

Page 7: QUESTION BANK - edumentor.co.inedumentor.co.in/Downloads/2018/Extra Content... · of India's oldest and most important uranium mine, ... Punjab banned e-cigarettes last year and Maharashtra

throat wealth management business in Asia's third-largest economy. Which of the following may be a probable cause of shut down of HSBC unit in India? (a) Higher operating ratio in India (b) Saturation of Indian market (c) Increase of the price of dollar against rupee (d) Political Instability in the country (e) All of the above

34. Statement: Retail inflation rose for the 4th month in a row in November, accelerating to a 14-month high of 5.41 per cent. Which of the following may be a probable reason for the increase in retail inflation? (a) Sharp increase in food prices (b) Decline in the price of pulses (c) Tight monetary policy by the RBI (d) Decline of crude oil prices in international market (e) Only a) and c)

35. Statement: Coal imports came down from 136.6 million tons in April-November 2014 to 119.9 (MT) in 2015. Which of the following may be a probable reason(s) for the decline in coal imports? (a) Unprecedented increase in output of the fossil fuels by state-owned coal India Ltd. (b) Shut down of coal-based power plants in the country. (c) Production companies have started using costlier fuel. (d) A large number of importers are not interested in import of coal. (e) Only a) and b)

36. Statement: The Government of India has decided to lower the interest rate on small savings schemes offered by the Post Office and on PPF account offered by the banks and the Post Office. Which of the following may not be a probable reason(s) for lowering the interest rate on small savings schemes? (A) Banks are not able to fully pass the cut by the RBI to consumers by lowering interest rates on loans in equal proportion. (B) Most people move towards small savings schemes offered by the Post Office and banks instead of depositing in bank FD, which offers lower rate of interest. (C) Depositors want higher return in a short span of time. (D) Depositors have lost interest in savings schemes offered by the banks. (E) Banks have to generally provide lending to people while Post Office does not offer any lending facility. (a) Only (B), (C) and (D) (b) Only (B) and (C) (c) Only (A) and (D) (d) Only (C) and (E) (e) Only (A), (C) and (D)

37. Statement: The government has proposed that ATMs should not be replenished with cash after 8 PM in cities, and private cash transportation agencies must collect cash from the banks in the first half of the day for replenishing ATMs. Which of the following may be a probable reason(s) for the above step taken by the government? (a) Robbery of cash from ATM at night (b) Not enough guards deployed to keep ATM safe (c) Customers not feeling safe at night while withdrawing money from ATM

(d) ATM companies not following safety measures and guidelines issued by the RBI (e) Only (a) and (d)

38. Statement: Indian Railway has decided to lay 2800 km broad-gauge track in the new financial year. Which of the following will not be an effect of the above statement? (a) New areas will be connected on the map of the Indian Railways. (b) Employment opportunity may be generated. (c) Only (a) and (d) (d) Railways will receive income in the long term from the newly commissioned routes. (e) None of the above

39. Statement: People are investing their money in Post Office fixed deposits (FDs) and recurring deposits (RDs) instead of bank FDs and RDs. Which of the following may be the probable causes for the people doing so? (A) Bank deposits are not considered safe and secure. (B) Bank allows low interest rates on FD and RD in comparison to Post Office. (C) Most of the people are not associated with banking system in India. (D) Post Offices are widely available in comparison to banks. (a) Only (A), (C) and (D) (b) Only (A), (B) and (C) (c) Only (B), (C) and (D) (d) Only (D) and (C) (e) None of the above

40. Global Financial Integrity has highlighted that India wasamong the top 10 nations in terms of illicit outflows, with an outgo of $123 billion in the decade to 2010. Which of the following courses of action should best be taken? a) There should be a single global standard for automatic exchange of financial account information by various countries. b) A Special Investigation Team should be made to probe blackmoney. c) The currency of the country should be changed.d) The governments should join hands to tackle terrorism.e) None of these

41. There are at least 15 e-commerce sites that have ‘kart’appended to their name. Which of the following is the most possible reason for this phenomenon? a) Jabong, as everyone knows, has done pretty well for itself,despite having no ‘kart’ in its name. b) What matters over time is service quality and not the name.c) Along with the name, it is the business model, execution and offering which make the brand difficult to be cloned or impacted. d) These are not cases of blatant copyright infringement.e) None of these

Critical Reasoning - IPM

Page 8: QUESTION BANK - edumentor.co.inedumentor.co.in/Downloads/2018/Extra Content... · of India's oldest and most important uranium mine, ... Punjab banned e-cigarettes last year and Maharashtra

Course of Action 42. Statement: Anti-cancer drugs in the state of Bihar are being sold at a price higher than MRP. Which of the following courses of action should be taken against the drug seller? (a) License of all drug sellers in the city should be cancelled. (b) Such sellers should be fined and a warning in this regard should be given. (c) All the sellers should be put behind bars. (d) The government should sell the anti-cancer drugs as its own stores. (e) None of the above

43. Statement : A train has derailed near a station while moving over a bridge and fell into the river. Courses of Action : I) The Railway Authorities should clarify the reason of the accidentto the Government. 2) The Government should allocate founds to compensate the destruction caused. 3) New safety devices should be immediately installed along all the risky railway tracks 4) Floating jackets should be made available in trains like in aeroplanes 5) Medical assistance should be sent to the accident site for helping the affected people

44. Statement: Exporters in the capital are alleging thatcommercial banks are violating a RBI directive and still operate and using demonetised money at international markets Courses of Action : 1) The officers concerned in the commercial banks are to be suspended. 2) The RBI Should be asked to stop giving such orders like demonetisation which hampers business. 3) The RBI Should be asked to give stern warning and put penalty on commercial banks violating norms. 4) RBI should send clear information brochures to the commercialbanks regarding the demonetisation. 5) Commercial banks should follow the orders in the ‘spirit of the words’

45. Statement: A survey carried out by the BMC of more than 22,000 people has revealed that in the last two years a third of them have suffered pothole damage to their cars - and the situation looks set to worsen thanks to 30% more potholes being reported on our road network year on year. Every year during monsoon, the condition of most of the roads in the city deteriorates causing immense problem to commuters. Courses of action taken by BMC: (A) The civic body should include a heavy penalty clause while awarding future contracts for road repairs. (B) The civic officials in charge of maintenance of city roads should be asked to explain why the condition of the roads worsens every year. (C) General public should avoid taking their vehicles out during monsoon. (D) All contractors should be instructed to repair the roads immediately. (E) Newer technologies should be researched in collaboration with engineering colleges to find ways to make roads with less pot holes

46. Recently a youth was shifted to another hospital from hospital ABC due to food poisoning. The patient had been admitted in hospital ABC for Malaria. “Because no outside food is allowed in the hospital premises, we are very sure that it was the food provided by the hospital staff that led to the food poisoning.” - A statement by the parents of the youth. Which of the following can be a course of action to avoid such a mishap? (1) The parent of the patients should be given a compensation to ensure that the hospital is not criticised in any way. (2) The reason for the food poisoning should be identified and the food provided in the hospital should be inspected regularly. (3) Local vendors which provide fruits, vegetables and other eatables to the hospital must be questioned. (4) CCTV cameras should be installed in the hospital cafeteria to ensure that spoilt food is not provided to the patients. (5) Patients should be asked to bring food from their houses rather than provide it at the hospital so that if such a case occurs again the hospital will not be responsible.

47. Statement: Company XYZ is hiring every year students fromcollege D, however since past two years managers of company are not happy with the students of college D as they resign from within a few months of employment for better salaries. Which of the following can be a course of action by HR managers? (1) The pay package for college D students should be revised and retention bonus be given on completion of one year (2) College D must be blacklisted and no students be recruited (3) Management of college D should be informed about the situation and asked to talk to students joining XYZ (4) Recruits of college D must be asked to sign a bond to stay with company for minimum 5 years (5) Immediately salary of college D recruits be hiked by an appropriate percentage.

Caselet based Directions Two questions: 48. Statement: In order to prevent catastrophic damages to its assets, Railways is going for offshore procurement of high-powered sensor-based equipment, a first-of-its-kind for the public transporter, to detect on tracks and rolling stock. Which of the following inference drawn is definitely true according to the above statement? (a) Railways will equip this system in selected trains on a trial basis in the current fiscal as it was one of the Rail Budget proposals (b) The sensor-based equipment will monitor the condition of tracks, coaches, wagons and locomotives (c) The sensor based equipment will send early signal to the control room in case of any deficiency (d) The system will be fitted on wheels and the control room will receive signals through vibrations (e) Only (a) and (d)

49. Which of the following has been assumed in the given information? (An assumption is something that is supposed or taken for granted.) (a) Accidents will be minimised with the regular maintenance of tracks (b) The safety aspect of track and rolling stock will be enhanced

Critical Reasoning - IPM

Page 9: QUESTION BANK - edumentor.co.inedumentor.co.in/Downloads/2018/Extra Content... · of India's oldest and most important uranium mine, ... Punjab banned e-cigarettes last year and Maharashtra

(c) Railways will be able to know the exact condition of tracks through micro-analysis of data and this will reduce the possibility of sudden catastrophic failures (d) Railway will prepare a predictive maintenance schedule for its tracks (e) Only (a) and (b)

Two questions directions: 50. Statement: The country’s largest lender State Bank of Indiahas cut its home loan rate to 9.4% with effect from April 1 from 9.5%. Which of the following will be a necessary effect of the cut in the interest rate by SBI? (a) The amount of retail loans will come down. (b) The customer base of SBI will increase. (c) Others banks will also cut down their home loan rates. (d) The profit of SBI will increase. (e) The NPA of the SBI will increase with the distribution of retail loans.

51. Which of the following can be inferred from the given information? (An inference is something that is not directly stated but can be inferred from the given information.) (a) Home loans will become cheaper. (b) Interest rate on other savings products will come down. (c) More people in the country will have their own homes. (d) Economic growth in the country will increase. (e) None of the above

Directions three questions: 52. Statement: As per the report released by the UNDP, the Human Development Index (HDI) rank of India has made improvements in life expectancy at birth, which has increased to 68 years in 2014 from 67.6 in, the previous year and 53.9 years in 1980. But when it comes to gender index, 80% of women are unbanked. Which of the following may be a probable reason for the maximum number of women being deprived of banking? (a) Higher illiteracy rate in the country (b) Less number of quality government schools in the country (c) Females are not interested in higher education (d) Women are more prone to household activities. (e) None of the above

53. Which of the following can be inferred from the statementgiven above? (An inference is something which is not directly stated but can be inferred from the given facts.) (a) The country does fairly well on the gender index. (b) India has made improvement in healthcare. (c) Life expectancy of people has decreased. (d) Illiteracy in the country is the main problem in India. (e) None of the above

Directions for two questions: 54. Mobile technology has played great role in growth and development of society. Earlier cellphone was used as a medium of conversation only. Now mobile phones also support a wide variety of other services, such as, texting, email, internet access etc. The price of mobile phones is also decreasing and people are being encouraged to buy a mobile phone set at a cheaper rate. The mobile technology and smartphones have the capabilities of

handling video calls, sharing large files. Mobile technology had made it more efficient to conduct business. Video calls and taking photographs have become possible as mobile phone has inbuilt camera. Therefore, there is no need to carry around a camera everywhere you go. Which of the following cannot be inferred from the given information? (An inference is something that is not directly stated but can be inferred from the given information). (a) One can share photos and videos via mobile phones provided that the other person has a similar device. (b) Many features are being added to mobile phones now-a-days. (c) The other features of mobile phones are not as useful as the built in camera. (d) Mobile phones can be used for purposes other than making calls. (e) Technological advances are taking place in fields other than cellphones as well.

55. Which of the following can be concluded from the given information? (a) Buying a phone with a camera is more convenient than buying two different devices. (b) Mobile phones that are basic last longer than the ones with added features. (c) Not many people are interested in clicking pictures with mobile phones. (d) It is possible to share picture with someone having a similar camera. (e) No one will buy cameras now onwards.

Directions for two questions: 56. Read the following information carefully and answer the questions which follow: Supermarkets are growing at a fast pace the Kirana stores. Kirana stores are such places where customer go to purchase their necessities. In place of five Kirana stores one or two supermarkets are being established. It has been found that customers’ requirements are looked after by trained staff. It has been found that food products are low life products which are manufactured from local manufactures. These products are typically purchased by the customer on the assurance. The market is appealing to supermarkets and retail outlet owners are sitting up their business in other areas where there are less Kirana stores. Which of the following can be a good argument in favour of shopping from Kirana stores instead of supermarkets? (a) People prefer supermarkets because they offer a larger range of products, ie products other than FMCG and they can buy everything under one roof. (b) People end-up buying other irrelevant things along with those on their shopping lists in supermarkets and then they have to stand in long queues at the billing counters. (c) Most Kirana stores are closed atleast one day in a week whereas supermarkets are open 365 days a year. (d) Kirana stores do not accept debit and credit cards. (e) Very few Kirana stores sell products at a bargained price.

57. Which of the following can be inferred from the given information? (An inference is something that is not directly stated but can inferred from the given information).

Critical Reasoning - IPM

Page 10: QUESTION BANK - edumentor.co.inedumentor.co.in/Downloads/2018/Extra Content... · of India's oldest and most important uranium mine, ... Punjab banned e-cigarettes last year and Maharashtra

(a) Most supermarkets and retail outlet owners choose to set up business in areas that have very few Kirana stores. (b) People tend to trust retail outlets and supermarkets when it comes to buying high value products as opposed to buying them from local shops. (c) It there are two or more Kirana stores in a typical residential area the competition among them is very high. (d) Kirana stores owners are vying to acquire franchisee of supermarkets as the local shops have lost their charm and business. (e) Kirana stores do not sell the products which consumers have on their regular lists, so they have to depend on supermarkets.

Caselet based 5 questions 58. Study the following information carefully and answer the questions given below.

Cotton acreage in India during the current year has fallen by 15% as cotton growers have moved on to cultivation of other cash crops. This is the result of the cotton glut in world markets in post-September 2008 and the consequent slowdown in the world economy. But this scenario brought with it benefits to one segment of the industry yarn manufacturers as they got higher prices for their produce. Some yarn manufacturers too had stuck up on low-priced cotton last year. The combined effect of all this is evident in the rise in net profits and net margins of yarn manufacturers.

Which of the following is an inference which can be made from the facts stated in the above paragraph? (a) The cotton industry grew tremendously post-September 2008. (b) The yarn manufacturers have marginally suffered during post September 2008 period. (c) India was the largest cotton producer earlier. (d) Cotton production will surely grow in upcoming years. (e) None of these

59. Which of the following conclusions can be drawn from the facts stated in the above paragraph? (a) Now cotton acreage will never face a crisis like the crisis of September 2008. (b) There has been a huge drop in the supply of cotton during the current year. (c) USA is the largest producer and consumer of cotton. (d) Cotton growers are no more interested in production of cotton. (e) None of these

60. What we can say about the following statement? “There has been shortage of cotton in the world market last year.” (a) Definitely true (b) Definitely false (c) Probably true (d) Probably false (e) Nothing can be said

61. Which of the following will strengthen the following statement? “The farmers will again grow cotton next year due to increased price of cotton yarn.” (a) Govt plans to increase subsidy on cash crops including cotton. (b) There is a high alert in entire Europe due to terrorist threats. (c) Alternative cash crops are evergreen and unaffected by an economic slowdown.

(d) Govt is planning to import more cotton from abroad. (e) None of these

62. Which of the following inferences can be drawn from the statements in the above passage? (a) Indian economy has been unaffected by any global slowdown. (b) India was affected most in the world due to global economy slowdown. (c) The world economy witnessed an upward trend during pre-September 2008. (d) Govt should discourage the production of cotton and encourage shifting to other cash crops. (e) None of these

Sectional test 15 marks 63. This question consists of a statement and two assumptions numbered I and II given below it. (An assumption is something which is not directly given but has been assumed in the given information.) You have to find out which of the given assumptions numbered I and II are implicit in the given statement.

Statement: Exercising does not necessarily mean gymnasium with high-technology machines. After all, people living in villages are well-maintained and are hardly overweight I. Gymnasiums are not very readily available or used in villages. II. People living in villages do not believe in the use of high-technology gymnasiums. (a) Both I and II have been assumed in the given information. (b) Either I or II has been assumed in the given information. (c) Only I has been assumed in the given information. (d) Neither I nor II has been assumed in the given information. (e) Only II has been assumed in the given information.

Read the given information and answer the question. 64. “Although the State government has been funding College XYZ for the past 10 years, it has decided not to do so this year.” – Local news report. Which of the following may NOT be a reason for the decision of the government to discontinue the funding of the said college this year? (a) College XYZ has doubled the fees charged from students this year, as a result of which it made three times profit as compared to other State-run colleges. (b) The college has received the permission this year to lend its campus for marriages and other social events and this would be an additional source of income for the college. (c) It was recently found out that most of the bills submitted about the usage of the money received by the college as government funds are fabricated and show much more amount than the actual expenditure. (d) Despite repeated warning from the authorities for past two years, the college has continued to reserve some seats for students who are willing to pay huge donation in addition to fees. (e) Although the performance of students of XYZ has improved a little in academics this year, yet it has deteriorated drastically in sports and other extracurricular activities.

65. Statement: Some 11,000 MW of thermal power capacity in the country is lying idle because electricity distribution companies

Critical Reasoning - IPM

Page 11: QUESTION BANK - edumentor.co.inedumentor.co.in/Downloads/2018/Extra Content... · of India's oldest and most important uranium mine, ... Punjab banned e-cigarettes last year and Maharashtra

are not drawing the quantum agreed; they are preferring to source it from the spot market, where power is cheaper. Which of the following can be inferred from the given information? (An inference is something that is not directly stated but can inferred from the given information). (a) Cheap power is supplied mostly by independent power producers. (b) Private companies produce more power when there is a peak in demand and remain idle when demand falls. (c) Companies prefer to buy power from cheaper sources. (d) Thermal power is the biggest source of power generation in the country. (e) None of the above

66. Statement: The ministry of railways has decided to develop 400 identified railway stations under the innovative Swiss Challenge method; it has decided to go ahead with the development of a few stations on its own. Which of the following has been assumed in the given statement? (An assumption is something that is not directly stated but is supposed or taken for granted.) (a) Railways has decided to revamp the railway stations with the help of private sector. (b) There are several methods to develop a railway station. (c) Railways has decided to upgrade a few stations with the help of the public sector. (d) Railways has decided to develop stations into commercial hubs along with all the amenities for passengers. (e)Only (a) and (b)

67. Statement: The government has unveiled a new Prime Minister Crop Insurance Scheme with premium to be paid by farmers as low as 1.5 percent of sum assured for all rabi crops and 2 percent for kharif crops. Which of the following will be an effect on the farmers with the launch of the new crop Insurance scheme? (a) Farmers will have to pay hardly any premium for their crops. (b) Farmers will not have to face financial constraints when there is a loss of crops. (c) Farmers will focus more on production with the launch of this scheme. (d) India’s crop production will boost up. (e) Only (b) and (c)

Directions Two statements 68. Statement: Rising concerns about pollution in major Indian cities have led to a dramatic increase in sales of home air purifiers. New Delhi, which has been called the most polluted city in the world by the WHO, is not only the biggest buyer of air purifiers but is also the city that is seeing the fastest growth in sales of these devices. Which of the following has been assumed in the given statement? (An assumption is something that is not directly stated but is supposed or taken for granted.) (a) An air purifier removes contaminants from the air in a room. (b) Air purifier is beneficial for those with allergies, asthma and other respiratory problems. (c) People are concerned about the rising pollution in the city. (d) Major consumer electronics brands have launched newer models to make air purifiers affordable for the mass market.

(e) None of the above

69. Which of the following can be concluded from the abovestatement? (a) Indoor air is 10 to 30 times more polluted than outdoor air, and pollutants include dust mites, bacteria, fungi, viruses and pollen. (b) Delhi has been market as the most polluted city in the country by several reputed bodies. (c) The highest demand for air purifiers is from Delhi. (d) The sales of air purifier increase due to rise in air pollution. (e) Only (c) and (d)

70. Statement: A postman from a post office B has reportedlystacked thousands of letters in his house for two years and has not distributed even a single letter. Which of the following courses of action should be taken against the postman by the head of post office B? (a) The department should suspend him for five years. (b) The department should give him a chance to improve himself. (c) The department should imprison him for 10 years. (d) He should be sacked from the office as he has not discharged his duty properly. (e) A strict action should be taken against him if found guilty.

71. Statement: Though no heavy rain has been received in the cityand water is receding from most areas in Chennai and massive relief operations are underway, the city is staring at an outbreak of epidemics with tones of stinking garbage littering the streets as bright sunshine further eased the situation. Which of the following has been assumed in the given statement? (An assumption is something that is not directly stated but is supposed or taken for granted.) (a) Massive rains are the major cause of epidemics in the cities. (b) Improper drainage system in Chennai is the major cause of flood in the city. (c) Chennai needs proper planning to overcome heavy rains in the city. (d) Garbage in the city is the major concern of epidemics. (e) None of these

72. Statement: The Delhi government has proposed an‘odd/even’ formula for cars. The cars with odd number plate and even number plate will play on roads on alternate days. The move is planned to be rolled out from January 1, 2016. Which of the following will definitely be an effect of the decision taken by the Delhi government? (a) Pollution of the city will decline. (b) Sales of car will reduce in Delhi. (c) People of Delhi will buy two cars. (d) There will be no traffic jams on the Delhi road. (e) All of these

73. Statement: The aviation ministry has asked all airports in the country to install solar energy plants on the lines of the Kochi airport, the world’s first aerodrome that runs completely on solar power. Which of the following can be inferred from the given information? (An inference is something that is not directly stated but can be inferred from the given information.)

Critical Reasoning - IPM

Page 12: QUESTION BANK - edumentor.co.inedumentor.co.in/Downloads/2018/Extra Content... · of India's oldest and most important uranium mine, ... Punjab banned e-cigarettes last year and Maharashtra

(a) The airports will be able to save thousands of rupees in electricity bills. (b) Airports in the country will generate electricity through solar energy. (c) The efficiency of airport in the country will increase. (d) Government will achieve its target of 170 Gigawatt of renewable energy. (e) Pollution at the airports will reduce.

74. Statement: Government of India’s Smart City mission is facing a serious hurdle as not even a single state has submitted its smart-city plan to the Union urban development ministry. Which of the following may be probable reason(s) for no submission of smart-city plan by the states? (A) Non-availability of free land with the states. (B) States not having sufficient fund to develop smart city (C) Strict environmental laws imposed by the Central Government (a) Only C and A (b) Only B and C (c) Only A and B (d) All of the above (e) None of the above

75. Statement: In a landmark verdict, the Supreme Court said thatilliterates should be debarred from contesting elections saying that it is a reasonable restriction on people’s constitutional right to contest elections. Which of the following can be inferred from the given information? (An inference is something that is not directly stated but can be inferred from the given information? (a) Good education is a must to contest elections. (b) Knowledge is power and knowledge comes from education. (c) The verdict will increase the literacy rate in the country. (d) Good people will be selected in the parliament. (e) None of the above

76. Statement: Dense fog disrupted movement of several passenger trains under East Central Railway (ECR) division. Many trains on Patna-New Delhi-Patna route were delayed due to dense fog. Which of the following has been assumed in the given statement? (An assumption is something that is not directly stated but is supposed or taken for granted.) (a) Fogs are the major reason for the cancellation of train in the winter season. (b) Fogs are the major reason for the delay of trains in winter season. (c) Only the trains operating under ECR get delayed in the winter season. (d) Revenue of ECR decreased in the months of winter. (e) None of the above

77. Statement: The National Green Tribunal (NGT) has imposed acomplete ban on plastic bags in the pilgrimage towns of Haridwar and Rishikesh, slamming the authorities for lackadaisical attitude in keeping the Ganga clean. Which of the following will be a strong argument in favor of the step taken by NGT? (A) NGT wants to make Ganga pollution-free and allow devotees to take a dip and collect clean water.

(B) NGT was quite dissatisfied as the situation on the banks of the Ganga River was pathetic. (C) Plastic bags are one of the major causes of the degradation of water quality in Ganga. (D) Shopkeepers were warned by NGT not to use any kind of plastic bags. (a) Only (C) and (D) (b) Only (A), (B) and (C) (c) Only (A), (B) and (D) (d) Only (A) and (B) (e) Only (B), (C) and (D)

TEST 2 1. Statement: Numbers of students qualified for IITs from state Xare comparatively higher than the number of students from state Y. Which of the following may indicate that the result of the state Y is not in line with the general trend? 1) The students of state X are qualitatively better than those of the State Y 2) The school curriculum of state Y is very tough3) In the past years the share of seats in IITs from state Y were always remained high. 4) Only 1 and 25) Other than given options

2. Statement: Now a day most of the candidates recruited in private companies are directly hired from campuses of various business schools and engineering colleges. Which of the following substantiates the fact stated in the above statement? 1) Most of the bright students only preferred campus interview.2) Now campus interview becomes a trend in hiring process in theprivate companies 3) Campus interview is the only process to hire talented people4) Both 1 and 25) Other than given options

3. Statement: The prices of the onion and other vegetables havesubstantially increased due to the prolonged drought situation in some part and heavy downpour in some other part of the country during the monsoon season. Which of the following could be a logical course of action to be pursued? 1) The government should set up a committee to review the alarming situation 2) The government should ban on the export of the onion and vegetables 3) Government should import onion and vegetables in adequate amount 4) Only 2 and 35) Only 1 and 3

4. Statement: Due to oversupply in global markets, a slump in demand, and OPEC countries’ decision not to curb production, crude oil prices have fallen to a four-year low. Which of the following may be an effect of the above cause? 1) Urban People will buy more vehicle as prices of the fuels hasfallen. 2) The fall in international oil prices will reduce subsidies that help sustain the domestic prices of oil products.

Critical Reasoning - IPM

Page 13: QUESTION BANK - edumentor.co.inedumentor.co.in/Downloads/2018/Extra Content... · of India's oldest and most important uranium mine, ... Punjab banned e-cigarettes last year and Maharashtra

3) Country’s current account deficit will substantially reduce.4) Only 1 and 25) Only 2 and 3

5. Statement: Crops acreage in India during the current year hasfallen substantially as the encroachment of human habitats over the cropland has increased and this will lead to fall in the production of food grains and vegetables. Which of the following may be the effect of the above statement? 1) The prices of food grains and other essential commodities mayincrease compared to previous years 2) Government will import food grains in huge quantity fromabroad 3) Lack of investments in agricultural development has played acrucial role in this levelling of yield decrease. 4) Only 1 and 25) All of the above

Directions (Q. 6-10) Read the following information carefully and answer the given questions: The civic authority of city X has appealed that all those who have come in contact with the patient suffering from dreaded infectious swine flu disease be quarantined in their house and appealed to the people to take precautionary action.

(A) Nobody should be quarantined unless they are tested and found to be infected by the swine flu H1N1 virus.

(B) The local citizen group may respond to the appeal of the civic authority and will go for a test detecting such virus.

(C) Swine flu virus is highly contagious, allowing it to spread quickly from person to person. A simple sneeze may cause thousands of germs to spread through the air

(D) Quarantining people in their house may be control the spread of the dreaded disease which have claimed hundreds of lives across the nation.

(E) The civic authority has appealed to citizens to cooperate with the medical staff while they are in isolation ward.

(F) The government should spread awareness about the disease among the masses thorough campaigning and engagement at the community level through inter-personal communication.

6. Which of the statements (A) (B) (C) (D) (E) and (F) can be inferred from the facts/information given in the statement? (An inference is something which is not directly stated but can be inferred from the given facts) 1) Only A 2) Only B3) Only C4) Only D5) Only F

7. Which of the statements (A) (B) (C) (D) (E) and (F) would be astrong argument can be made on the basis of the above statement? 1) Only A and B2) Only A and C

3) Only A 4) Only C5) All A, B and C

8. Which of the statements (A), (B), (C), (D), (E) and (F) mentionedabove can be an assumption from the facts/information given the statements? (An assumption is something that is supposed or taken for granted) 1) Only A 2) Only B3) Only C4) Only D5) Only E

9. Which of the following could be a valid course of action to this problem? 1) Only A 2) Only B3) Only C4) Only D5) Only F

10. Which of the following conclusion can be drawn from the facts stated in the above paragraph? 1) Only A 2) Only B3) Only C4) Only D5) Only E

11. Statement: The new financial year will begin with a new set ofcharges and higher penalties for customers of private banks. Which of the following can be inferred from the facts given in the statement? (a) People should use digital channels and move away from physical instruments such as cheques. (b) Customers will be less attracted towards private banks. (c) Banks will inform the account holders through emails or SMS about the imminent charges. (d) The charge for non-maintenance of average minimum quarterly balance is the most widely applied charged by banks. (e) RBI will impose heavy penalty on banks for not resolving customer complaints.

12. Statement : The Maharashtra government has announced thatthe sale of gutkha will be declared as a non-bailable offence. The ban will be applicable on all types of tobacoo-laced chewing products. Which of the following inferences is definitely true according to the above statement? (a) The government will cancel the licences of offenders. (b) Shopkeepers will smuggle gutkha from neighbouring states. (c) Other states will also ban gutkha. (d) Offenders will face rigorous imprisonment. (e) People will pay higher charges for gutkha.

13. Statement: The Delhi Government has decided to deploymarshals on a hundred Delhi Transport Corporation (DTC) buses and “dark spots” for making public transport and public places safer for the women of the Capital.

Critical Reasoning - IPM

Page 14: QUESTION BANK - edumentor.co.inedumentor.co.in/Downloads/2018/Extra Content... · of India's oldest and most important uranium mine, ... Punjab banned e-cigarettes last year and Maharashtra

Which of the following inferences is Probably False according to the above statement? (a) Women will feel safer in public transport and public places. (b) Women will use public transport instead of their own vehicles. (c) Crime against women will decrease. (d) Delhi will be declared as the safest capital for women. (e) None of these

14. Statement: Railways have doubled the cost of platform ticketsto Rs. 10 with effect from April 1, 2015. Which of the following may be the best reason for the above decision taken by the Railways? (a) Less revenue collection from platform tickets. (b) To check the number of passengers in peak season (c) Unnecessary crowd at the platform (d) Less fine imposed against the person entering the platform (e) All of these.

15. Statement: Candidates from topnotch foreign and private banks have made a beeline for being considered for the post of managing director and chief executive officer for public sector banks. Which of the following can be inferred form the given information? (An inference is something that is not directly stated but can be inferred form the given information.) (a) Private-bank candidates have a greater interest in public sector banks. (b) Public sector banks pay higher salary than private banks. (c) Public sector bank employees are not much qualified compared to private sector bank employees. (d) Private-bank candidates are more qualified and aggressive in business. (e) Positive atmosphere created by the new government.

TEST 3 – 15 marks

Q1. This question consists of a statement and two assumptions numbered I and II given below it. (An assumption is something which is not directly given but has been assumed in the given information.) You have to find out which of the given assumptions numbered I and II are implicit in the given statement.

Statement: Exercising does not necessarily mean gymnasium with high-technology machines. After all, people living in villages are well-maintained and are hardly overweight I. Gymnasiums are not very readily available or used in villages. II. People living in villages do not believe in the use of high-technology gymnasiums. (a) Both I and II have been assumed in the given information. (b) Either I or II has been assumed in the given information. (c) Only I has been assumed in the given information. (d) Neither I nor II has been assumed in the given information. (e) Only II has been assumed in the given information.

Q2. Read the given information and answer the question. “Although the State government has been funding College XYZ for the past 10 years, it has decided not to do so this year.” – Local news report. Which of the following may NOT be a reason for the decision of the government to discontinue the funding of the said college this year?

(a) College XYZ has doubled the fees charged from students this year, as a result of which it made three times profit as compared to other State-run colleges. (b) The college has received the permission this year to lend its campus for marriages and other social events and this would be an additional source of income for the college. (c) It was recently found out that most of the bills submitted about the usage of the money received by the college as government funds are fabricated and show much more amount than the actual expenditure. (d) Despite repeated warning from the authorities for past two years, the college has continued to reserve some seats for students who are willing to pay huge donation in addition to fees. (e) Although the performance of students of XYZ has improved a little in academics this year, yet it has deteriorated drastically in sports and other extracurricular activities.

Q3. Statement: Some 11,000 MW of thermal power capacity in the country is lying idle because electricity distribution companies are not drawing the quantum agreed; they are preferring to source it from the spot market, where power is cheaper. Which of the following can be inferred from the given information? (An inference is something that is not directly stated but can inferred from the given information). (a) Cheap power is supplied mostly by independent power producers. (b) Private companies produce more power when there is a peak in demand and remain idle when demand falls. (c) Companies prefer to buy power from cheaper sources. (d) Thermal power is the biggest source of power generation in the country. (e) None of the above

Q4. Statement: The ministry of railways has decided to develop 400 identified railway stations under the innovative Swiss Challenge method; it has decided to go ahead with the development of a few stations on its own. Which of the following has been assumed in the given statement? (An assumption is something that is not directly stated but is supposed or taken for granted.) (a) Railways has decided to revamp the railway stations with the help of private sector. (b) There are several methods to develop a railway station. (c) Railways has decided to upgrade a few stations with the help of the public sector. (d) Railways has decided to develop stations into commercial hubs along with all the amenities for passengers. (e)Only (a) and (b)

Q5.Statement: The government has unveiled a new Prime Minister Crop Insurance Scheme with premium to be paid by farmers as low as 1.5 percent of sum assured for all rabi crops and 2 percent for kharif crops. Which of the following will be an effect on the farmers with the launch of the new crop Insurance scheme? (a) Farmers will have to pay hardly any premium for their crops. (b) Farmers will not have to face financial constraints when there is a loss of crops. (c) Farmers will focus more on production with the launch of this scheme.

Critical Reasoning - IPM

Page 15: QUESTION BANK - edumentor.co.inedumentor.co.in/Downloads/2018/Extra Content... · of India's oldest and most important uranium mine, ... Punjab banned e-cigarettes last year and Maharashtra

(d) India’s crop production will boost up. (e) Only (b) and (c)

Directions (6-7): Study the following information carefully and answer the questions given below: Statement: Rising concerns about pollution in major Indian cities have led to a dramatic increase in sales of home air purifiers. New Delhi, which has been called the most polluted city in the world by the WHO, is not only the biggest buyer of air purifiers but is also the city that is seeing the fastest growth in sales of these devices.

Q6. Which of the following has been assumed in the given statement? (An assumption is something that is not directly stated but is supposed or taken for granted.) (a) An air purifier removes contaminants from the air in a room. (b) Air purifier is beneficial for those with allergies, asthma and other respiratory problems. (c) People are concerned about the rising pollution in the city. (d) Major consumer electronics brands have launched newer models to make air purifiers affordable for the mass market. (e) None of the above

Q7. Which of the following can be concluded from the above statement? (a) Indoor air is 10 to 30 times more polluted than outdoor air, and pollutants include dust mites, bacteria, fungi, viruses and pollen. (b) Delhi has been market as the most polluted city in the country by several reputed bodies. (c) The highest demand for air purifiers is from Delhi. (d) The sales of air purifier increase due to rise in air pollution. (e) Only (c) and (d)

Q8. Statement: A postman from a post office B has reportedly stacked thousands of letters in his house for two years and has not distributed even a single letter. Which of the following courses of action should be taken against the postman by the head of post office B? (a) The department should suspend him for five years. (b) The department should give him a chance to improve himself. (c) The department should imprison him for 10 years. (d) He should be sacked from the office as he has not discharged his duty properly. (e) A strict action should be taken against him if found guilty.

Q9. Statement: Though no heavy rain has been received in the city and water is receding from most areas in Chennai and massive relief operations are underway, the city is staring at an outbreak of epidemics with tones of stinking garbage littering the streets as bright sunshine further eased the situation. Which of the following has been assumed in the given statement? (An assumption is something that is not directly stated but is supposed or taken for granted.) (a) Massive rains are the major cause of epidemics in the cities. (b) Improper drainage system in Chennai is the major cause of flood in the city. (c) Chennai needs proper planning to overcome heavy rains in the city. (d) Garbage in the city is the major concern of epidemics. (e) None of these

Q10. Statement: The Delhi government has proposed an ‘odd/even’ formula for cars. The cars with odd number plate and even number plate will play on roads on alternate days. The move is planned to be rolled out from January 1, 2016. Which of the following will definitely be an effect of the decision taken by the Delhi government? (a) Pollution of the city will decline. (b) Sales of car will reduce in Delhi. (c) People of Delhi will buy two cars. (d) There will be no traffic jams on the Delhi road. (e) All of these

Q11. Statement: The aviation ministry has asked all airports in the country to install solar energy plants on the lines of the Kochi airport, the world’s first aerodrome that runs completely on solar power. Which of the following can be inferred from the given information? (An inference is something that is not directly stated but can be inferred from the given information.) (a) The airports will be able to save thousands of rupees in electricity bills. (b) Airports in the country will generate electricity through solar energy. (c) The efficiency of airport in the country will increase. (d) Government will achieve its target of 170 Gigawatt of renewable energy. (e) Pollution at the airports will reduce.

Q12. Statement: Government of India’s Smart City mission is facing a serious hurdle as not even a single state has submitted its smart-city plan to the Union urban development ministry. Which of the following may be probable reason(s) for no submission of smart-city plan by the states? (A) Non-availability of free land with the states. (B) States not having sufficient fund to develop smart city (C) Strict environmental laws imposed by the Central Government

(a) Only C and A (b) Only B and C (c) Only A and B (d) All of the above (e) None of the above

Q13. Statement: In a landmark verdict, the Supreme Court said that illiterates should be debarred from contesting elections saying that it is a reasonable restriction on people’s constitutional right to contest elections. Which of the following can be inferred from the given information? (An inference is something that is not directly stated but can be inferred from the given information? (a) Good education is a must to contest elections. (b) Knowledge is power and knowledge comes from education. (c) The verdict will increase the literacy rate in the country. (d) Good people will be selected in the parliament. (e) None of the above

Q14. Statement: Dense fog disrupted movement of several passenger trains under East Central Railway (ECR) division. Many trains on Patna-New Delhi-Patna route were delayed due to dense fog.

Critical Reasoning - IPM

Page 16: QUESTION BANK - edumentor.co.inedumentor.co.in/Downloads/2018/Extra Content... · of India's oldest and most important uranium mine, ... Punjab banned e-cigarettes last year and Maharashtra

Which of the following has been assumed in the given statement? (An assumption is something that is not directly stated but is supposed or taken for granted.) (a) Fogs are the major reason for the cancellation of train in the winter season. (b) Fogs are the major reason for the delay of trains in winter season. (c) Only the trains operating under ECR get delayed in the winter season. (d) Revenue of ECR decreased in the months of winter. (e) None of the above

Q15. Statement: The National Green Tribunal (NGT) has imposed a complete ban on plastic bags in the pilgrimage towns of Haridwar and Rishikesh, slamming the authorities for lackadaisical attitude in keeping the Ganga clean. Which of the following will be a strong argument in favor of the step taken by NGT? (A) NGT wants to make Ganga pollution-free and allow devotees to take a dip and collect clean water. (B) NGT was quite dissatisfied as the situation on the banks of the Ganga River was pathetic. (C) Plastic bags are one of the major causes of the degradation of water quality in Ganga. (D) Shopkeepers were warned by NGT not to use any kind of plastic bags. (a) Only (C) and (D) (b) Only (A), (B) and (C) (c) Only (A), (B) and (D) (d) Only (A) and (B) (e) Only (B), (C) and (D)

Critical Reasoning - IPM

Page 17: QUESTION BANK - edumentor.co.inedumentor.co.in/Downloads/2018/Extra Content... · of India's oldest and most important uranium mine, ... Punjab banned e-cigarettes last year and Maharashtra

IPM - IIM

CorrectionSentence

Page 18: QUESTION BANK - edumentor.co.inedumentor.co.in/Downloads/2018/Extra Content... · of India's oldest and most important uranium mine, ... Punjab banned e-cigarettes last year and Maharashtra

DIRECTIONS (Qs. 1-54): In the following questions, in the given sentences, a part of the sentence is underlined. Beneath each sentence, four d erent ways of phrasing the underlined part are indicated. Choose the best alternative.

1. e government has given subsidies to the Navratnas butthere is no telling whether the subsequent one will do.(a) whether the subsequent government will do so.(b) if the government to follow will accept the policy(c) if the government to follow will adhere to the policy(d) no telling whether the subsequent one will do so

2. Rahul Bajaj has done a great job of taking the company to itspresent status, but it is time that he let go of the reins.(a) let go of the reins(b) stepped down(c) let go o the reins(d) delegated responsibility

3. With the pick-up in the standard of education, expensiveprivate schools have started blooming up in every corner ofthe country.(a) started blooming in every corner of the country(b) have started mushrooming all over the country(c) have mushroomed all over the country(d) have blossomed all over the country

4. It is important that whatever else happens, these two factorsshould not be messed around with.(a) It is important that(b) It is a fact that(c) It should be urgently understood that(d) It should be understood that

5. It must be note that under no circumstance should thecompany go in for diversi cation.(a) It must be noticed that(b) It must be noted that(c) It must be pointed out that(d) It should be noticed that

6. British Airspace has been focusing on building Europeanlinks.(a) concentrating on creating European links.(b) pursuing ways of building European connectivity.(c) stressing on building European links.(d) focusing on forging European links.

7. e appetite of banks for funds was lost under the onslaughtof the slowdown, corporate refused to borrow-even as bankdeposit ourished.(a) bank deposits ourished(b) bank deposits swelled(c) bank deposits were enhanced(d) bank deposits ummoxed

8. He did many mischiefs.(a) made many a mischiefs(b) made much mischief(c) Committed many mischiefs(d) No correction required

9. e main point of his speech was well understood.(a) that he spoke (b) in the speech of his(c) made when he spoke (d) No correction required

10. e indecisive man was readily persuaded to change hismind again.(a) was persuaded ready (b) was ready to persuade(c) was ready persuaded (d) No improvement

11. e teacher asked the intruder who was he and why was heoccupying his chair.(a) who he was and why he was(b) who he was and why was he(c) who he had been and why he had been(d) No correction required

12. e custom of many centuries ago origin is slowlydisappearing.(a) which was originated ago many centuries(b) originating for many centuries(c) which orginated many centuries ago(d) with many centuries of origin

13. He stayed back so that it can look as if he was unaware of the entire incident.(a) may look (b) would look(c) will look (d) No correction required

14. e local library has recommended that the books put upfor the used book sale should be in good condition andshould have no writing in them or be underlined.(a) and should have no writing in them or be underlined(b) and should not have writing in them or not be underlined(c) and contain no writing or underlining(d) without containing writing or underlining

15. e news of her employment soon circulated around thesmall town.(a) circulated round the small town(b) circulated in the small town(c) was circulating across the small town(d) was circulating within the small town

16. It is the craziness for speeding that is maddening that isresponsible for many motor accidents.(a) the craziness for speeding that is mad(b) the mad craze for speed(c) the mad craze for speeding(d) the craze for speeding that is maddening

17. If they cooperate together by dividing up the work, theyshall be over with the work faster.(a) if they cooperate together by dividing the work(b) if they cooperate by dividing up the work(c) if they cooperate by dividing the work(d) if they cooperate together by division of work

18. Knowing the area was prone to earthquakes, all the buildings were reinforced with additional steel and concrete.(a) Having known that the area was prone to earthquakes(b) Since they knew the area was prone to earthquakes(c) Since the area was known to be prone to earthquakes(d) Being prone earthquakes

SENTENCE CORRECTION

Sentence Correction - IPM

Page 19: QUESTION BANK - edumentor.co.inedumentor.co.in/Downloads/2018/Extra Content... · of India's oldest and most important uranium mine, ... Punjab banned e-cigarettes last year and Maharashtra

19. He sailed for New York on Monday, arriving there onSaturday for the much-awaited inauguration of the newhospital.(a) and arrived there on Saturday for the much-awaited

inauguration of the new hospital.(b) arriving there on Saturday for the inauguration of the

much-awaited new hospital.(c) arriving there for the inauguration of the much awaited

new hospital on Saturday.(d) and arrived here on Saturday for the long awaited

inauguration of the new hospital. 20. A er trying to convince him for a long time, I realized that

he was one of those people who never listens to reason.(a) he was one of those people who never listen to reason.(b) he was one of those people who never .listen to reasoning(c) he is one of those people who never listen to reason.(d) he is one of those people who never listens to reason.

21. Scientists have long recognized the promise of researchinvolving human embryos and foetuses for the advancement of basic science as well as for the development of life savingvaccines and therapies.(a) the promise of research involving human embryos and

foetuses for the advancement of(b) the promise of research involving human embryos and

foetuses to advance(c) the promising research which involves human embryos

and foetuses for the advancing of(d) the promising research, which involves human embryos

and foetuses in advancing 22. By nding, dating, and interpreting the trash and treasures

of ancient generations, our curiosity about our past as aspecies is satis ed in archaeology, and shows us how wehave become what we are.(a) generations, our curiosity about our past as a species is

satis ed in archaeology, and shows(b) generations satisfy, in archaeology, our curiosity about

our past as a species and show(c) generations, our past as a species satis es our curiosity

in archaeology and shows(d) generations, archaeology satis es our curiosity about

our past as a species and shows 23. Full of speculation that the United States was in a ‘housing

bubble’ that was about to burst, the popular press ran feature stories depicting the irrational run-up in home prices and apotential crash which was very likely.(a) stories depicting the irrational run-up in home prices

and a potential crash which was very likely.(b) stories about the irrational run-up in home prices and

that there would be a potential crash.(c) Stories that describe about the irrational run-up in

home prices and about the potential for a crash.(d) stories about the irrational run-up in home prices and

about the potential for crash. 24. Seeing that there was an ongoing sale in one of her favourite

stores, Seeta made a bee line for it immediately a er entering the mall.(a) made a bee’s line for(b) make bees lined to(c) made a bee line to(d) No correction required

25. Sharon made it to work in the nicks of times, or else shewould have missed the meeting.(a) nick of time (b) nicked time(c) nick of timeliness (d) nick and time

26. Varun was on cloud nine a er having stood rst in his class.(a) in ninth cloud (b) on nine clouds(c) a cloudy nine (d) cloud on nine

27. Vithal had a habit of pass the buck when it came to important issues at work.(a) pass to bucking (b) passing buck(c) passing the buck (d) pass buck

28. Puneet raked his brains and tried to nd an answer to atricky question given in the paper but couldn't nd one.(a) rake his brain (b) racked his brains(c) racked brains (d) raked brain

29. US secretary of state made it clear that time running outfor diplomacy over Iran's nuclear programme and said thattalks aimed at preventing Tehran from acquiring a nuclearweapon would resume in April.(a) runs out (b) was running out(c) ran out (d) run

30. While the war of the generals rage on, somewhere in smalltown India, wonderful things are happening, quietly andminus fanfare.(a) rage (b) raging(c) rages on (d) raged on

31. According to WWF, the small island nation of Samoa wasthe rst in switch o its lights for Earth Hour.(a) rst to switch of (b) the rst to switch o(c) the rst of switch o (d) rst in switch of

32. e campaign is signi cant because not just the youths aredirectly appealing to the world but because their e ortschallenge the chimera of normalcy in the area.(a) not just because (b) just not because(c) not just (d) because just

33. e doctor's association has threatened to go on inde nitestrike support of their teachers.(a) on supporting to (b) to supporting(c) for support (d) in support of

34. If one prays honestly and sincerely, God will listen to one’sprayer.(a) If one pray (b) One if prays(c) If one will praying (d) No correction required

35. Not only I but also my parents are looking forward to seeyou soon.(a) am looking forward to seeing(a) are looking to see forward(c) are looking forward seeing(d) are looking forward to seeing

36. ey are waiting for her since morning.(a) are waiting (b) are awaiting to(c) have been a waiting (d) have been waiting for

37. If he had asked me, I would have helped him.(a) If he ask (b) had he asked(c) He had asked (d) No correction required

Sentence Correction - IPM

Page 20: QUESTION BANK - edumentor.co.inedumentor.co.in/Downloads/2018/Extra Content... · of India's oldest and most important uranium mine, ... Punjab banned e-cigarettes last year and Maharashtra

38. I am sure that scientists must discover a cure for AIDS bythe end of next decade.(a) will have discovered (b) might discovered(c) have discovered (d) must discover

39. It is important that whatever else happens, these two factorsshould not be messed around with(a) It is important that(b) It is a fact that(c) It should be urgently understood that(d) It should be understood that

40. e appetite of banks for funds was lost under the onslaughtof the slowdown, corporates refused to borrow- even asbank deposits ourished(a) bank deposits ourished(b) bank deposits swelled(c) bank deposits were enhanced(d) bank deposits ummoxed

41. e MP rose up to say that, in her opinion, she thought theWoman’s Reservation Bill should be passed on unanimously(a) rose to say that she thought the women’s reservation bill

should be passed(b) rose up to say that, the women’s reservation bill should

be passed on(c) rose to say that, in her opinion, she thought that the

women’s reservation bill should be passed(d) rose to say that, in her opinion, the women’s reservation

bill should be passed on 42. Doctors suggest that patients su ering on account of high

blood pressure take regular exercise stay away from nicotine and eat plenty of vegetables.(a) who su er from high blood pressure should be taking

regular exercise, stay away from nicotine and eat plenty of vegetables.

(b) su ering from high blood pressure have to be takingregular exercise, stay away from nicotine and eating plenty of vegetables.

(c) su ering high blood pressure should take regular exercise, stay from nicotine and eat plenty of vegetables.

(d) su ering from high blood pressure take regular exercise, stay away from nicotine and eat plenty of vegetables.

43. e irate resident dragged the builder to the court for failing to transfer the title of the property in their names.(a) to the court for failing to transfer the title of the property

to their names.(b) to court for failing to transfer the title of the property to

their names.(c) to court in failing to transfer the title of the property in

their names.(d) to court for failing to transfer the title to the property in

their names. 44. One of the reasons rich students opt for a foreign degree

is because they may have lost out in the race for a seat in agood, local institution.(a) because they may have lost out in the race for a seat in a

good, local institution.

(b) that they may have lost out in the race for a seat in good, local institution.

(c) for the reason that they may have lost out in the race for a seat in a good, local institution.

(d) so that they may have lost out in the race for a seat in a good, local institution.

45. During the literary renaissance of the 1920s, a large numberof new writers — William Faulkner, Ernest Hemingway,John Dos Passos, and F. Scott Fitzgerald — sought to recordthe inner life of Americans and to scrutinize the Americandream, the dream that anyone can earn his own fortuneand live happily ever a er through hard work, which hadbecome tarnished.(a) the dream that anyone can earn his own fortune and

live happily ever a er through hard work, which had become tarnished

(b) the tarnished dream that anyone can make his own fortune and live happily ever a er through hard work

(c) the tarnished dream that anyone can, through hard work, make his own fortune and live happily ever a er

(d) the dream that anyone can earn his own fortune and live happily ever a er, though tarnished, through hard work

46. Two recent statements on the tenure of university professors er con icting points of view: those that say that lifetime

tenure ensures academic freedom and those that saythat lifetime tenure encourages professional laziness andirresponsibility.(a) those that say that lifetime tenure ensures academic

freedom and those that say that lifetime tenure encourages professional laziness and irresponsibility

(b) some declare that lifetime tenure ensures academic freedom, and others say that it encourages professional laziness and irresponsibility

(c) saying that lifetime tenure either ensures academic freedom or encourages irresponsible laziness

(d) one emphasizes the academic freedom that tenure ensures, and the other stresses the professional laziness and irresponsibility it encourages

47. In the con ict between the Israelis and the Palestinians,the refusal of each side to acknowledge each other as alegitimate national movement is closer to the heart of theproblem than is any other issue.(a) the refusal of each side to acknowledge each other as a

legitimate national movement is closer to the heart of the problem than

(b) that the refusal of each side to acknowledge another as a legitimate national movement is closer to the heart of the problem as

(c) the refusal of each side to acknowledge another as a legitimate national movement is closer to the heart of the problem than

(d) that the refusal of each side to acknowledge another as a legitimate national movement is closer to the heart of the problem than

Sentence Correction - IPM

Page 21: QUESTION BANK - edumentor.co.inedumentor.co.in/Downloads/2018/Extra Content... · of India's oldest and most important uranium mine, ... Punjab banned e-cigarettes last year and Maharashtra

48. Agencies studying discrimination in housing haveexperimentally proved that minority clients are o endiscouraged as prospective buyers of residential real estateand the antidiscrimination legislation of recent decadeswere only mitigating, rather than abolishing, inequity inhousing practices.(a) the antidiscrimination legislation of recent decades

were only mitigating, rather than abolishing, inequity in housing practices

(b) in recent decades, the antidiscrimination legislation only mitigated, rather than abolishing, inequity in housing practices

(c) that antidiscrimination legislation of recent decades has only mitigated, rather than abolished, inequity in housing practices

(d) that, in recent decades, antidiscrimination legislation has only mitigated, rather than abolishing, housing practices’ inequity

49. Unlike German Shepherds or Doberman pinchers, there isan unwillingness on the part of many people to believe thatpit bulls might be fully domesticated.(a) Unlike German Shepherds or Doberman pinchers,

there is an unwillingness on the part of many people to believe that pit bulls might be fully domesticated.

(b) Many people, willing to believe that German Shepherds and Doberman pinchers might be fully domesticated, are unwilling to believe the same of pit bulls.

(c) Unlike German Shepherds or Doberman pinchers, pit bulls bring out an unwillingness in many people tobelieve that they might be fully domesticated.

(d) Many people are unwilling to believe that pit bulls might be fully domesticated even while they are willing to believe that German Shepherds and Doberman pinchers might be.

50. In contrast to accredited universities that can grantdegrees and whose students can be eligible for Federal Aid,nonaccredited colleges, while still able to enroll studentsand to provide instruction, cannot provide their studentswith the same types of services.(a) nonaccredited colleges, while still able to enroll students

and to provide instruction, cannot provide their students with the same types of services

(b) nonaccredited colleges cannot provide their students with the same types of services, though still able to enroll students and provide instruction

(c) enrolling students and providing instruction, while unable to provide their students with the same types of services, are nonaccredited colleges

(d) the services of nonaccredited colleges, while including the ability to enroll students and provide instruction, do not include the same types of services

51. Although the losing party disapproves of every aspect ofthe opponent’s platform, they later conceded that theremust be a basis for a cooperative government and agreed tocompromise.

(a) disapproves of every aspect of the opponent’s platform, they later conceded that there must be a basis

(b) disapproves of every aspect of the opponent’s platform, it later conceded that there must be a basis

(c) disapproved of every aspect of the opponent’s platform, they later conceded that there had to bo some basis

(d) had disapproved of every aspect of the opponent’s platform, it later conceded that there must be a basis

52. Large and experienced rms are more e cient at acquiringsmaller and distressed rms than are large and inexperienced rms, and converting them to pro table ventures.

(a) Large and experienced rms are more e cient atacquiring smaller and distressed rms than large and inexperienced rms.

(b) Large and experienced rms are more e cient than large and inexperienced rms at acquiring smaller and distressed rms

(c) Large and experienced rms, acquire smaller and distressed rms more e ciently than large andinexperienced rms

(d) Large and experienced rms, more e cient than large and inexperienced rms at acquiring smaller anddistressed rms

53. e economic growth increased from 7 to 9 percent inNovember 2010, supporting the expectations that industrialgrowth rate in October-December quarter more thandoubled that of the 4 per cent growth rate in industrialgrowth for the previous quarter.(a) that industrial growth in the October-December

quarter, more than the doubled that of(b) of industrial growth in the October-December quarter,

it more than doubled(c) of industrial growth in the October-December quarter,

that it would more than double that of(d) that industrial growth in October-December quarter

would more than double. 54. As a result of surging nancial greed, the international

rating agencies upgraded the rating of the credit derivativeinstruments, and hence analysts recommended a strongbuy, ignoring the advice of Warren Bu ett who warnedthat these instruments would prove not only dangerous butine ective in the long-run(a) who warned that those instruments would prove to be

both dangerous and(b) warning that these instruments would prove not only

dangerous and also(c) warning that these instruments would prove itself to be

both dangerous and(d) who was warning that these instruments would prove

not only dangerous but

Sentence Correction - IPM

Page 22: QUESTION BANK - edumentor.co.inedumentor.co.in/Downloads/2018/Extra Content... · of India's oldest and most important uranium mine, ... Punjab banned e-cigarettes last year and Maharashtra

DIRECTIONS (Qs. 55 - 59) : In the following questions a part of the sentence is underlined. Below are given alternatives to the underlined part at (A), (B) and (C) which may improve the sentence. Choose the correct alternative. In case no improvement is needed your answer is (D). Mark your answer in the Answer-55. You shall have attended if the court had instructed you to

do so.(a) would have had to attend(b) would attend(c) would have to(d) No improvement

56. The relics of Greece over which such a great deal of evidencehas been collected should be preserved.(a) from which (b) on which(c) ascent which (d) No improvement

57. When the beverage was ready, they drank possibly as muchas they could.(a) as much as they possibly could(b) as much as possibly they could(c) as much as they could possibly(d) No improvement

58. A citizen is expected to give allegiance to his country oforigin.(a) homage (b) loyalty(c) obedience (d) No improvement

59. We were with daggers drawn despite attempts to understandeach other.(a) in (b) on(c) at (d) No improvement

DIRECTIONS (Qs. 60 - 69) : In the following questions, a sentence/part of the sentence is underlined. Below are given alternatives to the underlined sentence/part of the sentence at (a), (b) and (c) which may improve the sentence. Choose the correct alternative. In case no improvement is needed, your answer is (d).

60. The climate of Karnataka is cooler than Tamil Nadu.(a) is cooler to (b) is cooler than of(c) is cooler than that of (d) No improvement

61. The Tsunami victims suffered of cholera.(a) suffered from (b) suffered under(c) suffered in (d) No improvement

62. I gave to Sana the keys.(a) I gave (b) I gave to the(c) I gave the (d) No improvement

63. If he smokes less he might get rid of his cough.(a) If he smoked less he would get rid of his cough.(b) If he had smoked less he might get rid of his cough.(c) If he smokes less he might have got rid of his cough.(d) No improvement.

64. He compensated the loss to me.(a) He compensated the loss for me.(b) He compensated me to the loss(c) He compensated me for the loss.(d) No improvement.

65. As employees, we are accountable for our stakeholders.(a) accountable with (b) accountable to(c) accountable against (d) No improvement

66. Recently he had insured for a mediclaim policy.(a) He had recently insured for(b) Recently he insured for(c) He insured recently for(d) No improvement

67. Everyday, we usually had lunch at 1.30 p.m.(a) we have had usually(b) we have usually(c) we usually have(d) No improvement

68. All nations must rst become agricultural strong.(a) become agricultural strong(b) become strong agriculture(c) become agriculture strong(d) No improvement

69. An orangutan’s intelligence is as superior to that of man.(a) is more superior to(b) is superior to(c) is superior than that of(d) No improvement

DIRECTIONS (Qs. 70-79): In the following questions, a sentence/a part of the sentence is underlined. Below are given alternatives to the underlined part at (a), (b), (c) which may improve the sentence. Choose the correct alternative. In case no improvement is needed your answer is (d). Mark your answer in the Answer Sheet.

70. Why should you be despaired of your success of yourundertaking?(a) you despair of the success of your undertaking(b) you despair of success of undertaking(c) you be despaired of the success of your undertaking(d) No improvement.

71. As Rees was going to town in the High Street a savage dogattacked him and bit him.(a) going to town a savage dog attacked him and bit him

in the High Street(b) in the High Street a savage dog attacked him and bit

him in the town(c) going to town in the High Street a savage dog bit him

and attacked him(d) No improvement

72. Something is pretty here that Vineeta can wear to the party.(a) Something here is pretty(b) Something is here pretty(c) Here is something pretty(d) No improvement

Sentence Correction - IPM

Page 23: QUESTION BANK - edumentor.co.inedumentor.co.in/Downloads/2018/Extra Content... · of India's oldest and most important uranium mine, ... Punjab banned e-cigarettes last year and Maharashtra

73. I have dreamt all my life to own a beautiful maroon colouredcar.(a) of owning (b) to owning

(c) at owning (d) No improvement74. Sitting on the hill top, the sun went down watching before

him.(a) he watched the sun go down.(b) the sun went down with him watching(c) the sun went down when he watched(d) No improvement

75. The of ce is soon to be closed.(a) just to (b) about to(c) immediately to (d) No improvement

76. He has achieved nothing out of his way worth mentioning.(a) out of the way (b) by the way(c) in a big way (d) No improvement

77. I prevailed on him to vote for you.(a) to (b) at (c) upon (d) No improvement

78. Eager to pass his nal exams, studying was the students top priority.(a) the student’s top priority was studying.(b) the student made studying his top priority.(c) the top priority of studying was made by the student.(d) No improvement.

79. Mr. Dev will not go to the wedding reception without beingcalled.(a) if he is not invited(b) till he is invited(c) unless he is invited(d) No improvement

Sentence Correction - IPM

Page 24: QUESTION BANK - edumentor.co.inedumentor.co.in/Downloads/2018/Extra Content... · of India's oldest and most important uranium mine, ... Punjab banned e-cigarettes last year and Maharashtra

IPM - IIM

Comprehension

Reading

Reading Comprehension - IPM

Page 25: QUESTION BANK - edumentor.co.inedumentor.co.in/Downloads/2018/Extra Content... · of India's oldest and most important uranium mine, ... Punjab banned e-cigarettes last year and Maharashtra

Passage – 1

While many points are worth making in an evaluation of the single six-

year presidential term, one of the most telling points against the single

term has not been advanced. This kind of constitutional limitation on

elections is generally a product of systems with weak or non-existent

political parties.

Since there is no party continuity or corporate party integrity in such

systems, there is no basis for putting trust in the desire for re-election as

a safeguard against mismanagement in the executive branch. Better

under those conditions to operate on the basis of negative assumptions

against incumbents. I do not know if the earliest proposal for a single,

nonrepeatable term was made in the 1820s because that was a period of

severely weak political parties. But I do feel confident that this is a major

reason, if not the only reason, that such a proposal has been popular

since the 1940s.

Though the association of the non-repeatable election with weak

political parties is not in itself an argument against the limitation, the

fallout from this association does contribute significantly to the negative

argument. Single-term limitations are strongly associated with

corruption. In any weak party system, including the presidential system,

the onus of making deals and compromises, both shady and honourable,

rests heavily upon individual candidates. Without some semblance of

corporate integrity in a party, individual candidates have few

opportunities to amortize their obligations across the spectrum of elective

and appointive jobs and policy proposals. The deals tend to be

personalized and the payoffs come home to roost accordingly.

If that situation is already endemic in conditions of weak or non-

existent parties, adding to it the limitation against re-election means that

candidates and officials, already prevented from amortizing their deals

across space, are also unable to amortize their obligations temporally.

This makes for a highly beleaguered situation. The single six-year term

for presidents is an effort to compensate for the absence of a viable party

system, but it is a compensation ultimately paid for by further weakening

the party system itself.

Observers, especially foreign observers, have often noted that one

source of weakness in American political parties is the certainty of

election every two or four years, not only because any artificial limitation

on elections is a violation of democratic principles but also because when

elections are set in a certain and unchangeable cycle, political parties do

not have to remain alert but can disappear into inactivity until a known

point prior to the next election. To rigidify matters by going beyond the

determinacy of the electoral cycle to add an absolute rule of one term

would hang still another millstone around the neck of already doddering

political parties.

Reading Comprehension - IPM

Page 26: QUESTION BANK - edumentor.co.inedumentor.co.in/Downloads/2018/Extra Content... · of India's oldest and most important uranium mine, ... Punjab banned e-cigarettes last year and Maharashtra

1. Suppose that America adopted a single-term political system. Considering the

foreign observers mentioned in the passage. how would they be expected to

respond to such a development?

A. They would endorse it because it further strengthens American

democracy.

B. They would condemn it because it further limits American democracy.

C. They would neither endorse nor condemn it.

D. They would condemn it because it gives the President too much power.

E. They would endorse it because it will reduce corruption

2. According to the passage, which of the following is most likely to be true of a

political system with weak political parties?

A. Politicians appoint unqualified people to important posts.

B. Political parties favour frequent elections.

C. Political bargains are made by individual candidates.

D. Elections tend to occur with very great frequency.

E. It encourages politicians to be more honest

3. Which of the following, if true, would most weaken the author‘s claim about

single-term political systems?

A. The discovery that foreign observers like this system

B. The discovery that most politicians are honest

C. The discovery that Americans dislike this system

D. The discovery that parliamentary systems are more democratic

E. The discovery that politicians favour such a system

Reading Comprehension - IPM

Page 27: QUESTION BANK - edumentor.co.inedumentor.co.in/Downloads/2018/Extra Content... · of India's oldest and most important uranium mine, ... Punjab banned e-cigarettes last year and Maharashtra

Passage – 2

By regarding the expanding universe as a motion picture, you can easily

imagine ―running the film backward.‖ If you do so, you find the universe

getting smaller and smaller, and eventually you come to the moment

when its whole mass is crammed into an infinitely dense point. Before

that time it didn‘t exist, or at least it didn‘t exist in its present form.

Though there is some controversy about its exact age, most

cosmologists would be inclined to agree that the universe has existed for

about ten to twenty billion years. For scale, this can be compared to the

four-and-a-half-billion-year age of the solar system, the time since the

disappearance of the dinosaurs (sixty-five million years), and the age of

the human race (about three million years).

The event that marked the beginning of the universe was christened

the Big Bang; the term has now entered the vernacular of our culture.

Originally the name referred only to the single initiating event; now,

however, astronomers have come to use it to mean the entire

developmental process of the birth and expansion of the cosmos.

The simple statement that the universe had a beginning in time is by

now so obvious to astrophysicists that few give it a second thought. Yet it

is a statement that has profound implications. Most civilizations embrace

one of two opposite concepts of time. Linear time has a beginning, a

duration, and an end; cyclical time, as its name suggests, continues

around and around forever. In a universe that functions through cyclical

time, the question of creation never arises; the universe always was and

always will be. The minute you switch to linear time you immediately

confront the vexing question not only of creation, but also of the Creator.

Although there is no logical reason for the assumption, many people

believe that if something comes into existence, it must do so in response

to the actions of some rational being. Because of that belief,

astronomers, even though they resist becoming involved in theological

discussion, find themselves in one when they posit the Big Bang universe.

It puts them squarely in the middle of an age-old debate.

One common misconception about the Big Bang that should be

disposed of immediately is the notion that the universal expansion is

analogous to the explosion of an artillery shell. The galaxies are not like

bits of shrapnel speeding away from a central explosion. The raisin-in-

dough analogy is a more satisfactory way to think about the whole

process.

Reading Comprehension - IPM

Page 28: QUESTION BANK - edumentor.co.inedumentor.co.in/Downloads/2018/Extra Content... · of India's oldest and most important uranium mine, ... Punjab banned e-cigarettes last year and Maharashtra

1. In the context of the passage, the phrase "age-old debate" (line 31) refers

to:

A. the question of whether ―the Creator‖ created the universe.

B. the controversy over linear versus cyclical time.

C. the debate over the disappearance of the dinosaurs.

D. the disagreement over the movement of galaxies E. whether God exists or not

2. According to the passage, which of the following statements is NOT true?

A. Many people believe that a rational impetus created the universe.

B. The solar system was created immediately after the Big Bang.

C. The universe is larger today than it was in the past.

D. Different societies measure time differently.

E. Most cosmologists believe the universe to be 10 to 20 billion years old

3. Why does the author compare the universe to a motion picture?

A. illustrate that the universe has operated according to linear time.

B. demonstrate that the universe is actually older than most astronomers

believe.

C. show that galaxies were formed about five billion years ago.

D. prove that the universe was created by a rational being.

E. to show the analogy between ‗God‘ and a ‗director‘

Reading Comprehension - IPM

Page 29: QUESTION BANK - edumentor.co.inedumentor.co.in/Downloads/2018/Extra Content... · of India's oldest and most important uranium mine, ... Punjab banned e-cigarettes last year and Maharashtra

Passage – 3

Of course, in his attempts at field investigation, the historian is at the

disadvantage that the countryside has changed in many respects since

the period which he is studying. He is not permitted to use H.G. Wells‘s

time machine, to enable him to see it as it actually was. Inevitably he is

concerned in the main, if not exclusively, with literary and other

materials, which have survived from that stretch of the past which

interests him.

Old maps may be plans of cities, charts of sea coasts and estuaries,

cartularies of landed estates, or topographic delineations of land areas.

These clearly engage the interest of historians and geographers alike, and

they call for a combination of the methods and viewpoints of each. Maps

can be conceived of and considered in several quite different ways, being

properly regarded, and so assessed, as works of art—at best as objects of

colour, skill, form, and beauty. They may alternatively be regarded purely

for their cartographic aesthetic.

The main queries which then arise are the following: how is it that the

map-maker has carried out his task and with skill of what echelon and

with what degree of success has he done so? Such an inquiry falls to the

specialist field of historical cartography. An antiquarian map may also be

approached in a means akin to that of the student who conceives it as a

font contemporaneous with the time of its production. Thus, the historical

cartographer may seek to bring grist to his mill and to consider the map‘s

reliability as a satisfactory source of empirical evidence. By such means

also the regional historian, in his search for essentials about such past

matters as the availability of roads, the extent of enclosed farmland, or

the number and location of mines and quarries, is no less an interested

party.

The value of old maps as documents useful for historicity depends

necessarily on to what degree they depict and on how accurately. For

virtually all periods of pre-modern history some maps have survived to

serve as historiography, depicting, however imperfectly, certain features

of past geography. The work of Claudius Ptolemy—who lived in the 2nd

century A.D.—for centuries provided the basis for maps of the known

world and its major regions. Although many were drawn on the scientific

basis which he provided, they nevertheless embodied many errors—of

location, distance, and the shape of areas of land and sea.

The medieval portolan charts of the Mediterranean Sea and the later

charts which provided sailing directions, produced in Holland, were

accurate enough to be useful in practical navigation. Plans of important

cities of Europe, so well-drawn as to yield evidence of their earlier form

and extent, are notably offered in Braun and Hogenberg‘s Civitates Orbis

Terrarum, published at Cologne and, in England, in John Speed‘s plans of

cities. Similarly, John Ogilby‘s Britannia, Volume the First, appearing in

1675, gives detailed information of England's road system as it existed

nearly three centuries ago. However, few of the early maps approach

modern standards, which require accurate representation of distances

and of heights above mean sea-level and the use of carefully

Reading Comprehension - IPM

Page 30: QUESTION BANK - edumentor.co.inedumentor.co.in/Downloads/2018/Extra Content... · of India's oldest and most important uranium mine, ... Punjab banned e-cigarettes last year and Maharashtra

distinguished symbols. This is because it was not until the 18th century

that cartography, as an exact science, was born.

1. According to the passage, which of the following statements is/are NOT true?

I. Most maps produced before the 18th century are not as accurate as

maps produced after the 18th century.

II. The maps of Claudius Ptolemy were not used as a model by later map-

makers.

III. Historians have generally been uninterested in using maps as a tool to

learn about the past.

A. II only

B. III only

C. I and II

D. II and III

E. I, II and III

2. With which of the following statements would the author be most likely to

agree?

A. Old maps provide important information about the past, even if they

are somewhat misleading.

B. Modern maps, in general, are more accurate than maps produced in

the 18th century.

C. The maps in Braun and Hogenberg‘s book have no historical value

because of their errors.

D. Claudius Ptolemy‘s maps were the most accurate ever made prior to

the birth of modern cartography.

E. The field of cartography is on a downward spiral

3. According to the passage, all of the following would be considered maps

EXCEPT:

A. a drawing of Mediterranean sea lanes in the 2nd century B.C.

B. a drawing of Rome‘s city streets in the 4th century B.C.

C. a drawing of Northern hemisphere star constellations in the 5th

century A.D.

D. a drawing of Scottish farm boundaries in the 10th century A.D.

E. a drawing of a important sea routes in the 18th century

Reading Comprehension - IPM

Page 31: QUESTION BANK - edumentor.co.inedumentor.co.in/Downloads/2018/Extra Content... · of India's oldest and most important uranium mine, ... Punjab banned e-cigarettes last year and Maharashtra

Passage – 4

The person who, with inner conviction, loathes stealing, killing, and

assault, may find himself performing these acts with relative ease when

commanded by authority. Behaviour that is unthinkable in an individual

who is acting of his own volition may be executed without hesitation

when carried out under orders. An act carried out under command is,

psychologically, of a profoundly different character than spontaneous

action.

The important task, from the standpoint of a psychological study of

obedience, is to be able to take conceptions of authority and translate

them into personal experience. It is one thing to talk in abstract terms

about the respective rights of the individual and of authority; it is quite

another to examine a moral choice in a real situation. We all know about

the philosophic problems of freedom and authority. But in every case

where the problem is not merely academic there is a real person who

must obey or disobey authority. All musing prior to this moment is mere

speculation, and all acts of disobedience are characterized by such a

moment of decisive action.

When we move to the laboratory, the problem narrows: if an

experimenter tells a subject to act with increasing severity against

another person, under what conditions will the subject comply, and under

what conditions will he disobey? The laboratory problem is vivid, intense,

and real. It is not something apart from life, but carries to an extreme

and very logical conclusion certain trends inherent in the ordinary

functioning of the social world. The question arises as to whether there is

any connection between what we have studied in the laboratory and the

forms of obedience we have so often deplored throughout history. The

differences in the two situations are, of course, enormous, yet the

difference in scale, numbers, and political context may be relatively

unimportant as long as certain essential features are retained.

To the degree that an absence of compulsion is present, obedience is

coloured by a cooperative mood; to the degree that the threat of force or

punishment against the person is intimated, obedience is compelled by

fear. The major problem for the individual is to recapture control of his

own regnant processes once he has committed them to the purposes of

others. The difficulty this entails represents the poignant and in some

degree tragic element in the situation, for nothing is bleaker than the

sight of a person striving yet not fully able to control his own behaviour in

a situation of consequence to him.

The essence of obedience is the fact that a person comes to view

himself as the instrument for carrying out another‘s wishes, and he

therefore no longer regards himself as culpable for his actions. Once this

critical shift of viewpoint has occurred, all of the essential features of

obedience—the adjustment of thought, the freedom to engage in cruel

behaviour, and the types of justification experienced by the person

(essentially similar whether they occur in a psychological laboratory or on

the battlefield)—follow. The question of generality, therefore, is not

resolved by enumerating all of the manifest differences between the

psychological laboratory and other situations, but by carefully

Reading Comprehension - IPM

Page 32: QUESTION BANK - edumentor.co.inedumentor.co.in/Downloads/2018/Extra Content... · of India's oldest and most important uranium mine, ... Punjab banned e-cigarettes last year and Maharashtra

constructing a situation that captures the essence of obedience—a

situation in which a person gives himself over to authority and no longer

views himself as the cause of his own actions.

1. According to the passage, which of the following statements is NOT false?

A. People will never commit acts that they judge to be wrong.

B. People will always obey those who are in positions of authority over

them.

C. Obedience is not an important subject because it affects only a very

limited number of acts.

D. It is possible to study obedience through a laboratory experiment.

E. Obedience is not impacted by a cooperative mood

2. In the context of the points being made by the author in the passage, the

phrase ―absence of compulsion‖ (line 30) refers to:

A. the lack of punishment in psychological experiments.

B. obedience that is willingly given to one‘s superior.

C. the freedom to disobey the orders of those in authority.

D. one‘s ability to consider the moral implications of an act.

E. having the free will to do what one wants

3. Which of the following findings would serve to most WEAKEN the author‘s

claim in the passage about obedience to authority?

A. A study that concludes that most obedience to authority is motivated

by fear

B. A study that demonstrates that most authority figures in government

behave immorally

C. A study that shows that most people do not have strongly held ethical

values

D. A study that asserts that people with a college education are less

likely to obey authority figures than those with only a high school

education

E. A study that proves that fear is an overriding emotion for most human

beings

Reading Comprehension - IPM

Page 33: QUESTION BANK - edumentor.co.inedumentor.co.in/Downloads/2018/Extra Content... · of India's oldest and most important uranium mine, ... Punjab banned e-cigarettes last year and Maharashtra

Passage – 5

As formal organizations, business corporations are distinguished by their

particular goals, which include maximization of profits, growth, and

survival. Providing goods and services is a means to this end. If, for

example, a number of individuals (outsiders or even insiders) believe that

a company‘s aggressive marketing of infant formula in third world

countries is morally wrong, the company is unlikely to be moved by

arguments based on ethos alone as long as what it is doing remains

profitable. But if those opposed to the company‘s practice organize a

highly effective boycott of the company‘s products, their moral views will

soon enter into the company‘s deliberations indirectly as limiting

operating conditions. They can, at this point, no more be ignored than a

prohibitive increase in the costs of certain raw materials.

Although the concepts and categories of ethics may be applied to the

conduct of corporations, there are important differences between the

values and principles underlying corporate behaviour and those

underlying the actions of most individuals. If corporations are by their

nature end- or goal-directed how can they acknowledge acts as wrong in

and of themselves? Is it possible to hold one criminally responsible for

acts that if performed by a human person would result in criminal

liability?

The first case of this type to achieve widespread public attention was

the attempt to prosecute the Ford Motor Company for manslaughter as

the result of alleged negligent or reckless decision making concerning the

safety engineering of the Pinto vehicle. Although the defendant

corporation and its officers were found innocent after trial, the case can

serve as an exemplar for our purposes.

In essence, the prosecution in this case attempted to show that the

corporation had produced and distributed a vehicle that was known to be

defective at the time of production and sale, and that even after a great

deal of additional information accumulated regarding the nature of the

problems, the corporation took no action to correct them. The obvious

non-corporate analogy would be the prosecution of a person who was

driving a car with brakes known to be faulty, who does not have them

repaired because it would cost too much, and who kills someone when

the brakes eventually fail and the car does not stop in time. Such cases

involving individuals are prosecuted and won regularly.

If corporations have no concept of right or wrong because they are

exclusively goal-directed, can they be convicted in cases of this type, and

what purpose would be served by such a conviction? Perhaps we can

make a utilitarian argument for convicting corporations of such crimes.

The argument would be that of deterrence; conviction and punishment

would deter other corporations from taking similar actions under similar

circumstances. However, there appears to be considerable evidence that

deterrence does not work on corporations, even if, arguably, it works on

individuals. The possibility of being discovered and the potential

magnitude of the fine merely become more data to be included in the

analysis of limiting conditions.

Reading Comprehension - IPM

Page 34: QUESTION BANK - edumentor.co.inedumentor.co.in/Downloads/2018/Extra Content... · of India's oldest and most important uranium mine, ... Punjab banned e-cigarettes last year and Maharashtra

1. A claim that things have ethical value to corporations only insofar as they are

instrumental in furthering the ultimate goals of the corporation is:

A. necessarily true, given the information presented in the passage.

B. perhaps true, and supported by the information presented in the

passage.

C. perhaps true, but not supported by any information in the passage.

D. necessarily false, given the information presented in the passage.

E. a figment of the author‘s imagination

2. If a company that produced shampoo products opted to stop the routine

testing of its products on animals because it decided that it is wrong to cause

the animals pain, what effect would this have on the argument made in the

passage?

A. It would strongly support the argument.

B. It would support the argument somewhat, but not conclusively.

C. It would neither support nor substantially weaken the argument.

D. It would substantially weaken the argument.

E. It would weaken the argument only if the company is a government

owned company

3. Which of the following assertions would most strengthen the author‘s claim

that deterrence will not work on corporations?

A. The possibility of punishment does not deter many individuals from

committing crimes.

B. The penalties imposed on companies have amounted to a small fraction

of their profits.

C. Strict anti-pollution laws have cut down on the waste dumped by

companies into rivers.

D. The trial of a corporation is often extended over a period of several years

E. Corporation have a battery of lawyers protecting their interests

Reading Comprehension - IPM

Page 35: QUESTION BANK - edumentor.co.inedumentor.co.in/Downloads/2018/Extra Content... · of India's oldest and most important uranium mine, ... Punjab banned e-cigarettes last year and Maharashtra

Passage – 6

Few ideas are more deeply entrenched in our political culture than that of

impending ecological doom. Beginning in 1962, when Rachel Carson

warned that pollution was a threat to all human and animal life on the

planet, pessimistic appraisals of the health of the environment have been

issued with increasing urgency.

And yet, thanks in large part to her warnings, a powerful political

movement was born and a series of landmark environmental bills became

law. These laws and their equivalents in Western Europe, along with a

vast array of private efforts spurred by environmental consciousness that

Carson helped raise, have been a stunning success in both the United

States and Europe where environmental trends are, for the most part,

positive; and environmental regulations, far from being burdensome and

expensive, have proved to be strikingly effective, have cost less than was

anticipated, and have made the economies of the countries that have put

them into effect stronger, not weaker.

Recycling, which was a fringe idea a decade ago, is now a major

growth industry, and is converting more than twenty per cent of

America‘s municipal wastes into useful products. Emissions of

chlorofluorocarbons, which deplete the ozone layer, have been declining

since 1987. Dozens of American cities once dumped raw sludge into the

ocean. Today, instead of being dumped into the ocean, municipal sludge

is either disposed of in regulated landfills or, increasingly, put to good use

as fertilizer.

America‘s record of protecting species threatened with extinction,

which is often depicted as dismal, is in truth enviable. Since 1973, when

the Endangered Species Act took effect, seven animal species in North

America have disappeared. Several hundred others once considered

certain to die out continue to exist in the wild. A number of species,

including the bald eagle and the Arctic peregrine falcon have been or are

being taken off the priority-protection list.

It‘s true, of course, that some environmental programs are muddled.

For instance, the Endangered Species Act can have the unfair effect of

penalizing landholders who discover rare creatures on their property, by

prohibiting use of the land. In the main, though, conservation has been

an excellent investment. Thanks to legislation, technical advances, and

lawsuits that have forced polluters to pay liability costs, America‘s air and

water are getting cleaner, forests are expanding, and many other

environmental indicators are on the upswing.

Nevertheless, the vocabulary of environmentalism has continued to be

dominated by images of futility, crisis, and decline. Nor are

environmentalists the only people reluctant to acknowledge the good

news; advocates at both ends of the political spectrum, each side for its

reasons, seem to have tacitly agreed to play it down. The left is afraid of

the environmental good news because it undercuts stylish pessimism; the

right is afraid of the good news because it shows that governmental

regulations might occasionally amount to something other than

wickedness incarnate, and actually produce benefits at an affordable cost.

Reading Comprehension - IPM

Page 36: QUESTION BANK - edumentor.co.inedumentor.co.in/Downloads/2018/Extra Content... · of India's oldest and most important uranium mine, ... Punjab banned e-cigarettes last year and Maharashtra

1. Which of the following statements is false as it pertains to the information

given in the passage?

A. Chlorofluorocarbons no longer damage the ozone layer.

B. Technical advances have contributed to conservation.

C. Raw sludge is no longer a source of ocean pollution for the United

States.

D. Recycling has had an impact on landfill dumping.

E. Some environmental programs are muddled

2. Based on information in the passage, each of the following statements is a

plausible explanation of why pessimistic appraisals of the environment

continue to be issued EXCEPT:

A. environmentalists and politicians are unaware of the successes of the

movement.

B. an immense amount of work still needs to be done to save the

environment.

C. optimistic evaluations would have unwanted political repercussions.

D. environmentalists garner support by arousing concerns and fears.

E. selfish interests of certain groups of people

3. If the claims made in the passage are correct, how would politicians on the

political right be expected to react to America‘s program to protect

endangered species from extinction?

A. They would extol it because its success is not attributable to

governmental regulation.

B. They would extol it because its success refutes the pessimistic claims

of the political left.

C. They would criticize it because its success was due to costly

regulations.

D. They would criticize it because it has not shown any measurable

success.

E. They would be indifferent towards it

4. What is the main function of the 3rd paragraph in the passage?

A. to criticise industry for increased pollution

B. to urge the government to ban the dumping of effluents in rivers

C. to suggest that things are not bad as are made out to be by certain

groups of people

D. to describe the positive impact of efforts to control environmental

degradation

E. to provide an agenda for pollution control

Reading Comprehension - IPM

Page 37: QUESTION BANK - edumentor.co.inedumentor.co.in/Downloads/2018/Extra Content... · of India's oldest and most important uranium mine, ... Punjab banned e-cigarettes last year and Maharashtra

Passage – 7

While most archaeologists believe that primitive European societies were

patriarchal in both their social and religious structures, a new

controversial theory challenges these traditional views. This theory

suggests that during the Stone Age there thrived in and around Europe

peace-loving, matriarchal communities in which men and women lived

together as equals, respected nature, and worshipped a nurturing deity

called the Great Goddess.

The people of ―Old Europe‖—Europe from 7000 B.C. to 3500 B.C.—

lived in stable agricultural societies in which women headed clans and

men laboured as hunters and builders, but neither sex acted as a

dominant force with respect to the other. War was shunned and

craftspeople created comfortable dwellings and graceful ceramics instead

of weapons. Like the woman-centred social system, the religion of Stone

Age Europe focused on women in its veneration of the life-generating

Great Goddess and other female deities. Worship was closely linked to

the themes of respect for life and regeneration.

Proponents of this theory contend that this peaceful and harmonious

society was shattered by waves of Indo-European invaders in about the

year 3500 B.C., when marauders from the Russian steppes transformed

Europe from a peaceful, agrarian culture to one in which men dominated

women and wars raged. Social and sexual egalitarianism were replaced

by patriarchy and hierarchy, and warrior gods dethroned the Great

Goddess. With the widespread decimation of Old Europe, the goddess-

centred religion went underground. However, its symbols have

reappeared over the centuries in the forms of the female deities of

Greece and Rome, in the Virgin Mary, and in the belief in spiritual forces

lurking within the natural world.

The theory of the Great Goddess has been hailed by feminist social

critics, artists, and religious thinkers for providing an important

alternative to traditional, patriarchal mythologies and paradigms, as well

as for providing a new and more positive model for the human

relationship to the natural world.

Eminent anthropologist Ashley Montagu calls the theory ―a benchmark

in the history of civilization,‖ yet many other investigators into prehistoric

Europe consider the theory an unsubstantiated and idealistic version of

history. To a number of critics, the chief problem in this radical theory is

one of method. Traditional archaeologists, taking issue with unorthodox

speculation on ancient belief systems, contend that archaeological

evidence may tell us something about what people ate in the small

villages of prehistoric Europe, how they built their homes, and what they

traded, but cannot tell us much about what the dwellers of the ancient

world actually thought. To them, such speculation is illegitimate. The

most severe critics warn that, in blurring the distinction between intuition

and fact, proponents of the new theory have failed as scientists.

But supporters of the theory of a goddess-worshipping Old Europe

counter that such critiques reveal a certain narrow-mindedness on the

part of scientists rather than weaknesses on the part of their theory

Reading Comprehension - IPM

Page 38: QUESTION BANK - edumentor.co.inedumentor.co.in/Downloads/2018/Extra Content... · of India's oldest and most important uranium mine, ... Punjab banned e-cigarettes last year and Maharashtra

arguing that some degree of speculation is important, perhaps even

necessary, for the sake of progress in archaeology and other fields. This

element of speculation helps reveal the implications of a theory.

1. Which of the following would be contrary to what a proponent of the theory

of the Great Goddess most likely believes?

A. The available archaeological evidence does not rule out the idea that

Old European matriarchal communities existed.

B. The field of archaeology has been dominated in the past by male-

oriented scholarship.

C. Matriarchy is conducive to establishing a healthy relationship with the

natural world.

D. The decimation of Old European society wiped away all traces of the

Great Goddess religion.

E. Most men and women worshipped the Great Goddess

2. Based on the information in the passage, which of the following statements

about prehistoric European society would traditional archaeologists most likely

consider illegitimate?

A. The people were agrarian and not nomadic.

B. Food was cooked in clay vessels over a fire.

C. Arrows and spears were the most commonly used instruments of

warfare.

D. The people were worried about invasion.

E. They had adopted a more patriarchal model

3. Which of the following maxims seems most in agreement with the argument

that the supporters of the Great Goddess theory put forth in response to

criticism?

A. Those who live by the sword will die by the sword.

B. A mind is like a parachute in that it only works when open.

C. He who does not understand his opponent‘s arguments does not

understand his own.

D. The squeaky wheel gets the grease.

E. The early bird gets the worm

Reading Comprehension - IPM

Page 39: QUESTION BANK - edumentor.co.inedumentor.co.in/Downloads/2018/Extra Content... · of India's oldest and most important uranium mine, ... Punjab banned e-cigarettes last year and Maharashtra

Passage – 8

Most diseases or conditions improve by themselves, are self-limiting, or

even if fatal, seldom follow a strictly downward spiral. In each case,

intervention can appear to be quite efficacious. This becomes all the more

patent if you assume the point of view of a knowing practitioner of

fraudulent medicine.

To take advantage of the natural ups and downs of any disease (as

well as of any placebo effect), it‘s best to begin your treatment when the

patient is getting worse. In this way, anything that happens can more

easily be attributed to your wonderful and probably expensive

intervention. If the patient improves, you take credit; if he remains

stable, your treatment stopped his downward course. On the other hand,

if the patient worsens, the dosage or intensity of the treatment was not

great enough; if he dies, he delayed too long in coming to you.

In any case, the few instances in which your intervention is successful

will likely be remembered (not so few, if the disease in question is self-

limiting), while the vast majority of failures will be forgotten and buried.

Chance provides more than enough variation to account for the sprinkling

of successes that will occur with almost any treatment; indeed, it would

be a miracle if there weren‘t any ―miracle cures.‖

Even in outlandish cases, it‘s often difficult to refute conclusively

some proposed cure or procedure. Consider a diet doctor who directs his

patients to consume two whole pizzas, four birch beers, and two pieces of

cheesecake for every breakfast, lunch, and dinner, and an entire box of

fig bars with a quart of milk for a bedtime snack, claiming that other

people have lost six pounds a week on such a regimen. When several

patients follow his instructions for three weeks, they find they‘ve gained

about seven pounds each. Have the doctor‘s claims been refuted?

Not necessarily, since he might respond that a whole host of auxiliary

understandings weren‘t met: the pizzas had too much sauce, or the

dieters slept sixteen hours a day, or the birch beer wasn‘t the right

brand. Number and probability do, however, provide the basis for

statistics, which, together with logic, constitutes the foundation of the

scientific method, which will eventually sort matters out if anything can.

However, just as the existence of pink does not undermine the distinction

between red and white, and dawn doesn‘t indicate that day and night are

really the same, this problematic fringe area doesn‘t negate the

fundamental differences between science and its impostors.

The philosopher Willard Van Orman Quine ventures even further and

maintains that experience never forces one to reject any particular belief.

He views science as an integrated web of interconnecting hypotheses,

procedures, and formalisms, and argues that any impact of the world on

the web can be distributed in many different ways. If we‘re willing to

make drastic enough changes in the rest of the web of our beliefs, the

argument goes, we can hold to our belief in the efficacy of the above diet,

or indeed in the validity of any pseudoscience.

Reading Comprehension - IPM

Page 40: QUESTION BANK - edumentor.co.inedumentor.co.in/Downloads/2018/Extra Content... · of India's oldest and most important uranium mine, ... Punjab banned e-cigarettes last year and Maharashtra

1. In the context of the passage, its discussion of various medical conditions,

and the particulars of those conditions, the term self-limiting (lines 15-16)

refers to medical conditions that:

A. run a definite course that does not result in the patient‘s death.

B. impair the patient‘s ability to engage in everyday activities.

C. have a very high rate of mortality.

D. never shows improvement.

E. cannot be cured by medicine

2. According to the passage, which of the following is most likely to be the best

way to determine whether a practitioner‘s intervention is worthwhile or not?

A. Keep a record of the time it takes for a patient to respond to the

practitioner‘s treatment

B. Keep a record of the number of patients the practitioner has treated

successfully

C. Keep a record of the dosage that the practitioner employs in his

treatment

D. Keep a record of both the successes and failures of the practitioner

E. Keep a record of the different claims made by the practitioner

3. Based on the information in the passage, which of the following opinions

could most reasonably be ascribed to the author?

A. Too often nothing truly effective can be done to ameliorate the illness

of a patient.

B. There is no way that pseudoscience will ever be eliminated.

C. Beliefs can be maintained even in the absence of strong supporting

evidence.

D. Experience never forces one to reject any particular belief.

E. Quack doctors should be banned

Reading Comprehension - IPM

Page 41: QUESTION BANK - edumentor.co.inedumentor.co.in/Downloads/2018/Extra Content... · of India's oldest and most important uranium mine, ... Punjab banned e-cigarettes last year and Maharashtra

Passage – 9

Tribal immunity is the doctrine of sovereign immunity applied on behalf of

Native American tribes. Under the Indian Commerce Clause, Congress

has ―plenary‖ authority over the tribes. Courts have held that these tribes

cannot be sued without the consent of Congress. The doctrine of tribal

immunity, however, is a judicially created doctrine that the federal courts

have independently fashioned.

At least one Supreme Court Justice has noted the necessity of a more

principled analysis of the doctrine of tribal immunity, expressing ―doubts

about the continuing vitality in this day of the doctrine of tribal immunity

as it was enunciated in the case of the United States v. United States

Fidelity and Guaranty Co.‖ and ―the view that that doctrine may well

merit re-examination in an appropriate case.‖

The doctrine first emerged in the case of the United States v. United

States Fidelity and Guaranty Co., where the Supreme Court held ―Indian

nations exempt from suit without congressional authorization.‖ The

Supreme Court suggested two grounds for the doctrine. First, Native

American tribes enjoy immunity as a result of being recognized as

sovereigns.

Within the last decade, the court has reaffirmed this position, holding

that these tribes retain all sovereign powers except those ―expressly

terminated by Congress‖ and ―inconsistent with their status.‖ These

powers ―are not, in general, delegated powers granted by express acts of

Congress‖, but rather ―inherent powers of a limited sovereignty which has

never been extinguished.‖

A second basis for tribal immunity stems from the desire to protect

tribal resources. While the Supreme Court did not explicitly pronounce

the protection of tribal resources as a ground for its decision, it cited

cases in support of its ruling that were primarily concerned with such

protection. Unlike the immunities enjoyed by states, the federal

government and foreign countries, no limitations have been placed on the

scope of tribal immunity.

For instance, courts consistently hold that a Native American tribe‘s

immunity can be waived only by its express consent or the consent of

Congress. In contrast to other governments, implied waivers are

generally not recognized even in cases where commercial activity by a

tribe on or off its reservation has taken place. Similarly, the purchase of

insurance by a tribe does not serve to waive immunity. Tribal immunity

is, therefore, broader in this respect than is the immunity possessed by

states, the federal government, and foreign countries.

The proprietary acts of Native American tribes have not been

distinguished from the governmental functions of tribes, although this

distinction has been made in cases concerning other sovereigns. In fact,

some courts have specifically upheld that ―the fact that a tribe was

engaged in an enterprise private or commercial in character, rather than

governmental, is not material.‖ Thus courts continue to find a broader

Reading Comprehension - IPM

Page 42: QUESTION BANK - edumentor.co.inedumentor.co.in/Downloads/2018/Extra Content... · of India's oldest and most important uranium mine, ... Punjab banned e-cigarettes last year and Maharashtra

immunity for Native American tribes than is still recognized for any other

sovereign.

1. Which of the following legal decisions would most weaken the author‘s claim

about the immunity granted to Native American tribes?

A. A decision to permit a Native American tribe to sue a foreign

corporation

B. A decision to prevent a Native American tribe from suing the federal

government

C. A decision to permit a business corporation to sue a Native American

tribe

D. A decision to prevent the federal government from suing a Native

American tribe

E. A decision to permit a Native American tribe to sue another Native

American tribe

2. Based on information in the passage, which of the following statements is

NOT true?

A. It is more difficult to sue a Native American tribe than a business

corporation.

B. It is more difficult to sue the federal government than a Native

American tribe.

C. It is less difficult to sue a foreign government than a Native American

tribe.

D. It is less difficult to sue a state government than a Native American

tribe E. Tribal immunity has virtually no limits

3. Based on information in the passage, each of the following statements is a

plausible explanation of why the judicial system has not changed the rules

governing tribal immunity EXCEPT:

A. Native American tribes are sovereign entities that cannot be sued

without their consent.

B. the resources possessed by Native American tribes should remain

under tribal control.

C. Native American tribes have generally been unable to purchase

insurance.

D. the sovereign powers of Native American tribes differ from those of

other governments.

E. it is essential to protect the tribes‘ natural resources

Reading Comprehension - IPM

Page 43: QUESTION BANK - edumentor.co.inedumentor.co.in/Downloads/2018/Extra Content... · of India's oldest and most important uranium mine, ... Punjab banned e-cigarettes last year and Maharashtra

Passage – 10

As opera becomes more popular in America the scarcity of theatres and

the unconscionably costly logistics of the lyric stage make it difficult to

meet the demand. Many a good-sized and well-to-do community would

be able to operate and maintain a modest but live opera theatre, but are

unwilling to do so because it would unfavourably compare with the

splendours of New York‘s Metropolitan Opera.

It is not realized that the rich operatic culture of Italy and Germany is

mainly due to their many small municipal theatres which alternate

repertory theatre with opera. These circumstances have led to concert or

―semi-staged‖ performances which, formerly an exception, now occupy

entire companies expressly formed for this purpose. However, stage

music, real operatic music, often fails to exert its full power in the frozen

formality of the concert platform. In a true opera the particular charm

and power of the music does not come through without staging and

acting, for gesture is an expression of feeling, and the decor and

costumes summarize the external aspects, providing a vision of the whole

action. Both are to a considerable degree determined by the music, but

they also complement it.

An opera is a play in music. If it is presented in concert version, then

it should not offer a half-hearted gesture towards the theatre. Indeed,

the ―partly staged‖ performances are even more unsatisfactory than the

concert variety. The tenor is all excited, but you do not know why; the

soprano is obviously dying, but she remains on her feet. Nor does the

stationary chorus, its members turning the pages of their scores without

looking at the person they sing about, contribute to the illusion.

Different aesthetic laws of governance apply to concert music and

theatrical music, for they are incongruous worlds calling for an entirely

different sort of imagination from both performers and audience. Opera is

theatre, the most involved, elaborate, and exciting form of theatre. The

Italian term ―opera‖ is far more inclusive than its English interpretation,

for it embraces not only the musical score but the whole theatre, ―the

work.‖

Without the stage, paucity of musical ideas immediately becomes

evident, often painfully so. Take for instance Richard Strauss, some of

whose late operas are being performed in concerts. Strauss was a

composer who knew every facet of the lyric stage as few have known it,

yet what can be quite pleasant on the stage, even if it is not particularly

inventive, appears bare and contrived when removed from its natural

habitat.

Some may say that the end justifies the means. I can see merit in the

concert performance of an opera which otherwise could not hope to be

heard, or of one deficient in true theatrical qualities yet of genuine

musical value. But neither Strauss, nor Bellini, nor Donizetti qualifies for

such a role. Even if we forget the vital function of staging, it is practically

impossible, for purely musical reasons, to present such a work on the

concert platform. The large orchestra belongs in the pit; when placed on

Reading Comprehension - IPM

Page 44: QUESTION BANK - edumentor.co.inedumentor.co.in/Downloads/2018/Extra Content... · of India's oldest and most important uranium mine, ... Punjab banned e-cigarettes last year and Maharashtra

the stage, together with the singers, it makes their position almost

untenable, even when led by an experienced opera conductor.

1. Which of the following statements seems most in agreement with the attitude

of most ―good-sized and well-to-do‖ communities regarding opera?

A. Certain pleasures can only be appreciated by the educated.

B. Much can be achieved even if inherent limitations exist.

C. There is no sense in trying if you can‘t be among the best.

D. The opinions of your neighbours are more important than those of

strangers.

E. The early bird gets the worm

2. Based on the information in the passage, with which of the following

statements would the author most likely NOT agree?

A. Staging and acting are an integral part of the operatic work.

B. Some acting in a concert is better than no acting at all.

C. An opera is a much more involved production than is a concert.

D. Understanding the characters is essential to an appreciation of

operatic music.

E. Italy and Germany have small municipal theatres

3. The author discusses ―opera‖ in a very particular way in the fourth paragraph

of the passage. Implicit in the author‘s discussion of the term is the idea

that:

A. Italian words typically have broader meanings than English words.

B. the term ―opera‖ in English refers to only some part of the theatrical

work.

C. the same word can have different meanings in only two different

languages.

D. there is a fundamental difference between Italian and American opera.

E. American opera is way inferior to Italian opera

Reading Comprehension - IPM

Page 45: QUESTION BANK - edumentor.co.inedumentor.co.in/Downloads/2018/Extra Content... · of India's oldest and most important uranium mine, ... Punjab banned e-cigarettes last year and Maharashtra

Passage – 11

The last ice age has left its tell-tales written quite clearly across the

landscape. When Louis Agassiz first promulgated his theory that ice had

once covered the Swiss countryside, he looked to the valleys there that

retain glaciers to this day. Like other observers, he noted the presence of

strange boulders, called ―erratics,‖ tossed down in valleys like flotsam

after a flood had drained away. He saw the strange polish along the

bedrock—a sheen imparted as if by some massive swipe of sandpaper; he

saw the debris of rocks and boulders fringing the margin of existing

glaciers. He saw what can be seen still, markings in stone that indicated

that ice once flowed over vast stretches of land now clear and verdant.

The Australian climate historian L.A. Frakes has prospected through

various theories proposed to account for those early ice ages. He isn‘t

terribly enthusiastic about any of the possible culprits, but his choice for

the least unlikely of them all emerges out of the recent revival of what

was once a radically unorthodox idea: that continents drift over the face

of the planet. Frakes argues that the glaciers originated at sites near the

poles and that the ice ages began because the continents of the early

earth had drifted to positions that took more and more of their land

nearer to the polar regions.

More land near the poles meant that more precipitation fell as snow

and could be compacted on land to form glaciers. With enough glaciers,

the increase in the amount of sunlight reflected back into space off the

glistening white sheen of the ice effectively reduced the amount by which

the sun warmed the earth, creating the feedback loop by which the

growth of glaciers encouraged the growth of more glaciers. Rocks have

been found in North America, Africa and Australia whose ages appear to

hover around the 2.3 billion-year-old mark. That date and their spread

are vague enough, however, to make it almost impossible to determine

just how much of the earth was icebound during the possible range of

time in which each of the glacial deposits was formed.

Uncertainties about both the timing and the extent of these glaciers

also muddy the search for the cause of the ancient ice ages. The record is

so spotty that geologists are not sure whether areas near the equator or

nearer the poles were the coolest places on earth. It‘s also possible that

volcanic eruptions had tossed enough dust into the atmosphere to screen

out sunlight and cool the earth.

Such traces are the currency of science—data—and like money, a

richness of data both buys you some credibility and ties you down,

eliminating at least some theoretically plausible explanations. For this

early period, theorists have come up with a variety of ideas to explain the

ancient ice ages, all elegant and mostly immune to both proof and

criticism. For example, a change in the earth‘s orbit could have reduced

the amount of sunlight reaching the planet. However, the only physical

signature of such an event that would show in the rocks would be the

marks of the glaciers themselves.

Reading Comprehension - IPM

Page 46: QUESTION BANK - edumentor.co.inedumentor.co.in/Downloads/2018/Extra Content... · of India's oldest and most important uranium mine, ... Punjab banned e-cigarettes last year and Maharashtra

1. There is an implicit assumption in the statement that geologists don‘t know

whether the coolest places on earth were near the poles or near the

equator. The assumption is that:

A. both polar and equatorial glacial deposits have been found.

B. certain geological information can be considered lost forever.

C. it is more important to determine the date of the ice ages than the

extent of the glaciers.

D. the glaciers were extremely mobile in spite of their mass.

E. areas around the equator are usually hotter than those around the poles

2. Suppose that an advocate of the ―change in orbit‖ theory of the ancient ice

ages criticizes a defender of the ―volcanic eruption‖ theory on the grounds

that only some of the glacial records contain evidence of prior volcanic

activity. The defender might justifiably counter this attack by pointing out

that:

A. a change in the earth‘s orbit would have increased rather than

reduced the sunlight reaching the planet.

B. volcanoes could not possibly release enough dust to block the

atmosphere.

C. a theory that has some supporting evidence is better than a theory

that cannot be proved.

D. a theory should be so constructed as to be immune from proof.

E. the ‗advocate‘ had got his facts from a dubious source

3. Suppose paleobotanists discover that during geological periods of reduced

sunlight, ancient forests died away, leaving fossilized remains. What is the

relevance of this information to the passage?

A. It supports the claim that dust from volcanic eruptions caused the ice

ages.

B. It weakens the claim that dust from volcanic eruptions caused the ice

ages.

C. It supports the claim that ice ages were accompanied by widespread

loss of vegetation.

D. It weakens the claim that the only evidence of a change in orbit would

be glacier marks.

E. It has no relevance to the passage

Reading Comprehension - IPM

Page 47: QUESTION BANK - edumentor.co.inedumentor.co.in/Downloads/2018/Extra Content... · of India's oldest and most important uranium mine, ... Punjab banned e-cigarettes last year and Maharashtra

Passage – 12

There are a great many symbiotic relationships in the marine

environment. A popular one, often noted for the striking beauty of the

juxtaposition, is that of the sea anemone and the clown fish. The

anemone has poison tentacles which—when they contact passing fish—

paralyze the fish and drag the prey in for a meal. The clown fish uses the

anemone‘s tentacle ―garden‖ as a safe haven while attracting prey for the

anemone to capture, for it alone is immune to the sting of the anemone.

Another symbiotic relation that remains the subject of scientific

puzzlement concerns the relationship between Scleractinia, the coral type

whose colonization produces reefs, and their symbiotic partners the

zooxanthellae, the unicellular algae present in the corals‘ endodermic

tissues. It is known that each symbiont plays an integral part in the

formation of a reef‘s protective limestone foundation. The coral polyps

secrete calceous exoskeletons which cement themselves into an

underlayer of rock, while the algae deposit still more calcium carbonate,

which reacts with sea salt to create an even tougher limestone layer.

It is also known that, due to the algal photosynthesis, the reef

environment is highly oxygen-saturated, while the similarly high amounts

of carbon dioxide are carried off rapidly. All this accounts for the amazing

renewability of coral reefs despite the endless erosion caused by wave

activity. However, the precise manner in which one symbiont stimulates

the secretion of calcium carbonate by the other remains unclear.

Scientists have also proposed various theories to explain the

transformation of ―fringing reefs,‖ those connected above sea level to

land masses, into ―barrier reefs‖ that are separated from shorelines by

wide lagoons, and then into free-floating atolls. Though the theory

postulated by Charles Darwin is considered at least partially correct,

some scientists today argue that the creation of the reef forms has more

to do with the rise of sea level that accompanied the end of the Ice Age.

However, recent drillings at Enewetak atoll have uncovered a large

underlay of volcanic rock, which suggests that Darwin‘s explanation may

have been more valid after all.

Even the name given to the reefs is something of a misnomer. The

Scleractinia themselves generally comprise no more than 10 percent of

the biota of the average reef community: zooxanthellae can account for

up to 90 percent of the reef mass, along with foraminifera, annelid

worms, and assorted mollusks. Moreover, reefs can flourish only in

shallow, highly saline waters above 70°F., because the algae require such

circumstances; yet non-reef-building corals occur worldwide under

various environmental conditions, from the Arctic to the Mediterranean,

home of the red coral prized for jewellery. The most likely reason that the

term ―coral reefs‖ persists is that the brilliant variety of coral shapes and

colours makes aesthetic considerations more vivid than biological ones.

Reading Comprehension - IPM

Page 48: QUESTION BANK - edumentor.co.inedumentor.co.in/Downloads/2018/Extra Content... · of India's oldest and most important uranium mine, ... Punjab banned e-cigarettes last year and Maharashtra

1. According to the author, some scientists consider the term ―coral reef‖ a

misnomer because:

A. the beautiful shapes and colours of reefs are produced by the

Scleractinia rather than the zooxanthellae.

B. the coral portion of a reef has little to do with the reef‘s survival.

C. ―non-reef-building‖ corals are found throughout the world.

D. the majority of a reef‘s substance comprises zooxanthellae,

foraminifera, annelid worms, and assorted molluscs while a small

portion comprises the Scleractinia.

E. the reef does not have any coral whatsoever

2. Based on the passage, which of the following is probably an assumption of

scientists studying coral reefs?

A. The theories of reef evolution through glacial melting and through

volcanic subsidence are mutually exclusive.

B. The three main types of coral reefs did not develop independently of

one another.

C. Zooxanthellae are always found in coral reefs.

D. Intense calcification single-handedly protects reefs from destruction

by waves and other natural causes.

E. Coral reefs are always blue in colour

3. The passage mentions the recent drillings at the Enewetak atoll. This

reference serves to:

A. stengthen the claims made by scientists today concerning reef

transformation.

B. weaken the claims made by scientists today concerning reef

transformation.

C. strengthen the claims made by Darwin concerning reef transformation.

D. weaken the claims made by Darwin concerning reef transformation.

E. has no impact on the claims made by Darwin concerning reef

transformation.

Reading Comprehension - IPM

Page 49: QUESTION BANK - edumentor.co.inedumentor.co.in/Downloads/2018/Extra Content... · of India's oldest and most important uranium mine, ... Punjab banned e-cigarettes last year and Maharashtra

Passage – 13

The latest prominent principle of criminal sentencing is that of ―selective

incapacitation.‖ Selective incapacitation, like general incapacitation,

involves sentencing with the goal of protecting the community from the

crimes that an offender would commit if he were on the street. It differs

from general incapacitation in its attempt to replace bluntness with

selectivity.

Under a strategy of selective incapacitation, probation and short

terms of incarceration are given to convicted offenders who are identified

as being less likely to commit frequent and serious crimes, and longer

terms of incarceration are given to those identified as more crime prone.

Selective incapacitation has the potential for bringing about a reduction in

crime without an increase in prison populations. This reduction could be

substantial.

Reserving prison and jail space for the most criminally active

offenders in some instances conflicts not only with other norms of legal

justice, but with norms of social justice as well. If we reserve the sanction

of incarceration only for the dangerous repeat offender, excluding the

white collar offender and certain other criminals who pose no serious

threat of physical injury to others, we may end up permitting harmful

people from the middle class to evade a sanction that less privileged

offenders cannot.

One of the most pervasive criticisms of selective incapacitation is that

it is based on the statistical prediction of dangerousness; because such

predictions are often erroneous, according to this point of view, they

should not be used by the court. This criticism is related to both the

nature of the errors and to the use of certain information for predicting a

defendant‘s dangerousness. Let‘s first consider the nature of errors in

prediction.

Prediction usually results in some successes and in two kinds of

errors: ―false positives‖ and ―false negatives.‖ The problem of false

positives in sentencing is costly primarily to incarcerated defendants who

are not really so dangerous, while false negative predictions impose costs

primarily on the victims of subsequent crimes committed by released

defendants. In predicting whether a defendant will recidivate, the

problem of false positives is widely regarded as especially serious, for

many of the same reasons that it has been regarded in our society as

better to release nine offenders than to convict one innocent person.

A tempting alternative is to reject prediction altogether; obviously, if

we do not predict, then no errors of prediction are possible. A flaw in this

logic is that, whether we like it or not—indeed, even if we tried to forbid

it—criminal justice decisions are now, and surely always will be, based on

predictions, and imperfect ones, at that. Attempts to discourage

prediction in sentencing may in fact produce the worst of both worlds:

the deceit of predictive sentencing disguised as something more tasteful,

and inferior prediction as well. If we are to reserve at least some prison

and jail space for the most criminally active offenders, then the prediction

of criminal activity is an inescapable task. Is selective incapacitation truly

Reading Comprehension - IPM

Page 50: QUESTION BANK - edumentor.co.inedumentor.co.in/Downloads/2018/Extra Content... · of India's oldest and most important uranium mine, ... Punjab banned e-cigarettes last year and Maharashtra

an effective and appropriate proposal, an ―idea whose time has come,‖ or

is it a proposal that carries with it a potential for injustice?

1. Suppose the number of dangerous criminals that would be imprisoned under

selective incapacitation but otherwise set free is greater than the number of

harmless criminals who would be set free under selective incapacitation but

otherwise imprisoned. How would this information be relevant to the

passage?

A. It weakens the claim that the goal of selective incapacitation is to

protect the community.

B. It strengthens the claim that there are more violent than non-violent

criminals.

C. It weakens the claim that selective incapacitation would not increase

prison populations.

D. It strengthens the claim that white-collar criminals unfairly receive

shorter sentences.

E. It is of no relevance to the passage

2. The author‘s statement that selective incapacitation may ―end up permitting

harmful people from the middle class to evade a sanction that less

privileged offenders cannot‖ assumes that:

A. there are more offenders in the lower-class than in the middle-class.

B. the dangerous repeat offenders are lower-class and not middle-class.

C. harmful middle-class people can use their money to avoid prison.

D. lower-class offenders do not deserve to suffer incarceration.

E. the rich do not ever commit crimes

3. Based on the passage, which of the following would most likely be cited by an

opponent of statistical prediction as the reason that prediction should be

abandoned?

A. The possibility of letting a dangerous criminal loose is too great.

B. The possibility of imprisoning a man who should be allowed to go free

is too great.

C. The court makes more accurate decisions when statistics is employed.

D. Dangerousness has yet to be adequately defined as a legal concept.

E. Statistics is an inexact science

Reading Comprehension - IPM

Page 51: QUESTION BANK - edumentor.co.inedumentor.co.in/Downloads/2018/Extra Content... · of India's oldest and most important uranium mine, ... Punjab banned e-cigarettes last year and Maharashtra

Passage – 14

A cause of fatal mining accidents was once the peculiar configuration of

the controls on the trams shuttling along mineshafts. Each tram had a

steering wheel that rose straight up from the floor, with a brake pedal on

one side and an accelerator pedal on the other. There was no room to

turn the tram around, so to reverse direction the driver simply took a

seat on the other side of the steering wheel, whereupon what had been

the brake became the accelerator, and vice versa. While this may sound

ingenious, it proved disastrous.

Many people set an electric burner on high thinking that it will heat up

faster that way: they have the mental model of a gas stove, whose knobs

actually do increase the heat‘s intensity. On an electric stove, however,

the knob is merely a switch that turns on the burner and then turns it off

when a certain temperature is reached.

Consider the humble wristwatch, which has been transformed into a

kind of wrist-mounted personal computer, with a digital display and a

calculator pad whose buttons are too small to be pressed by a human

fingertip. By replacing the watch‘s conventional stem-winding mechanism

with a mystifying arrangement of tiny buttons, the manufacturers created

a watch that was hard to reset.

One leading manufacturer was distressed to discover that a line of its

particularly advanced digitals was being returned as defective by the

thousands, even though the watches actually worked perfectly well.

Further investigation revealed that they were coming back soon after

purchase and thereafter in two large batches—in the spring and the fall,

when the time changed.

Charles Mauro, a consultant in New York City, is a prominent member

of a branch of engineering generally known as ergonomics, or human-

factors—the only field specifically addressing the question of product

usability. Mauro was brought in to provide some help to the watch

manufacturer, which was experiencing what Mauro calls the ―complexity

problem.‖ With complexity defined as a fundamental mismatch between

the demands of a technology and the capabilities of its user, the term

nicely captures the essence of our current technological predicament.

A growing number of technologists speak of user-centred design as a

means of scrupulously maintaining the user‘s perspective from start to

finish, adding technology only where necessary. When confronted by

some mystifying piece of high-tech gadgetry, consumers naturally feel

that there is something wrong with them if they can‘t figure it out. In

truth it is usually not their fault. Mauro attributes the confusion to the

fact that most products are ―technology-driven,‖ their nature determined

not by consumers and their needs and desires but by engineers who are

too often entranced with the myriad capabilities of the microprocessors

that lie at the devices‘ hearts

Reading Comprehension - IPM

Page 52: QUESTION BANK - edumentor.co.inedumentor.co.in/Downloads/2018/Extra Content... · of India's oldest and most important uranium mine, ... Punjab banned e-cigarettes last year and Maharashtra

1. Based on the passage, an ergonomics expert would be likely to place high

value on a product that:

A. required no instruction at all to use.

B. did not incorporate modern technology.

C. could be easily manipulated by hand.

D. solved complex problems for its user.

E. required elaborate instructions for proper usage

2. When consumers feel that there is something wrong with them if they can‘t

figure a high-tech gadget out, which of the following assumptions are they

making?

A. The gadget was designed for ready use by the average consumer.

B. Technology can only be understood by engineer-types.

C. The gadget designers were blind to the consumers‘ needs.

D. Everyone is equally capable of understanding new technology.

E. they are not as intelligent as the other person

3. According to one consumer survey, a third of all VCR owners have given up

trying to program their machines for time-delayed viewing. How would the

author probably explain this fact?

A. VCR owners have not yet found the correct mental model by which to

interpret the VCR.

B. Those owners have concluded that the VCR was not well designed.

C. Those trying to program the machine are not as technologically savvy

as they should be.

D. The VCR is the result of technology-driven rather than user-centred

design.

E. The author would view this as an aberration

Reading Comprehension - IPM

Page 53: QUESTION BANK - edumentor.co.inedumentor.co.in/Downloads/2018/Extra Content... · of India's oldest and most important uranium mine, ... Punjab banned e-cigarettes last year and Maharashtra

Passage – 15

Physicians have disagreed for years about whether they should be

involved in capital punishment of convicted criminals. Some physicians

vigorously support participation, often arguing that organs should first be

removed for transplantation. One frequent objection to capital

punishment is that sometimes techniques don‘t work the first time,

resulting in lingering, painful deaths. If physicians would guarantee that a

patient would not die in such a way, they would gain the trust of some

patients.

For any kind of killing, some physicians favour the creation of

―designated killer‖ technicians. This would free physicians from the taint

of killing, keeping their image pure and their hands clean. But is this

workable? Insofar as the designated killers are mere technicians, what

prevents them from abusing their role? Wouldn‘t it be better for

physicians, torn between saving life and honouring patients‘ wishes, to be

reluctant killers? Wouldn‘t physicians know best what to do if something

went wrong?

Many physicians paradoxically endorse mercy killing but refuse to do

it themselves. Nor do they think other physicians should kill. Physicians

who support mercy killing but who don‘t want physicians to kill commonly

emphasize the importance of maintaining the role of the physician as a

healer and preserver of life. One poll of American physicians showed 60

percent favouring euthanasia but less than half would perform it

themselves. To such physicians, taking life radically conflicts with the

symbolic image of physicians. Such conflict, they say, destroys trust in

physicians.

Discussing this problem of designated killers in 1988, New England

Journal of Medicine editor Marcia Angell called the idea ―an unsavoury

prospect.‖ She suggested that mercy killing may one day be the end

point of a continuum of good patient care. She asks how any physician

can excuse himself from this most basic notion? Dr. Angell concluded,

―Perhaps, also, those who favour legalizing euthanasia but would not

perform it should rethink their position.‖

Dr. Angell implies that it is hypocritical to favour mercy killing but would

be unwilling to perform it. Is this true? There are at least two schools of

thought. Some thinkers believe that if one favours, say, meat-eating, one

should be willing to kill and prepare animals for eating oneself. Others

conclude differently, seeing no reason why each person who favours a

position must be willing to implement it.

Must you be willing to kill a serial murderer to favour capital punishment?

Critics say one must. Being face-to-face with one‘s victims creates basic

moral qualms and such moral restraints are important to respect. In

Stanley Milgram‘s studies on obedience, naive subjects under an

experimenter‘s control were dramatically less willing to inflict injury as

the victims became closer to subjects under study. In contrast, as the

consequences of actions became more remote, such as by pressing a

switch which released a bomb on an unseen, unknown populace, it

became easier to inflict injury.

Reading Comprehension - IPM

Page 54: QUESTION BANK - edumentor.co.inedumentor.co.in/Downloads/2018/Extra Content... · of India's oldest and most important uranium mine, ... Punjab banned e-cigarettes last year and Maharashtra

1. Consider the main points that the author makes throughout the passage. The

primary purpose of this passage is to:

A. speculate on the symbolism of the physician as healer.

B. portray those doctors who argue against administering euthanasia as

hypocritical.

C. cast and explain the different arguments surrounding euthanasia.

D. introduce the concept of ―designated killers‖ to a receptive audience.

E. convince doctors to take up euthanasia

2. According to the passage, which of the following is most likely to be true of

those physicians who favour the creation of so-called ―designated killers?‖

A. They believe it is good patient care to provide a continuum of services.

B. They seek to keep the physician remote from acts of harm.

C. They understand that it raises a conflict with their opinions on capital

punishment.

D. They fear abuse of the privilege that comes from this unique role.

E. They are emotionally weak

3. According not necessarily to the author, but to those in favour of euthanasia

specifically, what is a potentially negative aspect of the use of ―designated

killers?‖

A. They would disrupt the continuum of patient care provided by a

physician.

B. They might release physicians from an association with death.

C. Their use might prevent lingering, painful deaths.

D. The prescription of euthanasia may become more prevalent as

physicians are removed from the act itself.

E. They might not be as qualified as the actual doctors

4. The reader can conclude that a basic assumption of those in favour of using

―designated killers‖ is that:

A. the practice would evolve into a readily available medical option.

B. very few physicians could be convinced to assume the role and duties.

C. physicians would have to be present with the patient in order to

conduct euthanasia.

D. many physicians are reluctant to administer euthanasia because they

are not in favour of capital punishment.

E. they are eventually complying with the patients‘ wishes

Reading Comprehension - IPM

Page 55: QUESTION BANK - edumentor.co.inedumentor.co.in/Downloads/2018/Extra Content... · of India's oldest and most important uranium mine, ... Punjab banned e-cigarettes last year and Maharashtra

Passage – 16

In August 1348 the bubonic plague, or Black Death, suddenly appeared

in England. Its germs were carried by the fleas on black rats that came

into the country on ships from abroad. The first outbreak of the plague

was of intense ferocity, for the people had no immunity and persons

living close to the margin of subsistence fell victims to the disease.

Returning in 1361, the plague caused high mortality among children

born since 1348; there were other visitations in 1368 and 1375. High

farming in the thirteenth century had been based on the scarcity of

land, a large population, and a great demand for food—conditions that

had forced the peasants to remain on their holdings and to accept the

burdens of serfdom. But when the demand for food was less, the profits

of agriculture shrank. High farming, which had already been slipping

before 1348, came to an end.

The startling fact about those figures is the amazing drop in

population between 1348 and 1377. It may be the number of people in

overcrowded England already was beginning to decline before the

coming of the Black Death. There were floods and famines in the years

between 1315 and 1317. Certainly the plague caused a high mortality.

In some monasteries the monks all but disappeared (it is thought that

half the clergy in England fell victims to the pestilence). The Black Death

had its most striking effect on the rural economy. The balance between

the number of labourers and the amount of land under cultivation and

the relations between lord and peasant were quickly altered. There were

deserted villages and many unoccupied peasant holdings. After the first

visitation widows and widowers remarried quickly and produced as

many children as before; but because of the high mortality among

young people this population increase was not maintained later in the

century.

The work of the manor could not be performed by the villeins who

had survived the plague; the lord had to employ casual labor at wages

that doubled within a decade. Moreover, a villein, once tied to his

holding by economic necessity, could easily run away to another manor

where employment would be offered to him with no questions asked.

Landowners complained bitterly of the labour shortage and of the

wages they had to pay. In 1351 they obtained the Statute of Laborers,

which fixed wages at the rates before the plague, declared that all

landless men must accept work when it was offered to them, and

prohibited peasants from moving from one manor to another. For a time

the statute had some effect, but in the long run it was useless, for

wages continued to rise and employers had to pay them. There was also

a scarcity of tenants. Few manors were without vacant holdings; hence

the yield was less and income from the land declined. Agricultural

products no longer fetched high prices. Yet the cost of luxuries and of

manufactured goods was rising.

Thereafter the plague subsided in the rural areas but remained endemic

in London and other towns, where it could become active at any time

Reading Comprehension - IPM

Page 56: QUESTION BANK - edumentor.co.inedumentor.co.in/Downloads/2018/Extra Content... · of India's oldest and most important uranium mine, ... Punjab banned e-cigarettes last year and Maharashtra

and could spread along lines of communication into the country. It

remained in England for more than 300 years.

1. Which of the following was NOT a contributing factor in the dependence of

the peasantry on high farming as a means of subsistence?

A. A large population

B. A widespread outbreak of plague

C. A great demand for food

D. A scarcity of land

E. Too many mouths to feed

2. According to information brought forth by the author in the passage, the

economic difficulties brought on by the Black Death were not quickly resolved

because:

A. potential workers were afraid to leave their homes due to the fear of

contracting disease.

B. population gains that might have been made by remarriages were

offset by a high infant mortality rate.

C. many landholdings were left unoccupied, often without recourse.

D. the Statute of Laborers fixed wages at the pre-plague levels.

E. there was no money in the economy

3. Which of the following claims would, if true, most substantially weaken the

author‘s claim that the plague brought an end to the practice of high

farming?

A. The practice of high farming was reinforced after the floods and

famines in the 1310s reduced the amount of arable land.

B. Immediately following the plague, the profits of agriculture would see

a rebound due to the stabilization in wages and food prices.

C. The numbers of peasants working on English farms decreased

throughout much of the years of plague.

D. The Statute of Laborers began to be strictly enforced when it became

apparent that wages were still rising.

E. Over the next few years following the plague, the incomes of

agriculturists kept falling lower and lower

Reading Comprehension - IPM

Page 57: QUESTION BANK - edumentor.co.inedumentor.co.in/Downloads/2018/Extra Content... · of India's oldest and most important uranium mine, ... Punjab banned e-cigarettes last year and Maharashtra

Passage – 17

In 1991, the issue of feminism resurfaced in the mainstream media on a

broad scale, from the release of the film Thelma & Louise to the

publication of such books as Naomi Wolf‘s The Beauty Myth, Susan

Faludi‘s Backlash, and Gloria Steinem‘s The Revolution Within: A Book of

Self-Esteem, to the attention given to the issue of sexual harassment in

the wake of the Anita Hill/Clarence Thomas hearings. Shortly after the

Hill/Thomas hearings brought the issue of sexual harassment into the

public eye, the music industry was rocked by the announcement of

allegations of sexual harassment involving executives at three major

record companies and an attorney at a leading L.A. law firm.

Even the Rock & Roll Hall of Fame, established in the mid-‘80s to

recognize the contributions of those involved in the music business, has

been criticized for overlooking women‘s contributions to the industry.

Mary Wilson noted this discrepancy in Supreme Faith when she wrote

about the Supremes‘ induction into the Hall of Fame in 1988, and her

participation in the all-star jam that traditionally occurs after the

ceremonies. Out of the nearly one-hundred performers, songwriters, label

executives, and promoters now in the Hall of Fame, the only female

inductees to date are Aretha Franklin and Lavern Baker, inducted as

performers, Carole King (with Gerry Goffin), inducted as a non-

performer, and Bessie Smith and Ma Rainey, inducted as ―Forefathers.‖

Though the battle over abortion rights caused women to recognize the

underlying fragility of the gains the feminist movement had made, the

media focus on other ―women‘s issues‖ further illuminated the struggles

women continued to face in society. A November 3, 1991, story in the

Los Angeles Times not only discussed the specific allegations mentioned

above regarding sexual harassment, it also examined sexual harassment

in the record industry as a whole, and revealed the ―put up or shut up‖

bind women who experience harassment are placed in.

As a result, instead of going through the legal system, women

working in the industry have been driven to create an informal grapevine

to pass on information about companies deemed ―safe havens‖ from

sexual harassment and to warn each other about the ―bimbo hounds‖ in

different record company departments.

Since then, the increasing threat to women‘s reproductive freedom in the

U.S. has mobilized growing number of women to reawaken from a state

of ―post-feminist‖ complacency. Attitudes toward a female presence in

the workplace showed little signs of change in other areas of the music

industry. An article in Billboard in March 2, 1991, noted that though

almost half the sales positions in radio were held by women, there were

far fewer women working in programming or on-air positions. In the

same article, Lisa Lyons, a program director at Dayton, Ohio station

WAZU, related a story about the necessity of ―dressing down‖ (a tactic

similar to the one Gail Colson had adopted when she was managing

director at Charisma Records in the ‗70s) that also sounds depressingly

familiar; ―I always make it a point to look like a slob. It‘s a little

humiliating and degrading when an artist shakes your MD‘s [music

director‘s] hand and asks you to sleep with him.‖

Reading Comprehension - IPM

Page 58: QUESTION BANK - edumentor.co.inedumentor.co.in/Downloads/2018/Extra Content... · of India's oldest and most important uranium mine, ... Punjab banned e-cigarettes last year and Maharashtra

1. Based on the examples provided in the passage, with which of the

following assertions is it most likely that the author would disagree?

A. Working women face few pressures to maintain a physically

attractive appearance.

B. Women have often tied the success of the feminist movement to the

fight over abortion.

C. Women working in the music industry are subject to similar obstacles

as working women in other fields.

D. Women working in the music industry have not achieved status

commensurate with their contribution to the art.

E. Women have always been discriminated against by men

2. Each of the following is cited by the author in the passage as evidence of

the reawakening of feminism EXCEPT:

A. the release of the film Thelma & Louise.

B. women‘s response to the increasing threat of abortion rights.

C. the induction of the first women into the Hall of Fame.

D. the publication of Naomi Wolf‘s The Beauty Myth.

E. the increased attention given to the issue of sexual harassment

3. Suppose the number of female executives within the music industry has

not increased measurably within the past two decades. If this statement is

true, what effect would it have on the author‘s argument?

A. It would support the argument that women already hold too many

positions at the executive level.

B. It would contradict the argument that men focus on women‘s

physical appearance.

C. It would support the argument that the number of females inducted

into the Hall of Fame will increase rapidly in the coming years.

D. It would support the argument that women are not moving ahead in

sectors such as the music industry.

E. It would neither support nor contradict the author‘s argument

Reading Comprehension - IPM

Page 59: QUESTION BANK - edumentor.co.inedumentor.co.in/Downloads/2018/Extra Content... · of India's oldest and most important uranium mine, ... Punjab banned e-cigarettes last year and Maharashtra

Passage – 18

In the 1930s the Payne Foundation funded studies attributing juvenile

crime to movie violence, complete with testimonials of youthful

offenders that they had gotten larcenous ideas from the silver screen.

Legions of censors from the Hays Office monitored Hollywood output to

make sure that, at the least, crime didn‘t pay. In the 1950s, Dr.

Frederic Wertham made a name for himself by attributing all manner of

delinquencies to the mayhem depicted in comic books. If today‘s

censorious forces smell smoke, it is not in the absence of fire.

In recent years, market forces have driven screen violence to an

amazing pitch. As the movies lost much of their audience—especially

adults—to television, the studios learned that the way to make their

killing, so to speak, was to offer on big screens what the networks

would not permit on the small. Thus, decades ago the ―action movie‖—a

euphemism for, among other things, grisly violence—aimed to attract

the teenagers who were the demographic category most eager to flee

the family room.

Aiming to recoup losses and better compete with cable, television

programmers struck back; the networks lowered their censorship

standards and pruned their ―standards and practices‖ staffs; the

deregulatory Federal Communications Commission clammed up; and the

local news fell all over itself cramming snippets of gore between

commercials.

There are indeed reasons to attribute violence to the media, but the

links are weaker than recent headlines would have one believe. The

attempt to demonize the media distracts attention from the real causes

of—and the serious remedies for—the epidemic of violence. The

question the liberal crusaders fail to address is not whether these

images are wholesome but just how much real-world violence can be

blamed on the media. Assume, for the sake of argument, that every

copycat crime reported in the media can plausibly be traced to

television and movies. Let us make an exceedingly high estimate that

the resulting carnage results in 100 deaths per year that would

otherwise not have taken place. These would amount to 0.28 percent of

the total of 36,000 murders accidents, and suicides committed by

gunshot in the United States in 1992.

That media violence contributes to a climate in which violence is

legitimate—and there can be no doubt of this—does not make it an

urgent social problem. Violence on the screens, however loathsome,

does not make a significant contribution to violence on the streets.

Images don‘t spill blood. Rage, equipped with guns, does. Desperation

does. Revenge does. As liberals say, the drug trade does; poverty does;

unemployment does. It seems likely that a given percent increase in

decently paying jobs will save thousands of times more lives than the

same percent decrease in media bang-bang. And once in a while—

meaning far too often—some grotesque images inspire emulation.

Reading Comprehension - IPM

Page 60: QUESTION BANK - edumentor.co.inedumentor.co.in/Downloads/2018/Extra Content... · of India's oldest and most important uranium mine, ... Punjab banned e-cigarettes last year and Maharashtra

1. The passage suggests that having more stringent controls on media violence

would NOT have a great effect on the death rate because:

A. the numbers of deaths resulting from so-called ―copycat‖ acts of

violence composes only a small portion of violent deaths each year.

B. the number of deaths resulting from so-called ―copycat‖ acts of

violence would remain unchanged nonetheless.

C. networks and film studios lack the personnel to enforce any new

regulation.

D. there exists no definite link between media violence and actual

violence.

E. very few people watch television these days

2. If delivered in a paper that sought to undermine the points of this passage,

which of the following statements, if true, would most seriously weaken the

passage‘s central argument?

A. The number of violent acts depicted in the media has remained more

or less constant for the past decade.

B. A Canadian study reported a sixteen-percent increase in violent crimes

after exposure to television and film episodes in which violent acts

were depicted.

C. Politicians and celebrities are assisting effectively in diminishing

violence.

D. Films belonging to the ―action‖ genre have found little acceptance at

the box office

E. Children become more violent after playing violent video games

3. The broadcast networks have recently proposed a system of rating program

content, similar to those ratings in the film industry. Which of the following

best characterizes the relevance of this statement?

A. The statement acknowledges that the networks have taken little

responsibility in patrolling the content of their programming.

B. The statement implies that those who speak out against media

violence have had significant success in convincing the networks to

enforce stricter content standards.

C. The statement suggests that some convincing evidence supporting a

stronger link between media violence and violent acts has been found.

D. The statement suggests that networks will decrease the amount of

shows that contain violent content.

E. The statement has no relevance to the argument in the passage

Reading Comprehension - IPM

Page 61: QUESTION BANK - edumentor.co.inedumentor.co.in/Downloads/2018/Extra Content... · of India's oldest and most important uranium mine, ... Punjab banned e-cigarettes last year and Maharashtra

Passage – 19

Since 1789, the Constitution has granted the President the authority to

veto legislation passed by Congress. The threat of a veto in many cases

precipitates compromise on the content of a bill that would be otherwise

mired in debate before it reached the President. The ―regular‖ veto is a

qualified negative veto, which necessitates a two-thirds vote by Congress

to be overridden. The ―pocket‖ veto, on the other hand, is exercised

when a bill sits on the President‘s desk without being signed before

Congress has adjourned (and is therefore unable to override the veto).

Opponents of the pocket veto allege that its absolute nature grants the

President excessive power. They liken it to a prerogative of the English

Kings that the Framers vehemently despised. The argument also

embraces a vast body of commentary on the ―Imperial Presidency,‖ that

is, the growing accumulation of power in the executive relative to the

legislative branch.

These arguments, in claiming an imbalance of federal powers,

misrepresent the pocket veto. Unlike the royal prerogative, the pocket

veto is exercised by a democratically-elected leader pursuant to a clearly

defined constitutional procedure in which presentation of a bill by

Congress may be arranged so as to thwart the possible execution of the

pocket veto. Moreover, an absolute veto forecloses further action on a

proposal whereas Congress may overcome a pocket veto by instituting a

reintroduction and passage of the rejected bill in a subsequent term.

The ―Imperial Presidency‖ developed from the encroachment of

executive action into areas where it has been assumed that the legislative

branch retains supremacy. The legislative process, however, clearly

orders shared responsibility between the President and Congress. One

should not mistake Presidential powers granted to block legislation for

those that would, in effect, supplant congressional authorization. The

latter threatens to override the constitutional system of checks and

balances; the former situation, typified by the pocket veto, is a part of

that system of checks and balances.

The arguments raised in Kennedy and Barnes implicitly claim that a

regular veto would be overridden, or not exercised at all. Consequently,

the pocket veto grants the President a special political tool against

―popular will‖ as exercised by Congress. Herein lies the fundamental

disagreement over the pocket veto. Opponents press for the President to

defer to a seemingly inevitable congressional victory while proponents of

this second type of veto stand behind its historical use by the President to

stall or delay legislation he thinks unwise. If circumspection and

deliberation are the more valued aspects of the law-making process, even

the most blatantly political use of the pocket veto passes muster.

Historical practice favours the President‘s role as an interloper.

Reading Comprehension - IPM

Page 62: QUESTION BANK - edumentor.co.inedumentor.co.in/Downloads/2018/Extra Content... · of India's oldest and most important uranium mine, ... Punjab banned e-cigarettes last year and Maharashtra

1. As used in line 42, the word ―interloper‖ most nearly means:

A. one who unjustly assumes power through the use of force.

B. one who acts as a liaison between different parties.

C. one who prevents certain actions from occurring.

D. one who thinks carefully before acting.

E. one who lopes intermittently

2. The author refers to Kennedy and Barnes in the passage in order to:

A. prove that Congress opposes the pocket veto as a limit to its

legislative power.

B. suggest that the validity of the pocket veto has been a matter of

judicial concern.

C. show how the pocket veto‘s weaknesses override its strengths.

D. praise how the pocket veto can delay the legislative process.

E. criticise the pocket veto

3. The author suggests that opponents of the pocket veto would most likely

agree that:

A. the President should not be allowed to exercise legislative authority.

B. use of the pocket veto unfairly removes power from the legislative

branch.

C. Congress should have the right to override the pocket veto.

D. the absolute veto should be reinstated by Congress.

E. pocket veto is unconstitutional in character

Reading Comprehension - IPM

Page 63: QUESTION BANK - edumentor.co.inedumentor.co.in/Downloads/2018/Extra Content... · of India's oldest and most important uranium mine, ... Punjab banned e-cigarettes last year and Maharashtra

Passage – 20

Over the past two decades, courts have gone far in their interpretations

of civil rights legislation to ensure African-American participation

throughout the work force. Much ground has been gained in this fight

against an institutionalized inequality that has become ingrained in our

collective psyche. There are some, though, who remain concerned that

the situation of African-American managers has made only limited

progress in certain industries.

It is particularly disturbing to find relatively few African-American

executives in an industry whose work force consists primarily of African-

Americans, namely professional team sports. In the 2001 Racial and

Gender Report Card published by the Centre for Sport in Society of

Northeastern University, which analysed the composition of players and

administrators in professional leagues, only the Women‘s National

Basketball Association and the National Basketball Association scored

well. According to the report, generally ―who‘s running the league doesn‘t

look like who‘s playing in the league.‖ Questions are being posed as to

whether general employment principles are properly applicable to the

sports business, or whether the sports industry in fact enjoys a special

status similar to baseball‘s antitrust exemption. With a growing number

of African-Americans achieving the educational standards as well as the

practical experience required for executive positions, it is foreseeable that

the professional team-sport industry will soon face challenges to its

executive employment decisions.

The organizational structures of clubs and leagues are similar to other

large businesses. It is within these two organizations that the absence of

African-American executives is most noticeable. Some professional club

owners justify the absence of African-American executives at these levels

by alluding to clubs‘ and leagues‘ organizational structure as being

familial in nature. Whether this characterization justifies the exclusion of

African-American executives is questionable at best.

The sports industry is within the purview of Title VII of the Civil Rights

Act of 1964, which exists to prohibit intentional discrimination in

employment on the basis of race, colour, religion, sex or national origin.

However, recent legal developments call into question whether the

principles applied to assure lower level jobs will be applied to protect

African-American executives from discrimination while seeking upper

level positions. The courts have traditionally been willing to assess an

applicant‘s qualifications in resolving claims of discrimination. Yet courts

in upper level cases often profess a lack of expertise and refuse to assess

an applicant‘s qualifications.

The Supreme Court has not yet confronted the issue of which Title VII

standards and rules should apply to discrimination cases involving

African-American executives. Lower federal courts have seemingly begun

to distort these standards in cases involving executive and professional

employees. Given that executive positions have unique characteristics,

resolution of these actions becomes all the more difficult. Rules

developed to deal with lower level Title VII cases may not always be

applicable to executive employment cases.

Reading Comprehension - IPM

Page 64: QUESTION BANK - edumentor.co.inedumentor.co.in/Downloads/2018/Extra Content... · of India's oldest and most important uranium mine, ... Punjab banned e-cigarettes last year and Maharashtra

1. Which of the following would be analogous to the situation described in

paragraph 2 regarding the absence of African-American executives in a field

with many African-American athletes?

A. The Board of a non-profit Protestant Church youth organization

consists solely of middle-aged men and women.

B. A social service agency, which serves minority and disadvantaged

youths, is run by a management team of suburban professionals.

C. Fewer than half of the managers of a national feminist bookstore

company, which employs mostly female cashiers and clerks, are

women.

D. A state anti-smoking campaign is organized and implemented by a

group of smokers and non-smokers.

E. A national basketball team only comprises player who weigh more than

100 kgs

2. The author of the passage mentions the Supreme Court in the final

paragraph in order to:

A. prove that courts have been slow to respond to Title VII cases

initiated by African-American executives who have been denied

upper-level positions in professional team sports.

B. suggest that upper courts need to provide standards for the proper

application of Title VII posits in upper-level employment

discrimination cases.

C. explain that the lower courts have been more willing to handle

executive employment decision cases under the auspices of Title VII.

D. argue that the Judicial Branch of the federal government should set

precedents for treating executive employment cases that are

protected by Title VII.

E. praise the actions taken by the Supreme court with regards to the issue

of discrimination in sports

3. The author suggests which of the following about the relatively low number

of African-American executives currently employed in the sports industry?

A. It is attributable to flaws in the American educational system.

B. It is caused by baseball‘s antitrust exemption.

C. It is the result of African-American executives‘ lack of managerial

experience.

D. It does not compare favourably with the industry‘s labour force as a

whole.

E. The number has been hugely exaggerated

Reading Comprehension - IPM

Page 65: QUESTION BANK - edumentor.co.inedumentor.co.in/Downloads/2018/Extra Content... · of India's oldest and most important uranium mine, ... Punjab banned e-cigarettes last year and Maharashtra

Passage – 21

What would be required for successful direct protection of human rights?

The authority to command violating parties to do otherwise? The ability

to enforce such a command? Overwhelming political pressure directed

against human rights violations to the exclusion of other interests? No

United Nations human rights body has such authority and power.

The United Nations‘ primary raison d‘etre in the human rights field as

acted upon by the Human Rights Committee is long-term. It may be that

the sum total of UN activity in this field is supposed to socialize or

educate actors into changing their views and policies on human rights

over time toward a cosmopolitan human rights standard as defined by

United Nations instruments. Conversely one can say that the entirety of

UN human rights activity is to dispense or withhold a stamp of legitimacy

on member states according to their human rights record. It can be

persuasively argued that in some cases a ruling regime lost ground in its

struggle for legitimacy in the eyes of important actors because of

violations of aforementioned rights. The United Nations‘ definition of

human rights probably contributed to the process.

At some point, socialization and manipulation of legitimacy must

directly change specific behaviour and must lead to direct protection by

some actor. In a few situations this linkage can already be demonstrated.

In the case of Filartiga v. Peña Irala in the United States, a federal court

held torture to be prohibited by customary international law, using United

Nations instruments and actions as part of its reasoning. ―Once a tort can

be considered to be in violation of the law of nations, Sec. 1350 allows

immediate access to a federal court.… It is now generally accepted by the

United States and the vast majority of other member nations of the

United Nations that gross violations of human rights are, as a matter of

international law, a legitimate concern of the world community.‖ This

case opened the possibility of express prosecution of torturers of any

nationality who appear in the jurisdiction of the United States. Other

courts in the U.S. have also used United Nations instruments and

activities as part of their decisions, and other states beyond the U.S.

show some influence from UN instruments in their legal and

administrative decisions. The 1998 Pinochet extradition case in London,

described by Human Rights Watch as a ―wake-up call‖ to tyrants

everywhere, was decided on the basis that both Britain and Chile had

ratified the United Nations Convention against Torture.

Reading Comprehension - IPM

Page 66: QUESTION BANK - edumentor.co.inedumentor.co.in/Downloads/2018/Extra Content... · of India's oldest and most important uranium mine, ... Punjab banned e-cigarettes last year and Maharashtra

1. According to various points made by the author of the passage, all of the

following are ways in which the UN can exert influence over human rights

EXCEPT:

A. by persuading member states to change certain laws to avoid

human rights violations.

B. by enforcing a UN command to cease any behaviour that does not

adhere to UN standards.

C. by recognizing certain countries based on their human rights record.

D. by affecting the legal and political policies of member states.

E. by providing a stamp of legitimacy to some member countries

2. The passage suggests that the author would most likely agree with which

of the following statements?

A. The UN has done little to affect the protection and establishment of

human rights.

B. Human rights violations should be the primary concern of the UN.

C. International policies can be influenced by UN activities and

proclamations.

D. Future human rights court cases may turn to UN policies for

assistance.

E. The UN needs to be given military powers

3. The author mentions the case of Filartiga v. Peña Irala primarily in order

to:

A. describe United Nations human rights activity that led to direct

protection by an actor.

B. demonstrate the dangers of the UN‘s concentration on long-term

effects.

C. provide evidence that torture is prohibited by international law.

D. cite a case in which the UN withheld legitimacy from a target state.

E. cite a case wherein UN intervention proved futile

4. Regardless of what the rest of the passage might be arguing, the author‘s

principal concern in the first paragraph is most likely to:

A. propose changes that would increase UN effectiveness in enforcing

human rights.

B. indicate indirectly the shortfalls of UN human rights activity

concerned with short-term change.

C. explain the UN‘s function in the field of human rights by giving

examples.

D. describe the major activity of the UN in the field of human rights

E. to praise the policies of the United Nations

Reading Comprehension - IPM

Page 67: QUESTION BANK - edumentor.co.inedumentor.co.in/Downloads/2018/Extra Content... · of India's oldest and most important uranium mine, ... Punjab banned e-cigarettes last year and Maharashtra

Passage – 22

The combination of consonant-vowel syllabic glyphs and logographs in

ancient Mayan gave the scribes a variety of choices with which to write

the words of their texts in detail. For example, one very common

honorific title in Maya texts is ahaw, meaning ―lord‖ or ―noble.‖ Ahaw

may be written in logographic form as a head in profile, with the

distinctive headband or scarf that marked the highest nobility in Maya

society. But it is also possible to write the word as a combination of three

phonetic, syllabic signs: a-ha-wa. Likewise, the word pakal (―shield‖) can

be indicated by a depiction of a shield or by the combination of syllabic

elements pa-ka-la.

Mayan signs are by nature highly pictorial, often representing in

considerable detail animals, people, body parts and objects of daily life.

The pictorial principle is taken to the extreme in inscriptions composed of

―full-figure‖ glyphs, in which individual signs and numbers become

animated and are shown interacting with one another. None of this

should be taken to mean that the Maya only wrote in simple pictures. The

Maya wrote both logographically and phonetically, and within their

phonetic system alone, the Maya had multiple options. All English words

are formed from various combinations of only 26 phonetic signs. By

contrast, all Maya words can be formed from various combinations of

nearly 800 consonant-vowel glyphs, each representing a full syllable.

Sounds are formed by combining a particular consonant with one of the

five vowels (hence a syllabary, rather than an alphabet).

Because many Maya signs remain undeciphered, it‘s not possible to

state precisely the relative proportions of logographic and syllabic signs.

But a significant number of the logograms have been deciphered and the

number of deciphered syllabic signs keeps growing. Epigraphers have

filled more than half of the syllabic grid, meant to plot the consonants of

the spoken Maya language against its vowels and thus represents the

totality of signs needed to write the language. It must be remembered

that the discovery of the structure of the syllabic elements—Knorozov‘s

main contribution—was made a little more than 30 years ago.

Furthermore, the consonant-vowel syllables that are already understood

are the common ones.

Nonetheless, the pace of phonetic decipherment is bound to increase in

the coming years as more resources are trained on it. One aspect of

Maya writing that may complicate this progress is the fact that different

signs can be allographs. Such equivalences are common in Maya texts

(there are at least five different signs that could be chosen to represent

the Maya syllable ba). Each scribe chose from several different signs to

convey the sounds. In evaluating a particular phonetic interpretation of a

syllable, it‘s helpful to identify as many as possible of the variant forms;

so the process of recognizing allographs depends on the slow work of

comparing many texts in order to find variant spellings of the same word.

Reading Comprehension - IPM

Page 68: QUESTION BANK - edumentor.co.inedumentor.co.in/Downloads/2018/Extra Content... · of India's oldest and most important uranium mine, ... Punjab banned e-cigarettes last year and Maharashtra

1. The author mentions Knorozov in the third paragraph in order to:

A. prove that the recent discovery of Maya signs has led to its lack of

decipherment.

B. offer an explanation for what may appear to be a relative paucity in

the completion of the Maya sign syllabic grid.

C. argue that expert linguists have been stymied in their attempts to

decipher and understand many allographic Maya signs.

D. show how the understanding of other linguistic structures may

improve the comprehension of Maya syllabic signs.

E. weaken the argument stated in the previous paragraph

2. As used in the passage by the author, the term ―logographic‖ most closely

refers to:

A. a written phonetic representation of a word.

B. a syllabic division of an individual word.

C. an imagistic representation of an idea.

D. a visual picture of an idiomatic phrase.

E. the process of designing a logo

3. The author of the passage would be LEAST likely to agree with which of the

following statements?

A. Languages whose writing is composed of pictorial signs can

demonstrate a remarkable degree of complexity and detail.

B. Linguistic signs based on syllabic or phonetic coding may be easier to

decipher than those based on visual images.

C. Logographic languages are restricted to the expression of simple

ideas because of their emphasis on image.

D. The existence of allographs in Maya signs indicates the complexity of

this linguistic system.

E. The Mayans made use of both logographics as well as phonetics

Reading Comprehension - IPM

Page 69: QUESTION BANK - edumentor.co.inedumentor.co.in/Downloads/2018/Extra Content... · of India's oldest and most important uranium mine, ... Punjab banned e-cigarettes last year and Maharashtra

Passage – 23

Although he rejected the prevailing Neo-Romanticism of the late forties

and early fifties, Philip Larkin was no admirer of modernism. Like many in

the English middle-class, for example, he thought Picasso a fake, and

believed that an artist should ―make a horse look like a horse.‖

When some disparaged his work as ―limited‖ and ―commonplace,‖

Larkin replied, ―I‘d like to know what dragon-infested world these lads

live in to make them so free with the word ‗commonplace‘.‖ His irritation

stemmed from his view that poetry ―was an act of sanity, of seeing things

as they are.‖ He thought that the connection between poetry and the

reading public, forged in the 19th century by such poets as Kipling,

Housman and Brooke, had by the mid-20th century been destroyed by

the growing unintelligibility of English poetry to the general reader. He

attributed this in part to the emergence of English literature (along with

the other arts) as an academic subject, demanding poetry that required

elucidation.

He saw no such need to explain his own work. When asked to expand

on The Whitsun Weddings, he remarked that the intent of each poem was

clear enough in itself, and he would only add that ―the poems had been

written in or near Hull, Yorkshire, with a succession of 2B pencils during

the years 1955 to 1963.‖ Influenced by the poetry of Thomas Hardy, he

made the mundane details of his life the basis for tough, unsparing,

memorable poems that rejected the Victorian belief in a benevolent God,

exploring life with a post-religious stoicism. The poems themselves are

deceptively simple. Through the details of advertisements, train-stations,

and provincial towns, they transform into something elevated and

strangely beautiful the central issues of ordinary life in the language of

ordinary speech. His underlying themes of love, solitude, and mortality

express intense personal emotion while they strictly avoid sentimentality

or self-pity, using rough-hewn rhythms and colloquial diction with an

extraordinary variety of meters and stanzaic forms. These qualities were

quickly identified, if not always appreciated, by reviewers. As the critic

Donald Hall put it (only half-admiringly), ― [Larkin‘s poem] ‗At Grass‘ is

the best horse picture ever painted.‖

Some critics went so far as to call him anti-social. In an interview,

Larkin questioned why he was described a melancholy man, protesting—

self-deprecatingly—that he was actually ―rather funny.‖ Neither of these

adjectives reflect the beauty of his poetry that is the source of a deep,

abiding pleasure.

Philip Larkin earned a living as a librarian until his death of cancer in

1985. His first poem was published in 1940, but he earned his reputation

as one of England‘s finest poets with the publication of The Less Deceived

in 1955, which was subscribed to by almost all recognized young English

poets: Amis, Bergonzi, Boyars, Brownjohn, Conquest, Davie, Enright,

Hamburger, Hill, Jennings, MacBeth, Murphy, Thwaite, Tomlinson, and

Wain. His status was confirmed with the release in 1963 of The Whitsun

Weddings (the title poem of which may be the finest in all his work), and

again with High Windows in 1974. The mood of each of these thin

volumes changed considerably from poem to poem; but, for all their

Reading Comprehension - IPM

Page 70: QUESTION BANK - edumentor.co.inedumentor.co.in/Downloads/2018/Extra Content... · of India's oldest and most important uranium mine, ... Punjab banned e-cigarettes last year and Maharashtra

range, they were clearly the products of a singular and accomplished

poetic sensibility.

1. The author quotes Larkin as saying ―I‘d like to know what dragon-infested

world these lads live in to make them so free with the word

‗commonplace‘‖ in lines 6-7 in order to:

A. show how Larkin dismissed critics of his work by pointing out their

personal failings.

B. show how Larkin mocked his critics for implying that everyday

experience must be trivial.

C. suggest that Larkin‘s critics attacked his work to make their own lives

seem more glamorous.

D. show that Larkin did not believe that the events he wrote about were

actually common.

E. show how deeply saddened Larkin was at the criticism of his work

2. The author‘s primary concern in this passage is to:

A. show that Larkin‘s verse was informed by his views on poetry.

B. describe how Larkin created verse of lasting value based on ordinary

events.

C. compare schools of poetry from the 19th and 20th centuries.

D. explain how the general reader became alienated from English poetry

by the mid-20th century.

E. criticise Larking for writing fanciful and esoteric poetry

3. The author cites the description of one of Larkin‘s poems by one of his

critics as ―the best horse picture ever painted.‖ This quotation serves

several purposes, including to demonstrate:

I. that critics considered Larkin‘s poetry poor and funny.

II. the commonplace subject matter of Larkin‘s work.

III. that critics often blurred Larkin‘s poetry with Larkin‘s views.

A. I only

B. I and II only

C. II and III only

D. II only

E. I, II and III

4. Based on the information provided in the passage, we can assume that

Larkin would be LEAST likely to write a poem taking as its subject:

A. a devout song of praise to God.

B. the working day of a London businessman.

C. the death in war of an upper-class academic.

D. a current, happy love affair.

E. a day in the life of a schoolboy

Reading Comprehension - IPM

Page 71: QUESTION BANK - edumentor.co.inedumentor.co.in/Downloads/2018/Extra Content... · of India's oldest and most important uranium mine, ... Punjab banned e-cigarettes last year and Maharashtra

Passage – 24

The media‘s particular understanding of the ways of influence and

decision-making in government colours the way they describe political

reality. It also defines their responsibility in reporting that reality;

contemporary reporters are in many ways the grandchildren of the

Progressive muckrakers.

Few aspects of American politics reinforce this Progressive world-view

as effectively as the American way of campaign finance. In assuming that

public officials defer to contributors more easily than they do to their

party, their own values, or their voting constituency, one has the perfect

dramatic scenario for the triumph of wealthy special interests over the

will of majorities and the public interest.

Much has been made recently about campaign finance reform. Various

politicians and voters' rights groups have petitioned for a reworking of

the campaign finance laws that govern how political candidates can solicit

and spent money on their races for office.

―Bias‖ is a word with many meanings. It suggests a single

explanation—one of conscious, even wilful preference—for a range of

instances in which the message misinterprets or misconveys the reality.

The media have been attacked as biased in a partisan direction by both

Democrats and Republicans, and from both the left and the right. To be

sure, media partisanship was apparent in earlier times, when the partisan

press was little more than a propagandist for the party it favoured.

But that overtly biased style seems to have given way in the 20th

century to a media more concerned with gaining audience than political

proselytes, and an electronic media fearful of government regulation if it

strays into political controversy. Few objective observers of, for instance,

the reporting of campaign finance would argue that conventional biases

are operating here. Rather one has to look to more intrinsic and ingrained

forms, to the structural biases of American newspapers and the political

assumptions of their reporters, editors, and headline-writers. Structural

biases are rooted in the very nature of journalism—in its professional

norms, in marketplace imperatives, in the demands of communicating

information to an unsophisticated audience.

Stories need identifiable actors, understandable activity, and elements of

conflict, threat or menace. They cannot be long, and must avoid

complexity—must focus on the horserace rather than on the substance of

a campaign; on controversy, personalities and negative statistics rather

than on concepts. These define the ―good‖ story.

Systematic bias and political assumption, finally, meet in an analytical

conundrum. A systematic bias dictates that newspapers print stories that

will be read. But does the press publish the story because readers have

been conditioned by newspapers to accept and believe such accounts, or

does it publish the story because of its conviction that it represents

political truth? Is there really any difference? Ultimately, the Progressive

view of reality becomes a part of the imperatives of publishing a

newspaper.

Reading Comprehension - IPM

Page 72: QUESTION BANK - edumentor.co.inedumentor.co.in/Downloads/2018/Extra Content... · of India's oldest and most important uranium mine, ... Punjab banned e-cigarettes last year and Maharashtra

1. In the course of presenting his arguments, the author suggests that

structural biases in American journalism result primarily—but not

necessarily exclusively—from:

A. problems intrinsic to the publishing and marketing of newspapers.

B. suppositions of journalists about the integrity of public officials.

C. reporters‘ cynicism about the public‘s level of intelligence.

D. growing competition among newspapers for a shrinking audience.

E. increasing influence of foreign nations

2. According to the passage, which of the following would indicate structural

biases inherent in journalists‘ work?

A. An article that adheres loyally to Progressivist dictates

B. An article that successfully masks its biased opinions

C. An article that is informed by political sophistication

D. An article that is entertaining and easy to comprehend

E. An article that criticises the current government

3. Which of the following best describes the ―analytical conundrum‖ referred

to in the sentence, ―Systematic bias and political assumption, finally,

meet in an analytical conundrum,‖ in the last paragraph?

A. Newspapers promote Progressive ideas in which they do not believe.

B. Since systematic biases and political assumptions have similar

effects, it is difficult to differentiate their roles in journalistic

publishing decisions.

C. Systematic biases and political assumptions exert contradictory and

conflicting pressures on newspaper publishers.

D. Readers‘ preferences for dramatic news accounts reflecting

Progressive ideas, rather than journalists‘ objective understanding

of the political system, determine what is published.

E. the confusion over what types of articles to publish in newspapers

Reading Comprehension - IPM

Page 73: QUESTION BANK - edumentor.co.inedumentor.co.in/Downloads/2018/Extra Content... · of India's oldest and most important uranium mine, ... Punjab banned e-cigarettes last year and Maharashtra

Passage – 25

Since the time of Darwin, morphological structures have been used to

identify phylogenetic relations. For example, the similarity between a

man‘s arm and a bat‘s wing is taken as evidence of their common origin.

There are innumerable examples of this in nature. From the whiskers of

lions and domestic cats to the bone structure in the fins of a whale and

that of a human hand, it seems one would be hard pressed to fine an

attribute in a particular species that did not illustrate some kind of

relationship to another species.

Similarities in behaviour patterns can also serve in reconstructing

evolutionary history. It is not always clear, however, how certain types of

innate behaviour evolved through natural selection. In its modern form

the Darwinian interpretation of evolution asserts that evolution consists of

changes in the frequency of appearance of different genes in populations,

and that the frequency of the appearance of a particular gene can only

increase if the gene increases the ―Darwinian fitness‖ (the expected

number of surviving offspring) of its possessors.

The discovery of a genetic predisposition to be especially responsive to

certain stimuli was an important contribution to the study of evolution.

Genetically determined responses must be subject to the pressures of

natural selection. Hence innate behaviour must evolve. Ethologists were

able to show how a motor pattern employed in a noncommunicatory

context such as feeding could evolve into a ritualized form employed as a

signal in, say, courtship.

Differentiation in innate behaviour patterns could be traced to selection

pressures arising from the environment. There are many instances of

animal behaviour patterns that seem not to contribute to the survival of

the individual displaying that behaviour. The classic example is the

behaviour of the worker bee: this insect will sting an intruder and thereby

kill itself in defense of the hive. The problem is evident: How can a gene

that makes suicide more likely become established? The concern over this

type and other types of apparently anomalous behaviour led to the

development of a new phase in the study of the evolution of behaviour: a

marriage of ethology and population genetics.

Animal behaviour was formerly thought to consist of simple responses,

some of them innate and some of them learned, to incoming stimuli.

Complex behaviour, if it was considered at all, was assumed to be the

result of complex stimuli. Over the past 60 years, however, a group of

ethologists, notably Konrad Lorenz, Nikolaas Tinbergen and Karl von

Frisch, have established a new view of animal behaviour. Studying whole

patterns of innate animal behaviour in natural environments (rather than

focusing primarily on learned behaviour, as animal behaviourists do),

they have shown that the animal brain possesses certain specific

competences, that animals have an innate capacity for performing

complex acts in response to simple stimuli. As Gould put it in 1982,

―Rather than encompassing merely the rigid and impoverished

behavioural repertoire of primitive organisms, instinct has been shown to

possess a stunning flexibility and overwhelming richness. As a result, we

Reading Comprehension - IPM

Page 74: QUESTION BANK - edumentor.co.inedumentor.co.in/Downloads/2018/Extra Content... · of India's oldest and most important uranium mine, ... Punjab banned e-cigarettes last year and Maharashtra

no longer need to invoke the barren behaviouristic tenet of learning as an

‗explanation‘ of complexity.‖

1. In the context of the arguments being made by the author in this passage,

the term ―phylogenetic‖ (line 2) most closely means:

A. structural.

B. inter-species.

C. innate.

D. functional.

E. acquired

2. Which of the following scenarios would be most analogous to the example

given by the author of the worker bee?

A. A male spider reacts to intruding predators by releasing venom that

kills both the predator and itself.

B. A female marsupial abandons her weakest offspring as prey for her

natural enemies in order to protect the rest of her brood.

C. The youngest member of a canine pack sacrifices himself by fatally

wounding an attacking predator so that the pack itself can escape.

D. A drone ant kills an insect preying on his collective by stinging the

insect‘s eyes.

E. A young Cheetah tries to hunt and fails repeatedly

3. The author of the passage would be most likely to agree with which of the

following statements?

A. Unusual animal behaviours can be understood in terms of natural

selection when they are studied in the context of procreation

patterns and needs for survival of that particular species.

B. Overpopulated animal colonies often weed out their excess or weak

members by abandoning them to their natural predators.

C. Darwin‘s evolutionary theories of natural selection have been

unnecessarily modified by modern scientists in order to make them

accord with observations of animal behaviour patterns.

D. The evolution of certain types of innate animal behaviour

demonstrate the inadequacy of the notion of ―Darwinian fitness‖ as

an approach to studying evolution.

E. There are some natural phenomena that cannot be explained by

logic

Reading Comprehension - IPM

Page 75: QUESTION BANK - edumentor.co.inedumentor.co.in/Downloads/2018/Extra Content... · of India's oldest and most important uranium mine, ... Punjab banned e-cigarettes last year and Maharashtra

Passage – 26

Before there were books, before, even, there was the written word in

civilization, there must surely have been stories told. Relating stories to

one another is a unique way that we, as humans, communicate thoughts,

needs, desires, and instruction. Whether it be the true story of what

happened on the way to the well yesterday—a story meant to instruct

about the latest water situations—or a dramatic retelling of a long-ago

battle—a cautionary tale meant to warn against unnecessary warfare—

stories have the unique ability to bring home information and instruct in a

way a mere recitation of the facts cannot.

The Tale, the Parable, and the Fable are all common and popular

modes of conveying instruction—each being distinguished by its own

special characteristics. The true Fable, if it rises to its high requirements,

ever aims at one great end and purpose: the representation of human

motive, and the improvement of human conduct, and yet it so conceals

its design under the disguise of fictitious characters, by clothing with

speech the animals of the field, the birds of the air, the trees of the wood,

or the beasts of the forest, that the reader receives the advice without

perceiving the presence of the adviser. Thus the superiority of the

counsellor, which often renders counsel unpalatable, is kept out of view,

and the lesson comes with the greater acceptance when the reader is led,

unconsciously to himself, to have his sympathies enlisted on behalf of

what is pure, honourable, and praiseworthy, and to have his indignation

excited against what is low, ignoble, and unworthy.

The true fabulist, therefore, is charged with a most important

function. He is neither a narrator, nor an allegorist, he is a great teacher,

a corrector of morals, a censor of vice, and a commender of virtue. In

this consists the superiority of the Fable over the Tale or the Parable. The

fabulist is to create a laugh, but yet, under a merry guise, to convey

instruction. Phaedrus, the great imitator of Aesop, plainly indicates this

double purpose to be the true office of the writer of fables.

The Fable partly agrees with, and partly differs from the Tale and the

Parable. It will contain, like the Tale, a short but real narrative; it will

seek, like the Parable, to convey a hidden meaning, not so much by the

use of language, as by the skilful introduction of fictitious characters; and

yet unlike to either Tale or Parable, it will ever keep in view, as its high

prerogative, and inseparable attribute, the great purpose of instruction,

and will necessarily seek to inculcate some moral maxim, social duty, or

political truth.

The Tale consists simply of the narration of a story either founded on

facts, or created solely by the imagination, and not necessarily associated

with the teaching of any moral lesson. The Parable is the designed use of

language purposely intended to convey a hidden and secret meaning

other than that contained in the words themselves; and which may or

may not bear a special reference to the hearer, or reader.

Reading Comprehension - IPM

Page 76: QUESTION BANK - edumentor.co.inedumentor.co.in/Downloads/2018/Extra Content... · of India's oldest and most important uranium mine, ... Punjab banned e-cigarettes last year and Maharashtra

1. The passage suggests that the fable is superior to the parable and the tale

for none of the following reasons EXCEPT:

I. the fable contains a moral lesson within its narrative.

II. the parable‘s message may be too enigmatic for a reader to

comprehend.

III. the tale is a chronicle of recent historical events.

A. I only

B. I and II

C. II and III

D. I, II, and III

E. None of the above

2. According to the passage, which of the following is NOT a requirement for a

narrative text to be classified as a fable?

A. Use of fictional characters, such as personified animals and natural

objects

B. Inclusion of social, moral, or political references relevant to

contemporary readers

C. Constant awareness of and attention to a particular instructional goal

D. Figurative or poetic language to demonstrate the author‘s creative

talent

E. Every fable must have a ‗moral‘ at the end

3. Which of the following best characterizes the claim that the fabulist is a

―great teacher, a corrector of morals, a censor of vice, and a commender of

virtue?‖

A. It is an analysis of the importance of the fabulist‘s role in society.

B. It is a conclusion that fabulists should be honoured above writers of

parables or tales.

C. It is appreciation for the fabulist‘s ability to multi-task.

D. It advocates increased honour and respect for the fabulist.

E. It suggests that more and more people should become fabulists

Reading Comprehension - IPM

Page 77: QUESTION BANK - edumentor.co.inedumentor.co.in/Downloads/2018/Extra Content... · of India's oldest and most important uranium mine, ... Punjab banned e-cigarettes last year and Maharashtra

Passage – 27

For better or for worse, race places a very large part in people‘s

perception of others in the world and in their own society. The notion of

one‘s own race often influences the actions and interests of an individual

(either towards or away from activities typically associated with a group)

and, in many cases, the perception of another individual‘s race influences

the perceiver‘s actions toward that individual. Races are inaccurate as

biological categories. The existence of racism, and the genesis of our

racial taxonomies themselves in the history of colonialism and slavery,

argue for abandoning racial categories altogether.

Few would deny the importance of racial categories in our everyday

lives, nor the social problems and conflict race has caused. Less well

known are the scientific problems with race: racial categories cannot be

reconciled with what scientists know about human biological diversity.

Biological races are branches of a species that have been unable to

reproduce with each other for a significant period of time. Their

separation may be due to geographic or other barriers, but anatomically,

members of different races can interbreed, since they are of the same

species.

Breeds of domesticated dogs are an example of races cultivated by

humans. In contrast, human groups have interbred for our entire history

as a species, and none have been isolated long enough to be considered

true races. The American racial classification system is no more

scientifically valid than are other racial taxonomies, local conceptions of

race affirmed in other societies or countries. Racial taxonomies in

different countries are not biological races, but rather what anthropologist

Charles Wagley calls ―social races.‖

Racial classification presupposes that people with certain phenotypes

share a common recent ancestry that others do not share. However,

physical traits are not a reliable indicator of recent shared descent. There

are no sharp borders between human groups, as there are between so-

called races, because physical traits change gradually. Anatomical

features in human populations represent adaptations to evolutionary

forces: skin colour is an adaptation to latitude, facial shape to climate or

altitude, and blood type to endemic diseases. Any particular trait is

shared by groups of people of varied heritages, people who adapted to

similar conditions in different parts of the world. Since different features

do not vary together, no assortment of traits can accurately delineate any

group as a true race.

If race were biological, different societies would understand race in

similar ways. In fact, societies use widely varying criteria to determine

race. Nor are these criteria all internally consistent. Although most

Americans believe that appearance or genetics form the basis of race, in

the United States, a person's race, legally, is determined by his ancestry,

the race of his parents. Further, some state laws, legacies of slavery,

place biracial individuals into the race of the minority parent, without

regard to chromosomes or physical appearance. In Brazil, on the other

hand, people do not consider ancestry when identifying a person's race,

and there exist many more racial categories than in the US. Race in Brazil

Reading Comprehension - IPM

Page 78: QUESTION BANK - edumentor.co.inedumentor.co.in/Downloads/2018/Extra Content... · of India's oldest and most important uranium mine, ... Punjab banned e-cigarettes last year and Maharashtra

derives solely from appearance. One's race can change from day to day,

and may differ from the race of one's family, including that of full

siblings. The racial taxonomies in Brazil and the United States differ, but

neither one is based on scientific principles.

1. The passage cites Brazil as a country in which racial definitions have a

meaning that might not otherwise be readily apparent. The author suggests

that a person who is Brazilian might change his race by:

A. altering his birth certificate.

B. marrying a person of a different race.

C. having his DNA tested.

D. getting a sun tan.

E. surrendering his passport

2. The overall purpose of this passage is to:

A. present a hypothesis that may explain a recent discovery.

B. compare and contrast two methods of classification.

C. criticize the basis of a popular belief.

D. describe worldwide variations in a cultural phenomenon.

E. praise a widely accepted belief

3. The author of this passage would be most likely to agree with which of the

following statements about abandoning racial classification?

A. We can improve our society through conscious and concerted effort.

B. The United States' racial classification system should be replaced with

that of Brazil.

C. It would be disastrous for scientists to strip people of their valued

beliefs.

D. All beliefs that are not scientifically sound should be abandoned.

E. The society has reached the point of no return and nothing can be done

to change it

4. The author presents the example of racial classification in the United

States in the passage most probably in order to show that:

A. racial taxonomies may be logically inconsistent and widely

misunderstood.

B. the system of racial classification in North America is grounded in

scientific research.

C. individuals should be allowed to choose and to change their own racial

identification.

D. racial classifications are most accurate when they take all factors

(appearance , ancestry, and DNA) into account.

E. there is not enough being done by the authorities to control this

problem

Reading Comprehension - IPM

Page 79: QUESTION BANK - edumentor.co.inedumentor.co.in/Downloads/2018/Extra Content... · of India's oldest and most important uranium mine, ... Punjab banned e-cigarettes last year and Maharashtra

Passage – 28

Gautier was indeed a poet and a strongly representative one – a French

poet in his limitations even more than in his gifts; and he remains an

interesting example of the manner in which, even when the former are

surprisingly great, a happy application of the latter may produce the most

delightful works. Completeness on his own scale is to our mind the idea

he most instantly suggests. Such as his finished task now presents him,

he is almost sole of his kind. He has had imitators who have imitated

everything but his spontaneity and his temper; and as they have

therefore failed to equal him we doubt whether the literature of our day

presents a genius so naturally perfect. We say this with no desire to

transfer Gautier to a higher pedestal than he has fairly earned – a poor

service, for the pedestal sometimes sadly dwarfs the figure. His great

merit was that he understood himself so perfectly and handled himself so

skilfully. Even more than Alfred de Musset (with whom the speech had a

shade of mock-modesty) he might have said that, if his glass was not

large, as least it was all his own glass.

There are a host of reasons why we should not compare Gautier with

such a poet as Browning; and yet there are several why we should. If we

do so, with all proper reservations, we may wonder whether we are the

richer, or, at all events, the better entertained, as a poet‘s readers should

before all things be, by the clear, undiluted strain of Gautier‘s minor key,

or by the vast, grossly commingled volume of utterance. It is idle at all

times to point a moral. But if there are sermons in stones, there are

profitable reflections to be made even on Théophile Gautier; notably this

one – that a man‘s supreme use in the world is to master his intellectual

instrument and play it in perfection.

He brought to his task a sort of pagan bonhomie which makes most of

the descriptive and pictorial poets seem, by contrast, a group of shivering

ascetics or muddled metaphysicians. He excels them by his magnificent

good temper and the unquestioning serenity of his enjoyment of the

great spectacle of nature and art. His world was all material, and its

outlying darkness hardly more suggestive, morally, than a velvet canopy

studded with silver nails. To close his eyes and turn his back on it must

have seemed to him the end of all things; death, for him, must have

been as the sullen dropping of a stone into a well. His observation was so

penetrating and his descriptive instinct so unerring, that one might have

fancied grave nature, in a fit of coquetry, or tired of receiving but half-

justice, had determined to construct a genius with senses of a finer strain

than the mass of human family.

Reading Comprehension - IPM

Page 80: QUESTION BANK - edumentor.co.inedumentor.co.in/Downloads/2018/Extra Content... · of India's oldest and most important uranium mine, ... Punjab banned e-cigarettes last year and Maharashtra

1. In the passage, the author suggests that the French poet Théophile Gautier‘s

talents included all of the following EXCEPT:

A. an innovative and unique artistic view of nature.

B. the ability to quickly and immediately compose poetry.

C. extensive training in rhetorical and literary techniques.

D. a strong understanding of his world and himself.

E. he had sharp observation powers

2. For what purpose can it reasonably be concluded does the author reference

other writers in this passage, including Musset and Browning?

A. To prove that Gautier, as a poet, was unique among his

contemporaries

B. To show that Gautier‘s poetry was representative of French lyricism at

the time

C. To criticize Gautier‘s limited talent and creativity

D. To refute the idea that Gautier‘s colleagues could easily imitate his

style

E. To prove how inferior the other writers were compared to Gautier

3. The author makes a few noteworthy remarks about Gautier‘s attitude towards

death. This attitude would most support which of the following conclusions?

A. None of Gautier‘s literary works focused on human frailty.

B. Gautier believed that people are inherently linked to the divine.

C. The fleeting passage of time was a common poetic theme that Gautier

neglected.

D. In his poetry, Gautier often focused on the vibrancy of human and

natural life.

E. Gautier was a strong believer in the theory of life after death

Reading Comprehension - IPM

Page 81: QUESTION BANK - edumentor.co.inedumentor.co.in/Downloads/2018/Extra Content... · of India's oldest and most important uranium mine, ... Punjab banned e-cigarettes last year and Maharashtra

Passage – 29

From the beginning, Johannes Kepler (1571-1630) was convinced that

the basic astronomical verities must have a geometrical interpretation.

This conviction has been shared by all the great natural philosophers,

from Pythagoras to Einstein—the conviction that the cosmos was laid out

according to a mathematical design and that this design is "simple" and

accessible to human intelligence. For Kepler, mathematics meant the

pure geometry of the Greeks.

His early scientific career is especially interesting because the ideas

that seemed to him to be the most significant, and which he tried to

exploit for the rest of his life, appear to a modern reader to be almost

completely mad. It was the fact that he could never get them to work

that drove him to make the series of astronomical discoveries that appear

to us to be so significant.

God was for Kepler a master Greek geometer, and the "book of the

world" must therefore be contained among the theorems of Euclid. One

theory was that there are only five "perfect solids." A perfect solid (the

most familiar example is the cube) is a solid all of whose faces are

"perfect" plane figures (in the cube, these figures are squares). The other

perfect solids are the tetrahedron, octahedron, dodecahedron, and

icosahedron. There were known to be six planets - Mercury, Venus,

Earth, Mars, Jupiter, and Saturn, in order of increasing distance from the

sun, around which, Kepler believed, the planets moved in circular orbits.

Carrying on with his geometry, he considered a universe in which a

cube, a tetrahedron, a dodecahedron, an icosahedron, and an octahedron

would be arranged concentrically, one inside another; the orbit of

Mercury would be fitted within the first of these perfect solids, the orbit of

Venus outside it, and outside each of the other solids the orbit of another

planet. This, he thought, might make it possible to calculate the

interplanetary distances and also explain why there were no more than

six planets.

With the superior vision of hindsight, it is all too easy for us to pass

judgment on the weakness of Kepler's youthful notion. (Apart from

anything else, we know that there are nine planets.) In fact, however,

had Kepler's mysticism not also been coupled with a fanatic obsession to

make his theory fit the observed facts quantitatively, he might as well

have gone down in scientific history as just another visionary crank,

along with the more unenlightened alchemists who abounded at that

time.

It is interesting to note that Newton also devoted his "spare" time to

alchemy. What would have driven this man of science, this father of our

modern physics, to spend his free time trying to turn base metals into

gold? Undoubtedly, this fact shows us that the desire for wealth often

trumps the pursue of pure science, even in the most noteworthy of

individuals. This combination of mysticism and devotion to the "facts" as

he knew them was Kepler's great strength. Einstein characterized the

interrelation between mystic intuition and the need to deal with hard

Reading Comprehension - IPM

Page 82: QUESTION BANK - edumentor.co.inedumentor.co.in/Downloads/2018/Extra Content... · of India's oldest and most important uranium mine, ... Punjab banned e-cigarettes last year and Maharashtra

facts as a formula that "Science without religion is lame. Religion without

science is blind.‖

1. Which of the following statements most nearly captures the author‘s central

argument as articulated in the passage?

A. The originality of Kepler‘s early scientific work can be fully appreciated

by studying its influence on the mature work of Newton and Einstein.

B. Kepler's early beliefs were often erroneous, but his mysticism coupled

with an attachment to scientific fact led to many of his later, key

discoveries.

C. Kepler laid the groundwork for our current understanding of the

universe in his early studies of the pure geometry of the Greeks.

D. An investigation of Kepler's youthful work yields relatively few clues

about the method he employed in his most remarkable work.

E. Kepler‘s early beliefs were more accurate compared to his later beliefs

2. The passage suggests that which of the following scientific beliefs held by

Kepler in his youth was, in fact, correct?

A. The planets are arranged concentrically, within perfect solids.

B. The orbit of the planets are circular.

C. The number of perfect solids is equal to the number of planets

D. There is an underlying order to the cosmos which is accessible to the

human intelligence.

E. Humans can never fully understand the mysteries of the universe

3. The author quotes Einstein in the sixth paragraph. His primary purpose in

doing this is to:

A. suggest that Kepler's thought was misconstrued by Einstein.

B. clarify a difference between scientific and religious thought.

C. indicate the extent of Einstein's personal admiration of Kepler.

D. emphasize a particular attribute of Kepler's own method and outlook.

E. point out a flaw in Kepler‘s methodolgy

4. Which of the following statements is implied by the author in paragraphs five

and six?

A. The history of science is full of scientists who have failed to esteem

what was of greatest significance in their own work.

B. It is during periods of youthful enthusiasm that the fundamental

guidelines to the most important scientific discoveries nearly always

emerge.

C. Such is the paradox of the human personality that, despite such

problems, Kepler became one of the most determined seekers of

cosmic harmony in history.

D. Kepler, too, was aware of the dangers of pure speculation conducted

without taking into consideration observed phenomena

E. It is very easy to blame Kepler for his weaknesses bur perhaps not very

appropriate to do so

Reading Comprehension - IPM

Page 83: QUESTION BANK - edumentor.co.inedumentor.co.in/Downloads/2018/Extra Content... · of India's oldest and most important uranium mine, ... Punjab banned e-cigarettes last year and Maharashtra

Passage – 30

Without entering now into the why, let me observe that the printer may

always ascertain when the dash of the MS is properly and when

improperly employed, by bearing in mind that this point represents a

second thought⎯an emendation. In using it just above I have exemplified

its use. The words ―an emendation‖ are, speaking with reference to

grammatical construction, put in apposition with the words ―a second

thought.‖ Having written these latter words, I reflected whether it would

not be possible to render their meaning more distinct by certain other

words.

Now, instead of erasing the phrase ―a second thought,‖ which is of

some use⎯which partially conveys the idea intended⎯which advances me

a step toward my full purpose⎯I suffer it to remain, and merely put a

dash between it and the phrase ―an emendation.‖ The dash gives the

reader a choice between two, or among three or more expressions, one

of which may be more forcible than another, but all of which help out the

idea.

It stands, in general, for the words⎯“or, to make my meaning more

distinct.” This force it has⎯and this force no other point can have; since

all other points have well-understood uses quite different from this.

Therefore, the dash cannot be dispensed with. It has its phases⎯its variation of the force described; but the one principle⎯that of second

thought or emendation⎯will be found at the bottom of all. That

punctuation is important all agree; but how few comprehend the extent

of its importance!

The writer who neglects punctuation, or mis-punctuates, is liable to

be misunderstood⎯this, according to the popular idea, is the sum of the

evils arising from heedlessness or ignorance. It does not seem to be

known that, even where the sense is perfectly clear, a sentence may be

deprived of half its force⎯its spirit⎯its point⎯by improper punctuation. For

the want of merely a comma, it often occurs that an axiom appears a

paradox, or that a sarcasm is converted into a sermonoid. There is no

treatise on the topic⎯and there is no topic on which a treatise is more

needed.

There seems to exist a vulgar notion that the subject is one of pure

conventionality, and cannot be brought within the limits of intelligible and

consistent rule. And yet, if fairly looked in the face, the whole matter is

so plain that its rationale may be read as we run. If not anticipated, I

shall, hereafter, make an attempt at a magazine paper on ―The

Philosophy of Point.‖ In the meantime let me say a word more of the

dash.

Every writer for the press, who has any sense of the accurate, must

have been frequently mortified and vexed at the distortion of his

sentences by the printer‘s now general substitution of a semicolon, or

comma, for the dash in the MS. The total or nearly total disuse of the

latter point, has been brought about by the revulsion consequent upon its

excessive employment about twenty years ago. The Byronic poets were

all dash.

Reading Comprehension - IPM

Page 84: QUESTION BANK - edumentor.co.inedumentor.co.in/Downloads/2018/Extra Content... · of India's oldest and most important uranium mine, ... Punjab banned e-cigarettes last year and Maharashtra

1. The passage indicates that if given the chance to respond to the following

claims, the author is LEAST likely to agree with which of the following

statements?

A. There is a single ideal way in which any thought can be expressed.

B. The rules of punctuation are simple and rational.

C. Punctuation helps to convey the writer‘s intended meaning and tone.

D. Most people do not understand the correct use of punctuation.

E. The full stop is superior to the comma

2. The author most likely mentions his intention to write an article entitled

―The Philosophy of Point‖ in order to:

A. remind the reader that grammar is a branch of philosophy.

B. indicate the possibility of explaining correct punctuation concisely.

C. furnish his own credentials as an expert on punctuation.

D. emend his statement about punctuation.

E. point out the similarity between punctuation and philosophy

3. According to the passage, which of the following is true of the relationship

between words or phrases separated by a dash?

A. Each word or phrase partially conveys the author‘s meaning.

B. The second word or phrase renders the first one superfluous.

C. The first word or phrase states the main topic, and the second states

the sub-topic.

D. The two words or phrases pertain to separate topics.

E. The dash can be easily replaced with a colon

Reading Comprehension - IPM

Page 85: QUESTION BANK - edumentor.co.inedumentor.co.in/Downloads/2018/Extra Content... · of India's oldest and most important uranium mine, ... Punjab banned e-cigarettes last year and Maharashtra

Passage – 31

In the fast new choreography of American compassion, explanation is

twirled into excuse, and the spotlight‘s shine endows feelings with a

prominence that facts could only hope for. Perception has become more

important than reality. In homes, classrooms, and workplaces, we prefer

to understand viewpoints rather than discern truths.

After recounting the prevalent view of Nicholas II, which faults the

last czar for failure to recognize dire conditions of the day, neglect of

astute advisors, and reliance instead on sources incompetent to influence

state behaviour, Y. S. Bark, in Nicholas the Unlucky, concedes that

Nicholas was a poor leader. However, Bark‘s main contention is that

Nicholas II was a doomed figure who had the misfortune of presiding

over, but not responsibility for significantly contributing to, the

calamitous demise of Czarist Russia in 1917.

The product of an accomplished historian known for nice scholarship

on inter-war diplomatic history, Bark‘s first foray into popular political

biography proceeds with a deft review of the social, economic, and

political conditions of Nicholas‘s day. In every respect but governance,

Nicholas‘s Russia was, or was rapidly becoming, modern. Political

alliances with Europe proper had existed for centuries, as had kinship

with European art and literature. Developments in technology,

communication, and transportation only increased the magnitude of

Russia‘s European-ness.

After 1860, even Russian economic life began, however

embryonically, to resemble Western forms. Only governance remained

unchanged, yet it was governance that most needed transformation.

Then begins a confused attempt to vindicate Nicholas: ―At the time, calls

came for a compromise of czarism, yet it was in their tradition that the

czars saw the sine qua non of Russian life. This was the impossible

situation confronting Nicholas. Given these circumstances, it is

implausible to suppose that Nicholas should have viewed the

abandonment or even compromise of autocracy as Russia‘s salvific hope.

To the contrary, turbulent times are perfect for redoubling the faith of

ages; the first reaction to discomforting ideas is hatred. (The rest is

detail—witness history‘s smile on stalwart Woodrow Wilson.)‖

Nicholas the Unlucky is ultimately unsatisfying because Nicholas is a

poor choice for arguing historical inevitability and historical compassion.

Worthwhile sources claim, not that Nicholas originated the causes of the

revolution, but that at best he did nothing to alleviate them, and at worst

he intensified them. Monarchists‘ astute, if reluctant, embrace of

modernity in Prussia and Japan attests to how the demise of monarchy

can be delayed. And while, like Nicholas, the Hohenzollerns of Austria-

Hungary did not outlast World War I, they had faced the assault of

modernity beginning much earlier, and probably would have fallen earlier,

in 1848, had they behaved as Nicholas did.

To demonstrate Nicholas‘s unshakable faith in the czarist tradition, Bark

devotes an entire section to Count Pobedonostsev, by whom Alexander

III, Nicholas‘s father, was tutored in childhood and closely advised as

Reading Comprehension - IPM

Page 86: QUESTION BANK - edumentor.co.inedumentor.co.in/Downloads/2018/Extra Content... · of India's oldest and most important uranium mine, ... Punjab banned e-cigarettes last year and Maharashtra

Czar. A singular influence on Nicholas‘s own development, Pobedonostsev

in his memoirs wrote of ―…Parliamentarism, which…has deluded much of

the so-called ‗intelligence‘…although daily its falsehood is exposed more

clearly to the world.‖ Grounded in the inalienable Russian truth that the

czar was ―the Little Father, God‘s chief earthly agent and protector,‖

Nicholas‘s commitment to autocracy, in Bark‘s view, rendered major

reform unthinkable.

1. As used in the end of the fourth paragraph in the statement: ―The rest is

detail—witness history‘s smile on stalwart Woodrow Wilson,‖ the words ―The

rest is detail‖ refer to:

A. Bark‘s belief that popular commitment to core values, even though the

values are subjective, is essential to persevering through periods of

national turmoil.

B. Bark‘s implication that policies advanced by Woodrow Wilson, though

more successful than those of Nicholas, similarly reflected a strong

commitment to traditional beliefs.

C. the author‘s contention that weighing the merits of alternative reform

policies is less important than a ruler‘s overall commitment to reform.

D. the author‘s assumption that Woodrow Wilson‘s activist policies do not

constitute a reasonable basis for comparison to Nicholas‘s

conservative policies.

E. to illustrate that a particular policy of Woodrow Wilson was much more

important than the rest

2. The author‘s discussion in the passage of the Hohenzollerns assumes which of

the following?

A. In at least some significant ways, the political challenges faced by the

rulers of Austria-Hungary around 1848 resemble those faced by

Nicholas around 1917.

B. Like Nicholas, Hohenzollern rulers perceived themselves as having not

only a historical, but also a divine, mandate.

C. For the purposes of historical analysis, modernity and European-ness

can be treated as interchangeable terms.

D. Nicholas should have implemented the same policy reforms as those

affected by rulers in Japan, Prussia, and Austria-Hungary.

E. There is no similarity between the Hohenzollems rulers and Nicholas

3. Which of the following, if true, would most challenge the author‘s assertion

that ―the compassion craze has swept up biography?‖

A. Most readers regard as unflattering Bark‘s portrayal of Count

Pobedonostsev in Nicholas the Unlucky.

B. For their subjects, many biographers choose figures who the biographers

believe ought to be viewed in a forgiving and sympathetic light.

C. Nicholas genuinely believed that his attempt to preserve czarism was in

the best interest of the Russian people.

D. Several decades ago, when Bark wrote Nicholas the Unlucky, she had very

little exposure to American cultural values

Reading Comprehension - IPM

Page 87: QUESTION BANK - edumentor.co.inedumentor.co.in/Downloads/2018/Extra Content... · of India's oldest and most important uranium mine, ... Punjab banned e-cigarettes last year and Maharashtra

E. Due to her cosmopolitan upbringing, Bark was well exposed to American

culture

Reading Comprehension - IPM

Page 88: QUESTION BANK - edumentor.co.inedumentor.co.in/Downloads/2018/Extra Content... · of India's oldest and most important uranium mine, ... Punjab banned e-cigarettes last year and Maharashtra

Passage – 32

The extent to which analysis of social phenomena is compatible with the

scientific method is a hotly contested question. Among international

relations scholars, historico-deductivist opponents of positivism claim that

in the pursuit of objective depictions of the causes, course, and

consequences of international phenomena the character and operation of

which are purported to exist independently of the observer, positivists

miss or dismiss the implicit attitudes, values, and ideologies embedded in

their work, which personalize and subjectivize their conclusions.

Positivism, these critics contend, attempts to impose on world politics a

coherent facticity akin to that of the natural sciences, but to which the

basic nature of world politics is indisposed.

For historico-deductivists, the problem of a posteriori

overdetermination is a case in point. In the natural sciences, replicability

and verifiability afford the findings of laboratory experimentation

potentially nomothetic status. In international relations, however, such

law-like generalizations about cause and effect are rarely if ever possible,

not only because events are unique, but also because of the multiplicity

of potential causes. Whether World War I resulted from disequilibrium in

the international distribution of power, the ascendancy of government

factions committed to aggression, or the accuracy of an assassin‘s bullet,

is, ultimately, unknown. For opponents of positivism, it is better to

recognize darkness than to pretend to see light.

While some leading positivists, most notably Pastore, admit as

―knowledge‖ only the sum of all tested propositions, for most it is the

very cloudlike nature of political phenomena that requires a clocklike

approach. Conceding that their subject does not permit nomothetic

propositions, the majority of positivists appear committed to Williams‘

more moderate rule: ―The propensity to error should make us cautious,

but not so desperate that we fear to come as close as possible to

apodictic findings. We needn‘t grasp at the torch with eyes closed, fearing

to be blinded.‖

Positivists point to the potential of scientific analysis to yield

counterintuitive truths. A frequently cited example is Grotsky‘s study of

the role of non-state actors in international trade. Published at a time

when many scholars were convinced that multinational organizations had

effectively ―elbowed the traditional sovereign nation-state…out of

analytical existence in our field,‖ Grotsky‘s research of the structure,

timing, and variance of state expenditures on foreign direct investment

effectively restored the state to its position as the dominant unit in

international relations scholarship. Despite several efforts, historico-

deductivists who had championed the new relevance of non-state actors

have not, as yet, successfully refuted Grotsky‘s findings—a consideration

that bodes well for those of us who believe that an end to this

longstanding debate, which has produced much timely and relevant

research, is not necessarily to be desired.

In addition to claiming that critics have mischaracterized their

methodological commitments, positivists also contend that the historico-

deductivist approach is subject to many of the same criticisms leveled

Reading Comprehension - IPM

Page 89: QUESTION BANK - edumentor.co.inedumentor.co.in/Downloads/2018/Extra Content... · of India's oldest and most important uranium mine, ... Punjab banned e-cigarettes last year and Maharashtra

against positivism. For example, on the twentieth anniversary of her

seminal article depicting the Peloponnesian War as the archetypal case of

power politics in action, Nash, perhaps the exemplar of the historico-

deductivist school, revisited her earlier findings, only to conclude that the

interaction between the Athenians and Spartans included significant

instances of cooperation and reciprocity. Even as Nash‘s confederates

praised the ―illuminating evolution‖ in her thinking, many positivists

questioned whether Nash‘s antipodal findings corresponded to a shift in

her initial assumptions over time. The implication, of course, is that if

positivists‘ commitments at the level of proto-theory colour their eventual

conclusions, then they are not alone in this regard.

1. According to information given by the author in the passage, which of the

following is true of a posteriori overdetermination?

I. It presents a challenge to scholars‘ ability to produce nomothetic

statements about world politics.

II. It exemplifies the analytical confusion created by unique events that

often have multiple effects.

III. It suggests that the historico-deductivism is better suited than is

positivism to the study of international relations.

A. I only

B. III only

C. I and II only

D. II and III only

E. I, II and III

2. As used in the passage by Williams at the end of the third paragraph in the

statement,, ―We needn‘t grasp at the torch with eyes closed, fearing to be

blinded,‖ the word ―torch‖ refers to:

A. propensity to error.

B. nomothetic propositions.

C. political phenomena.

D. methodological commitments.

E. myths and superstitions

3. It can reasonably be inferred that the author of the passage is a:

A. professor of history.

B. professor of international relations.

C. diplomat.

D. journalist.

E. politician

Reading Comprehension - IPM

Page 90: QUESTION BANK - edumentor.co.inedumentor.co.in/Downloads/2018/Extra Content... · of India's oldest and most important uranium mine, ... Punjab banned e-cigarettes last year and Maharashtra

Passage – 33

Between 1965 and 1970, welfare caseloads more than doubled and costs

tripled. The Nixon administration was unable to secure a legislative

majority for comprehensive welfare reform. Legislative welfare reform

raised contentious issues of who is entitled to support, how much, and on

what terms—precisely the types of issues that have defied political

resolution throughout welfare‘s history.

As a mechanism of policy change, the Nixon administration turned to

a common managerial tool—performance monitoring. Middle-level

officials at the Office of Management and Budget (OMB) and the

Department of Health, Education and Welfare (HEW) crafted quality

control—a system for monitoring the accuracy of state welfare

payments—into an instrument for indirectly influencing states to become

more restrictive in the provision of welfare. Quality control‘s manifest

purpose was to achieve fiscal accountability.

Through this instrument HEW could monitor state welfare payments

and withhold federal reimbursement from those that it deemed to be

improper. However, quality control also served a latent, political function,

partly reflected in its design. It penalized states only for overpayments

and payments made to ineligible individuals.

Quality control‘s effectiveness depended on the uncoordinated

responses of street-level bureaucrats in hundreds of local welfare offices

to new demands that administrative reform imposed at the workplace.

For example, welfare workers translated administrative concern for

procedural uniformity into demands that welfare applicants routinely

produce scores of documents of dubious relevance to their eligibility.

Applicants who could not meet these procedural demands, whether

reasonable or not, were denied welfare.

Administrative reform traded errors of liberality for errors of

stringency. Behaviours directed toward the helping aspects of welfare

policy were virtually displaced as workers responded to incentives to

maximize measured attributes of performance, namely procedural

uniformity and productivity. At the same time, worker discretion to make

unreasonable procedural demands was virtually unchecked.

Quality control did not overtly breach the integrity of theoretical

entitlement to welfare promised by statute and supported by legal

precedent. Rather, it seemed designed to protect this promise. But in

practice, quality control appears to have initiated a process of effective

disentitlement. Its adverse effects were unmeasured and unobserved,

leaving quality control‘s manifest legitimacy unimpaired. Government

institutions and officials were thus insulated from the effects of their

actions. In this sense, quality control ironically eroded the government

accountability that it was ostensibly intended to guarantee. Furthermore,

through quality control, federal authorities could indirectly influence state

administrative practices without directly encroaching on areas of nominal

state authority. Performance measurement backed by fiscal sanctions

proved to be a relatively potent, if imperfectly cast, instrument for

penetrating a decentralized bureaucracy.

Reading Comprehension - IPM

Page 91: QUESTION BANK - edumentor.co.inedumentor.co.in/Downloads/2018/Extra Content... · of India's oldest and most important uranium mine, ... Punjab banned e-cigarettes last year and Maharashtra

1. All of the following are mentioned in the passage by the author as adverse

effects of quality control EXCEPT:

A. undue emphasis on administrative paperwork and procedures.

B. arbitrary and inconsistent penalties for state welfare agencies.

C. a decrease in the number of people who were eligible for welfare

benefits.

D. lack of accountability for certain systematic infringements of the

welfare system.

E. initiation of a process of effective disentitlement

2. In paragraph 4, the phrase ―uncoordinated responses of street-level

bureaucrats‖ is used in order to:

A. support the author‘s claim that unreasonable administrative

procedures caused many applicants to be denied welfare benefits.

B. refute the theory that quality control was used to hold states to a

higher standard of accountability in their fiscal administration.

C. prove that quality control policies were implemented to serve a

political rather than a social agenda.

D. provide a potential reason for the ineffectiveness of performance

monitoring on general welfare reform.

E. criticise bureaucrats for the state of affairs with regards to quality

control

3. What does the author of the passage suggest about the use of common

managerial tools to effect policy changes in the welfare system?

A. Procedural changes in welfare agencies should be established in ways

that assure adherence to regulations for both workers and applicants.

B. Administrative reform methods like performance monitoring may

cause welfare organizations to become overly restrictive in their

policies.

C. State payments and federal reimbursement funding can be effectively

monitored through changes in welfare administration at the national

level.

D. Implementation of quality control methods helped to hold the federal

government accountable for its actions.

E. Such tools have completely failed to effect policy changes in the past

Reading Comprehension - IPM

Page 92: QUESTION BANK - edumentor.co.inedumentor.co.in/Downloads/2018/Extra Content... · of India's oldest and most important uranium mine, ... Punjab banned e-cigarettes last year and Maharashtra

Directions for Data Sufficiency Questions:

Each question is followed by two statements (1) and (2).

Mark [A] if statement 1 alone is sufficient to answer the question.

Mark [B] if statement 2 alone is sufficient to answer the question.

Mark [C] if statement 1 and 2 are together required to answer the question.

Mark [D] if both statements 1 and 2 together are also not sufficient to answer the question.

Mark [E] if either statement 1 or 2 is sufficient to answer the question.

Data Sufficiency

Page 93: QUESTION BANK - edumentor.co.inedumentor.co.in/Downloads/2018/Extra Content... · of India's oldest and most important uranium mine, ... Punjab banned e-cigarettes last year and Maharashtra

Data Sufficiency

1. A garden store purchased a number of shovels and a number of rakes. If the cost of each shovel was $14 and the cost of each rake was $9, what was the total cost of the shovels and rakes purchased by the store?

(1) The ratio of the number of shovels to the number of rakes purchased by the store was 2 to 3.

(2) The total number of shovels and rakes purchased by the store was 50.

2. Of the students who eat in a certain cafeteria, each student either likes or dislikes lima beans and each student either likes

or dislikes brussels sprouts. Of these students,32 dislike lima beans; and of those who dislike lima beans,

53 also dislike

brussels sprouts. How many of the students like brussels sprouts but dislike lima beans?

(1) 120 students eat in the cafeteria.

(2) 40 of the students like lima beans.

3.

How many different prime numbers are factors of the positive integer n ?

(1) Four different prime numbers are factors of 2n.

(2) Four different prime numbers are factors of n2.

4.

Is the product of a certain pair of integers even?

(1) The sum of the integers is odd.

(2) One of the integers is even and the other is odd.

5. If k is an integer and 2 < k < 8, what is the value of k ?

(1) k is a factor of 30.

(2) k is a factor of 12.

6.

On the number line above, is the product of w, x, y, and z negative?

(1) z is positive.

(2) The product of w and x is positive.

7. If y and z are integers, is y(z + 1) odd?

(1) y is odd

(2) z is even.

8. If n and m are positive integers, what is the remainder when 3^(4n + 2 + m) is divided by 10 ?

Page 94: QUESTION BANK - edumentor.co.inedumentor.co.in/Downloads/2018/Extra Content... · of India's oldest and most important uranium mine, ... Punjab banned e-cigarettes last year and Maharashtra

(1) n = 2

(2) m = 1

9. If x is a positive integer, is x < 16 ?

(1) x is less than the average (arithmetic mean) of the first ten positive integers.

(2) x is the square of an integer.

10.Of the 800 students at a certain college, 250 students live on campus and are more than 20 years old. How many of the

800 students live on campus and are 20 years old or less?

(1) 640 students at the college are more than 20 years old.

(2) 60 students at the college are 20 years old or less and live off campus.

11.If p is a positive odd integer, what is the remainder when p is divided by 4 ?

(1) When p is divided by 8, the remainder is 5.

(2) p is the sum of the squares of two positive integers.

12.Of the 25 cars sold at a certain dealership yesterday, some had automatic transmission and some had antilock brakes. How many of the cars had automatic transmission but not antilock brakes?

(1) All of the cars that had antilock brakes also had automatic transmission.

(2) 2 of the cars had neither automatic transmission nor antilock brakes.

13.Is x to the right of -5 on the number line?

(1) x is to the right of -7 on the number line.

(2) x is between -4 and -3 on the number line.

14.Is y between -2 and 1 on the number line?

(1) y is to the right of -1 on the number line.

(2) y is to the left of 2 on the number line.

15.Is r to the right of -6 on the number line?

(1) r is between -4 and -1 on the number line.

(2) r is between -3 and 1 on the number line.

16.Lines n and p lie in the xy-plane. Is the slope of line n less than the slope of line p ?

(1) Lines n and p intersect at the point (5,1).

Page 95: QUESTION BANK - edumentor.co.inedumentor.co.in/Downloads/2018/Extra Content... · of India's oldest and most important uranium mine, ... Punjab banned e-cigarettes last year and Maharashtra

(2) The y-intercept of line n is greater than the y-intercept of line p.

17.Six countries in a certain region sent a total of 75 representatives to an international congress, and no two countries sent the same number of representatives. Of the six countries, if Country A sent the second greatest number of representatives, did Country A send at least 10 representatives?

(1) One of the six countries sent 41 representatives to the congress.

(2) Country A sent fewer than 12 representatives to the congress.

18.What is the hundredths digit of the decimal z ?

(1) The tenths digit of 100z is 2.

(2) The units digit of 1,000z is 2.

19.Is z equal to the median of the three positive integers x, y, and z ?

(1) x < y + z

(2) y = z

20.A certain one-day seminar consisted of a morning session and an afternoon session. If each of the 128 people attending the seminar attended at least one of the two sessions, how many of the people attended the morning session only?

(1) 43 of the people attended both sessions.

(2) 87 of the people attended the afternoon session.

21.If a certain charity collected a total of 360 books, videos, and board games, how many videos did the charity collect?

(1) The number of books that the charity collected was 40 percent of the total number of books, videos, and board games that the charity collected.

(2) The number of books that charity collected was 6632 percent of the total number of videos and board games that

charity collected.

22.Of the 800 sweaters at a certain store, 150 are red. How many of the red sweaters at the store are made of pure wool?

(1) 320 of the sweaters at the store are neither red nor made of pure wool.

(2) 100 of the red sweaters at the store are not made of pure wool.

23.At least 100 students at a certain high school study Japanese. If 4 percent of the students at the school who study French also study Japanese, do more students at the school study French than Japanese?

(1) 16 students at the school study both French and Japanese.

(2) 10 percent of the students at the school who study Japanese also study French.

24.At a certain restaurant, if each hamburger costs the same amount, what is the cost, excluding sales tax, of 1 hamburger?

Page 96: QUESTION BANK - edumentor.co.inedumentor.co.in/Downloads/2018/Extra Content... · of India's oldest and most important uranium mine, ... Punjab banned e-cigarettes last year and Maharashtra

(1) The total cost, including a 6 percent sales tax, is $4.77 for 3 hamburgers.

(2) The total cost, including a 6 percent sales tax, is less than $6.50 for 4 hamburgers.

25.What is the value of n ?

(1) n is between 0 and 1.

(2) 167 is

83 more than n.

26.In a certain conference room each row of chairs has the same number of chairs, and the number of rows is 1 less than the number of chairs in a row. How many chairs are in a row?

(1) There is a total of 72 chairs.

(2) After 1 chair is removed from the last row, there is a total of 17 chairs in the last 2 rows.

27.What is the price for a certain meal listed on a menu?

(1) The total paid for the meal, sales tax, and gratuity is $10.84.

(2) The sales tax on food is 6 percent.

28.Which of Company X and Company Y earned the greater gross profit last year?

(1) Last year the expenses of Company X were 65 of the expenses of Company Y.

(2) Last year the revenues of Company X were $6 million less than the revenues of Company Y.

29.What is the value of 6x - 10 ?

(1) 3x - 5 = 16

(2) 12x - 10 = 74

30.Is w greater than 1 ?

(1) w + 2 > 0

(2) w2 > 1

31.At a refreshment stand, each can of soda sells for the same price and each sandwich sells for the same price. What is the total price for 2 sandwiches and 3 cans of soda at the stand?

(1) At the stand the total price for 1 sandwich and 1 can of soda is $3.

(2) At the stand the total price for 3 sandwiches and 2 cans of soda is $8.

32.Al, Pablo, and Marsha shared the driving on a 1,500-mile trip. Which of the three drove the greatest distance on the trip?

(1) Al drove 1 hour longer than Pablo but at an average rate of 5 miles per hour slower than Pablo.

Page 97: QUESTION BANK - edumentor.co.inedumentor.co.in/Downloads/2018/Extra Content... · of India's oldest and most important uranium mine, ... Punjab banned e-cigarettes last year and Maharashtra

(2) Marsha drove 9 hours and averaged 50 miles per hour.

33.If x =

21 , is y equal to 1 ?

(1) y2 ( x +21 ) = 1

(2) y ( 2x – 1 ) = 2x - y

34.During an experiment, some water was removed from each of 6 water tanks. If the standard deviation of the volumes of water in the tanks at the beginning of the experiment was 10 gallons, what was the standard deviation of the volumes of water in the tanks at the end of the experiment?

(1) For each tank, 30 percent of the volume of water that was in the tank at the beginning of the experiment was removed during the experiment.

(2) The average (arithmetic mean) volume of water in the tanks at the end of the experiment was 63 gallons.

35.If p and n are positive integers and p > n, what is the remainder when p2 - n2 is divided by 15 ?

(1) The remainder when p + n is divided by 5 is 1.

(2) The remainder when p - n is divided by 3 is 1.

36.If x is positive, is x > 3 ?

(1) (x - 1)2 > 4

(2) (x - 2)2 > 9

37.When 1,000 children were inoculated with a certain vaccine, some developed inflammation at the site of the inoculation and some developed fever. How many of the children developed inflammation but not fever?

(1) 880 children developed neither inflammation nor fever.

(2) 20 children developed fever.

38.

If r and s are positive integers, is sr an integer?

(1) Every factor of s is also a factor of r.

(2) Every prime factor of s is also a prime factor of r.

39.For the students in class A, the range of their heights is r centimeters and the greatest height is g centimeters. For the students in class B, the range of their heights is s centimeters and the greatest height is h centimeters. Is the least height of the students in class A greater than the least height of the students in class B ?

(1) r < s

(2) g > h

40.Beth's bank charges a service fee on a regular checking account for each month in which the balance on the account falls

Page 98: QUESTION BANK - edumentor.co.inedumentor.co.in/Downloads/2018/Extra Content... · of India's oldest and most important uranium mine, ... Punjab banned e-cigarettes last year and Maharashtra

below $100 at any time during the month. Did the bank charge a service fee on Beth's regular checking account last month?

(1) During last month, a total of $1,000 was withdrawn from Beth's regular checking account.

(2) At the beginning of last month, Beth's regular checking account balance was $500.

41.Machines X and Y work at their respective constant rates. How many more hours does it take machine Y, working alone, to fill a production order of a certain size than it takes machine X, working alone?

(1) Machines X and Y, working together, fill a production order of this size in two-thirds the time that machine X, working alone, does.

(2) Machine Y, working alone, fills a production order of this size in twice the time that machine X, working alone, does.

42.John and Mary own shares of stock in a certain company. Does John own more shares of the company's stock than Mary?

(1) Mary owns more than 500 shares of the company's stock.

(2) The number of shares of the company's stock that John owns is 400 less than twice the number of shares of the company's stock that Mary owns.

43.Each person in a certain group supports only one of the two candidates R and T. Of the people in the group, 45 percent support Candidate R and the rest support Candidate T. How many people in the group are in favor of a flat tax?

(1) Of the people in the group who support Candidate R, 58 percent are in favor of a flat tax.

(2) Of the people in the group who support Candidate T, 22 are in favor of a flat tax.

44.In the xy-plane, at what two points does the graph of y = (x + a)(x + b) intersect the x-axis?

(1) a + b = -1

(2) The graph intersects the y-axis at (0, -6).

45.How many of the 42 people in a group are employed students?

(1) 29 of the 42 people are employed.

(2) 24 of the 42 people are students.

46.How many hours did it take Helen to drive from her house to her parents' house?

(1) Helen's average speed on this trip was 72 kilometers per hour.

(2) If Helen's average speed on this trip had been 8 kilometers per hour greater, it would have taken her 1 hour less.

47.Is m ≠ n ?

(1) m + n < 0

(2) mn < 0

48.Last Friday each of the pets at a certain veterinary clinic was given either 1 treat or 2 treats. What was the total number of

Page 99: QUESTION BANK - edumentor.co.inedumentor.co.in/Downloads/2018/Extra Content... · of India's oldest and most important uranium mine, ... Punjab banned e-cigarettes last year and Maharashtra

treats given to pets at the clinic last Friday?

(1) The total number of pets at the clinic last Friday was 90.

(2) 32 of the pets at the clinic last Friday were given 2 treats each.

49.Does x + y = 5 ?

(1) 4x + y = 17

(2) x + 4y = 8

50.A construction company was paid a total of $500,000 for a construction project. The company's only costs for the project were for labor and materials. Was the company's profit for the project greater than $150,000 ?

(1) The company's total cost was three times its cost for materials.

(2) The company's profit was greater than its cost for labor.

Page 100: QUESTION BANK - edumentor.co.inedumentor.co.in/Downloads/2018/Extra Content... · of India's oldest and most important uranium mine, ... Punjab banned e-cigarettes last year and Maharashtra

IPM - IIM

Quantitative Aptitude

Page 101: QUESTION BANK - edumentor.co.inedumentor.co.in/Downloads/2018/Extra Content... · of India's oldest and most important uranium mine, ... Punjab banned e-cigarettes last year and Maharashtra

Number System

Q1. If a, b, c and d are prime numbers such that 1 < a < b < c < d and abcd = 1430, then what

is the value of d?

A) 3

B) 9

C) 11

D) 13

E) 22

Q2. What is the least positive integer that is divisible by each of the integers 1 through 7,

inclusive?

A) 420

B) 840

C) 1260

D) 2520

E) 5040

Q3. If x is equal to the sum of even integers from 40 to 60 inclusive and y is the number of

even integers from 40 to 60 inclusive. What is the value of x + y?

A) 550

B) 551

C) 560

D) 561

E) 572

Q4. If S is a set of four numbers w, x, y and z. Is the range of the numbers in S greater than

2?

1) w – z > 2

2) z is the least number in S

Q5. If a, b and c are integers, is the number 3(a + b) + c divisible by 3?

1) a + b is divisible by 3

2) c is divisible by 3

QA - IPM

Page 102: QUESTION BANK - edumentor.co.inedumentor.co.in/Downloads/2018/Extra Content... · of India's oldest and most important uranium mine, ... Punjab banned e-cigarettes last year and Maharashtra

Q6. Is x an even integer?

1) x is the square of an integer

2) x is the cube of an integer

Q7. If x and y are the integers between 10 and 99, inclusive. Is �–�� an integer?

1) x and y have the same two digits, but in reverse order.

2) The tens digit of x is 2 more than the units digit and the tens digit of y is 2 less than

the units digit.

Q8. How many distinct prime factors does 540 + 537 have?

A) 1

B) 2

C) 3

D) 4

E) 5

Q9. If r, s and t are consecutive integers, what is the greatest prime factor of 3r + 3s + 3t ?

A) 3

B) 5

C) 7

D) 11

E) 13

Q10. What is the sum of digits of the number 1050 – 74?

A) 9

B) 18

C) 27

D) 36

E) 440

QA - IPM

Page 103: QUESTION BANK - edumentor.co.inedumentor.co.in/Downloads/2018/Extra Content... · of India's oldest and most important uranium mine, ... Punjab banned e-cigarettes last year and Maharashtra

Q11. If x, y and z are positive integers such that ‘x’ is a factor of’ y’ and x is a multiple of ‘z’.

Which of the following is NOT necessarily an integer?

A) � ���

B) ����

C) � ���

D) ���

E) ���

Q12. Is the integer z divisible by 6?

1) The greatest common factor of z and 12 is 3.

2) The greatest common factor of z and 15 is 15.

Q13. If x and y are perfect squares, then which of the following is not necessarily a perfect

square?

A) x2

B) xy

C) 4x

D) x +y

E) x5

Q14. If p and q are positive integers, how many integers are larger than pq and smaller than

p(q + 2)?

A) 3

B) P + 2

C) p – 2

D) 2p – 1

E) 2p + 1

QA - IPM

Page 104: QUESTION BANK - edumentor.co.inedumentor.co.in/Downloads/2018/Extra Content... · of India's oldest and most important uranium mine, ... Punjab banned e-cigarettes last year and Maharashtra

Q15. If x > y > 0, which of the following must be true?

I. ������ >

��

II. ������ =

��

III. ������ > 1

A) I only

B) II only

C) III only

D) I and II only

E) II and III only

Q16. If 2 is the greatest number that will divide evenly into both x and y, what is the

greatest number that will divide evenly into both 5x and 5y?

A) 2

B) 4

C) 6

D) 8

E) 10

Q17. If p divided by 9 leaves a remainder of 1, which of the following must be true?

I. p is even

II. p is odd

III. p = 3z + 1, for some integer z

A) I only

B) II only

C) III only

D) I and II only

E) None of the above

Q18. If the sum of two prime numbers x and y is odd, then the product of x and y must be

divisible by

A) 2

B) 3

C) 4

D) 5

E) 8

QA - IPM

Page 105: QUESTION BANK - edumentor.co.inedumentor.co.in/Downloads/2018/Extra Content... · of India's oldest and most important uranium mine, ... Punjab banned e-cigarettes last year and Maharashtra

Q19. If ����–� = 3 and x and y are integers, then which one of the following must be true?

A) x is divisible by 4

B) y is odd

C) y is even

D) x is even

E) x is irreducible fraction

Q20. If y is an odd integer and the product of x and y equals 222, what is the value of x?

1) x is a prime number

2) y is a three digit number

Q21. If x and y are prime numbers such that x > y > 2, then x2 – y2 must be divisible by

which one of the following numbers?

A) 3

B) 4

C) 5

D) 9

E) 12

Q22. If the positive integer N is a perfect square, which of the following must be true?

I. Number of distinct factors of N is odd

II. Sum of distinct factors of N is odd

III. The number of distinct prime factors of N is even

A) I only

B) II only

C) I and II only

D) I and III only

E) I, II and III

Q23. In a certain game, a large bag is filled with blue, green, purple and red chips worth 1,

5, x and 11 points each, respectively. The purple chips are worth more than the green chips

but less than the red chips. A certain number of chips are then selected from the bag. If the

product of the point values of selected chips is 88,000, how many purple chips were

selected?

A) 1

B) 2

C) 3

D) 4

E) 5

QA - IPM

Page 106: QUESTION BANK - edumentor.co.inedumentor.co.in/Downloads/2018/Extra Content... · of India's oldest and most important uranium mine, ... Punjab banned e-cigarettes last year and Maharashtra

Q24. Is the positive integer x a perfect square?

1) The number of distinct factors of x is even

2) The sum of all distinct factors of x is even

Q25. Let a = sum of integers from 1 to 20 and b = sum of integers from 21 to 40. What is the

value of b – a ?

A) 21

B) 39

C) 200

D) 320

E) 400

Q26. Does x – y = 0?

1)�� > 0

2) x2 = y2

Q27. If p and q are integers, is pq + 1 even?

1) If p is divided by 2, the remainder is 1

2) If q is divided by 6, the remainder is 1

Q28. S is a set of integers such that

I. If x is in S, then –x is in S

II. If both x and y are in S, then so is x + y.

Is -2 in S?

1) 1 is in S

2) 0 is in S

Q29. A botanist select n2 trees on an island and studies (2n + 1) trees everyday where n is

an even integer. He does not study the same tree twice. Which of the following cannot be

the number of trees that he studies on the last day?

A) 13

B) 17

C) 28

D) 31

E) 79

QA - IPM

Page 107: QUESTION BANK - edumentor.co.inedumentor.co.in/Downloads/2018/Extra Content... · of India's oldest and most important uranium mine, ... Punjab banned e-cigarettes last year and Maharashtra

Q30. If x = 2891 × 2892 × 2893 ×……………..× 2898 × 2899 × 2900, the what is the remainder

when x is divided by 17?

A) 0

B) 4

C) 7

D) 10

E) 14

Q31. For every positive integer n, the function h(n) is defined to be the product of all the

even integers from 2 to n, inclusive. If p is the smallest factor of h(100) + 1, then p is

A) Between 2 and 10

B) Between 11 and 20

C) Between 21 and 30

D) Between 31 and 40

E) Greater than 40

Q32. If the sum of five consecutive positive integers is A, then the sum of the next five

consecutive integers in terms of A is

A) A + 1

B) A + 5

C) A + 25

D) 2A

E) 5A

Q33. A, B, C and D are all different digits between 0 and 9. If AB + DC = 7B (AB, DC and 7B

are two digit numbers), what is the value of C?

A) 0

B) 1

C) 2

D) 3

E) 5

Q34. If 2x < y and 2x2 > xy, which of the following must be true?

A) x > 1

B) x < 0

C) x > 0

D) y < 0

E) y > 1

QA - IPM

Page 108: QUESTION BANK - edumentor.co.inedumentor.co.in/Downloads/2018/Extra Content... · of India's oldest and most important uranium mine, ... Punjab banned e-cigarettes last year and Maharashtra

Q35. If x and y are different positive integers and 3x + y = 14, what is the product of all the

possible values of x?

A) 6

B) 8

C) 14

D) 20

E) 24

Q36. If x is an integer and �√x × x�– x = a, which of the following must be true?

I. a is even

II. a is positive

III. a is an integer

A) I only

B) II only

C) III only

D) I and II only

E) None of the above

Q37. Which of the following expressions has the greatest value? (numbers)(650-750)

A) ��������������

B) ��������������

C) ��������������

D) ��������������

E) ��������������

Q38. If ‘m’ and ‘n’ are integers then what is the smallest possible value of integer m such

that �� = 0.3636363636……..?

A) 3

B) 4

C) 7

D) 13

E) 22

QA - IPM

Page 109: QUESTION BANK - edumentor.co.inedumentor.co.in/Downloads/2018/Extra Content... · of India's oldest and most important uranium mine, ... Punjab banned e-cigarettes last year and Maharashtra

Q39. If N = 1234@ and @ represents the units digit. Is N a multiple of 5? (numbers)(600-

700)

1) @! is not divisible by 5

2) @ is divisible by 9

Q40. If (|p|!)p = |p|!, which of the following could be the value(s) of p?

A) -1

B) 0

C) 1

D) -1 and 1

E) -1, 0 and 1

Q41. If 5x = y +7, is (x – y) > 0?

1) xy = 6

2) x and y are consecutive integers with the same sign

Q42. If ‘s’ and ‘t’ are positive integer such that �� = 64.12, which of the following could be

the remainder when ‘s’ is divided by ‘t’?

A) 2

B) 4

C) 8

D) 20

E) 45

Q43. If ‘x’ is positive, which of the following could be the correct ordering of ��, 2x and x2?

I. x2 < 2x < ��

II. x2 < �� < 2x

III. 2x < x2 <��

A) None of the above

B) I only

C) III only

D) I and II only

E) II and III only

Q44. If p is a prime number greater than 2, what is the value of p?

1) There are total of 100 prime numbers between 1 and p + 1.

2) There are a total of ‘p’ prime numbers between 1 and 3912

QA - IPM

Page 110: QUESTION BANK - edumentor.co.inedumentor.co.in/Downloads/2018/Extra Content... · of India's oldest and most important uranium mine, ... Punjab banned e-cigarettes last year and Maharashtra

Q45. If a, b and c are positive integers and are assembled into the six digit number abcabc,

which of the following must be a factor of abcabc?

A) 16

B) 13

C) 5

D) 3

E) None of the above

Q46. What is the remainder when 11 + 22 + 33 +………………..+ 1010 is divided by 5?

A) 0

B) 1

C) 2

D) 3

E) 4

Q47. If Q and T are integers, what is the value of Q?

1) Q = �$�

2) $��� = �%&��'

Q48. If ‘m’ and ‘n’ are integers and √mn = 10, which of the following cannot be a value of

m + n?

A) 25

B) 29

C) 50

D) 52

E) 101

Q49. The three digits of a number add upto 11. The number is divisible by 5. The leftmost

digit is double the middle digit. What is the product of the three digits?

A) 40

B) 72

C) 78

D) 88

E) 125

Q50. If x = y + y2 and y is a negative integer, when y decreases in value, then x

A) Increases in value

B) Fluctuates

C) Decreases in value

D) Remains the same

E) Decreases in constant increments

QA - IPM

Page 111: QUESTION BANK - edumentor.co.inedumentor.co.in/Downloads/2018/Extra Content... · of India's oldest and most important uranium mine, ... Punjab banned e-cigarettes last year and Maharashtra

Exponents & Basic Algebra

1. lies between: 1 & 2 2 & 3 3 & 4 4 & 5 5 & 6

2. List the following in increasing order from left to right: ?

3. lies between: 4 & 5 5 & 6 6 & 7 7 & 8 8 & 9

4. 8a(1/4)b = ? (1) b = 1.5a (2) a = 2

5. A, B, C, D, E, F, G, and H are all integers, listed in order of increasing size. When these numbers are arrangedon a number line, the distance between any two consecutive numbers is constant. If G and H are equal to 512

and 513, respectively, what is the value of A?

6. (35x + 35x + 35x)(45x + 45x + 45x + 45x) =

7. The three-digit positive integer x has the hundreds, tens, and units digits of a, b, and c, respectively. Thethree-digit positive integer y has the hundreds, tens, and units digits of k, l, and m, respectively. If(2a)(3b)(5c) = 12(2k)(3l)(5m), what is the value of x – y?21 200 210 300 310

8. Is x > 1010 ? (1) x > 234 (2) x = 235

9. = ?

10. What is the value of 2a4b?(1) a = -2b (2) b = 4

11. If 274x + 2 × 162-2x × 36x × 96 – 2x = 1, then what is the value of x?-9 -6 3 6 9

12. If (22x+1)(32y-1) = 8x27y, then x + y =-3 -1 0 1 3

13. If (62)(44)(5x)(20) / (82)(9) = 1375, what is the value of x? -1 0 1 2 3

14. If 5x = y, what is x? (1) y2 = 625 (2) y3 = 15,625

15. Wendy, Jim, and Pedro are golfing. Collectively, they have 24 golf balls. How many golf balls does Jim have?(1) Jim has 1/3 of the number of golf balls that Wendy has.(2) Pedro has 1/2 of the total number of golf balls.

16. If x, y, and z are integers greater than 1, and (327)(3510)(z) = (58)(710)(914)(xy), then what is the value of x?(1) z is prime (2) x is prime

17. If 44x = 1600, what is the value of (4x–1)2?40 20 10 5/2 5/4

18. If x and y are integers and (15x + 15x+1) / 4y = 15y, what is the value of x?2 3 4 5 Cannot be determined

19. If 3m3m3m = 9n, then m/n =

-24(512) -23(512) -24(56) 23(512) 24(512)

125x+1 315x + 420x 255x 735x 255x+1

3

Page 112: QUESTION BANK - edumentor.co.inedumentor.co.in/Downloads/2018/Extra Content... · of India's oldest and most important uranium mine, ... Punjab banned e-cigarettes last year and Maharashtra

1/3 2/3 1 3/2 3

20. If a = 3b – 1, what is the value of a + b ? (1) 3b + 2 = 243 (2) a = 32b – 4

21. What is the value of ? -26 2√29 14 – 4√5 14 14 + 4√5

22. If x2/9 – 4/y2 = 12, what is the value of x?(1) x/3 + 2/y = 6 (2) x/3 – 2/y = 2

23. What is the value of (a + b)2 ? (1) a = 15/b (2) (a – b)2 = 4

24. If x and y are positive and x2y2 = 18 – 3xy, then x2 =18 – 3y / y3 18 / y2 18 / y2 + 3y 9 / y2 36 / y2

25. If y = √(3y + 4), then the product of all possible solutions for y is-4 -2 0 3 6

26. If the sum of the cubes of a and b is 8 and a6 – b6 = 14, what is the value of a3 – b3?¼ ½ 5/4 7/4 2

27. If x does not equal y, and xy does not equal 0, then when x is replaced by 1/x and y is replaced by1/y everywhere in the expression (x + y) / (x – y), the resulting expression is equivalent to- (x + y) / (x – y) (x - y) / (x + y) (x + y) / (x – y) (x + y) (x – y)

28. If x and y are non-zero integers, and 9x4 – 4y4 = 3x2 + 2y2, which of the following could be the value of x2 interms of y?

29. What is the ratio of r to s? (1) r + s = 7 (2) r2 – s2 = 7

30. If there are x men and y women in a choir, and there are z more men than there are women in that choir,what is z?(1) x² – 2xy + y² – 9 = 0 (2) x² + 2xy + y² – 225 = 0

31. The value of x is one quarter of z. The sum of x, y, and z is equal to 26. If the value of y is twice the value ofz, what is the largest factor of the sum of y and z?2 3 8 12 24

32. If 2 + 5a – b/2 = 3c, what is the value of b?(1) a + c = 13 (2) -12c = -20a + 4

33. If xy does not equal zero, what is the value of xy?(1) 2/x + 2/y = 3 (2) x3 – (2/y)3 = 0

34. The expression 3 / (2 + √3) is equal to:

35. If (1/5)m × (1/4)18=1/2 × (10)35, then m=?

36. If 521 × 411=2 × 10n, what is the value of n?11 21 22 23 32

37. Which of the following best approximates the value of q if 528+311=5q ?39 30 28 27 17

-4y2/3 -2y2 (2y2+1)/3 2y2 6y2/3

6 + 3√3 6 - 3√3 (6 + 3√3) / 7 (6 - 3√3) / 7 1.5 + √3

Page 113: QUESTION BANK - edumentor.co.inedumentor.co.in/Downloads/2018/Extra Content... · of India's oldest and most important uranium mine, ... Punjab banned e-cigarettes last year and Maharashtra

Permutations and Combinations

1. How many different anagrams can you make for the word KEAT? How many different anagrams can youmake for the word MATHEMATICS?

2. If there are 7 people and only 4 chairs in a room, how many different seating arrangements are possible?

3. A man wants to visit at least two of the four cities A, B, C and D. How many travel itineraries can he make?All cities are connected to one another.

4. There are 2 black balls, one red ball and one green ball, identical in shape and size. How many different lineararrangements can be generated by arranging these balls?

5. From a list of 10 songs, a DJ has to play either 2 or 3 songs. What is the total number of song sequences thathe can create?

6. A password contains at least 8 distinct digits. It takes 12 seconds to try one combination, what is theminimum amount of time required to guarantee access to the database?

7. Greg, Marcia, Peter, Jan, Bobby and Cindy go to a movie and sit next to each other in 6 adjacent seats in thefront row of the theater. If Marcia and Jan will not sit next to each other, in how many ways differentarrangements can the 6 people sit?

8. If a team of 4 people is to be chosen from 7 people in a room, how many different teams are possible?

9. In a college, 8 students play at the State level and 10 at the National level. If 6 students play at both Nationaland State levels, in how many ways can 9 students be selected from among these?

10. An engagement team consists of a project manager, team leader, and four consultants. There are 2candidates for the position of project manager, 3 candidates for the position of team leader, and 7 candidatesfor the 4 consultant slots. If 2 out of 7 consultants refuse to be on the same team, how many different teamsare possible?

11. In how many ways can 3 letters out of five distinct 5 distinct letters A, B, C, D and E be arranged in a straightline so that A and B never come together?

12. A nickel, a dime, and two identical quarters are arranged along a side of a table. If the quarters and the dimehave to face heads up, while the nickel can face either heads up or tails up, how many differentarrangements of coins are possible?

13. At a certain laboratory, chemical substances are identified by an unordered combination of 3 colors. If nochemical may be assigned the same colors, what is the maximum number of substances that can be identifiedusing 7 colors?

14. An equity analyst needs to select 3 stocks for the upcoming year and rank these securities in terms of theirinvestment potential. If the analyst has narrowed down the list of potential stocks to 7, in how many wayscan she choose and rank her top 3 picks?

15. How many different five-letter combinations can be created from the word TWIST?

16. If an employee ID code must consist of 3 non-repeating digits and each digit in the code must be a primenumber, how many ID codes can be created?

17. A university cafeteria offers 4 flavors of pizza – pepperoni, chicken, Hawaiian and vegetarian. If a customerhas an option to add, extra cheese, mushrooms, or both to any kind of pizza, how many different pizzavarieties are available?

18. Mario's Pizza has two choices of crust: deep dish and thin-and-crispy. The restaurant also has a choice of 5toppings: tomatoes, sausage, peppers, onions, and pepperoni. Finally, Mario's offers every pizza in extra-cheese as well as ‘regular’. If Linda's volleyball team decides to order a pizza with four toppings, how manydifferent choices do the teammates have at Mario's Pizza?

19. A book store has received 8 different books, of which 3/8 are novels, 25% are study guides and theremaining are textbooks. If all books must be placed on one shelf displaying new items and if books in thesame category have to be shelved next to each other, how many different arrangements of books arepossible?

QA - IPM

Page 114: QUESTION BANK - edumentor.co.inedumentor.co.in/Downloads/2018/Extra Content... · of India's oldest and most important uranium mine, ... Punjab banned e-cigarettes last year and Maharashtra

20. A group of 5 students bought movie tickets in one row next to each other. If Bob and Lisa are in this group,what is the number of ways of seating if both of them will sit next to only one other student from the group?

21. Mark’s clothing store uses a bar-code system to identify every item. Each item is marked by a combination of2 letters followed by 3 digits. Additionally, the three-digit number must be even for male products and odd forfemale products. If all apparel products start with the letter combination AP, how many male apparel itemscan be identified with the bar code?

22. Fernando purchased a university meal plan that allows him to have a total of 3 lunches and 3 dinners perweek. If the cafeteria is closed on weekends and Fernando always goes home for a dinner on Friday nights,how many options does he have to allocate his meals?

23. If the President and the Vice President must sit next to each other in a row with 4 other members on theBoard, how many different seating arrangements are possible?

24. To apply for the position of photographer at a local magazine, Veronica needs to include 3 or 4 of her picturesin an envelope accompanying her application. If she has pre-selected 5 photos representative of her work,how many choices does she have to provide the photos for the magazine?

25. A retail company needs to set up 5 additional distribution centers that can be located in three cities on theeast coast (Boston, New York, and Washington D.C.), one city in the mid-west (Chicago), and three cities onthe west coast (Seattle, San Francisco and Los Angeles). If the company must have 2 distribution centers oneach coast and 1 in the mid-west, and only one center can be added in each city, in how many ways can themanagement allocate the distribution centers?

26. Three couples need to be arranged in a row for a group photo. If the couples cannot be separated, how manydifferent arrangements are possible?

27. If 6 fair coins are tossed, how many different coin sequences will have exactly 3 tails, if all tails have to occurin a row?

28. A telephone company needs to create a set of 3-digit area codes. The company is entitled to use only digits 2,4 and 5, which can be repeated. If the product of the digits in the area code must be even, how manydifferent codes can be created?

29. Jake, Lena, Fred, John and Inna need to drive home from a corporate reception in an SUV that can seat 7people. If only Inna or Jake can drive, how many seat allocations are possible?

30. In how many ways can a teacher write an answer key for a mini-quiz that contains 3 true-false questionsfollowed by 2 multiples-choice questions with 4 answer choices each, if the correct answers to all true-falsequestions cannot be the same?

31. A student committee on academic integrity has 90 ways to select a president and vice-president from a groupof candidates. The same person cannot be both president and vice-president. How many students are in thegroup?

32. A pod of 6 dolphins always swims single file, with 3 females at the front and 3 males in the rear. In howmany different arrangements can the dolphins swim?

33. A British spy is trying to escape from his prison cell. The lock requires him to enter one number, from 1-9,and then push a pair of colored buttons simultaneously. He can make one attempt every 3 seconds. If thereare 6 colored buttons, what is the longest possible time it could take the spy to escape from the prison cell?

34. Every morning, Casey walks from her house to the bus stop. She always travels exactly nine blocks from herhouse to the bus, but she varies the route she takes every day. (One sample route is shown.) How many dayscan Casey walk from her house to the bus stop without repeating the same route?

35. Three dwarves and three elves sit down in a row of six chairs. If no dwarf will sit next to another dwarf andno elf will sit next to another elf, in how many different ways can the elves and dwarves sit?

QA - IPM

Page 115: QUESTION BANK - edumentor.co.inedumentor.co.in/Downloads/2018/Extra Content... · of India's oldest and most important uranium mine, ... Punjab banned e-cigarettes last year and Maharashtra

36. Gordon buys 5 dolls for his 5 nieces. The gifts include two identical Sun-and-Fun beach dolls, one ElegantEddie dress-up doll, one G.I. Josie army doll, and one Tulip Troll doll. If the youngest niece doesn't want theG.I. Josie doll, in how many different ways can he give the gifts?

37. How many different 5-person teams can be formed from a group of x individuals?

(1) If there had been x + 2 individuals in the group, exactly 126 different 5-person teams could have been formed.

(2) If there had been x + 1 individuals in the group, exactly 56 different 3-person teams could have been formed.

38. There are x people and y chairs in a room where x and y are positive prime numbers. How many ways canthe x people be seated in the y chairs (assuming that each chair can seat exactly one person)?

(1) x + y = 12 (2) There are more chairs than people.

39. A gambler began playing blackjack with $110 in chips. After exactly 12 hands, he left the table with $320 inchips, having won some hands and lost others. Each win earned $100 and each loss cost $10. How manypossible outcomes were there for the first 5 hands he played? (For example, won the first hand, lost thesecond, etc.)

(A) 10 (B) 18 (C) 26 (D) 32 (E) 64

40. In a 4 person race, medals are awarded to the fastest 3 runners. The first-place runner receives a gold medal,the second-place runner receives a silver medal, and the third-place runner receives a bronze medal. In theevent of a tie, the tied runners receive the same color medal. (For example, if there is a two-way tie for first-place, the top two runners receive gold medals, the next-fastest runner receives a silver medal, and nobronze medal is awarded). Assuming that exactly three medals are awarded, and that the three medalwinners stand together with their medals to form a victory circle, how many different victory circles arepossible?

(A) 24 (B) 52 (C) 96 (D) 144 (E) 648

41. The organizers of a week-long fair have hired exactly five security guards to patrol the fairgrounds at night forthe duration of the event. Exactly two guards are assigned to patrol the grounds every night, with no guardassigned consecutive nights. If the fair begins on a Monday, how many different pairs of guards will beavailable to patrol the fairgrounds on the following Saturday night?

(A) 9 (B) 7 (C) 5 (D) 3 (E) 2

42. Larry, Michael, and Doug have five donuts to share. If any one of the men can be given any whole number ofdonuts from 0 to 5, in how many different ways can the donuts be distributed?

(A) 21 (B) 42 (C) 120 (D) 504 (E) 5040

43. A woman has seven cookies—four chocolate chip and three oatmeal. She gives one cookie to each of her sixchildren: Nicole, Ronit, Kim, Deborah, Mark, and Terrance. If Deborah will only eat the kind of cookie that Kimeats, in how many different ways can the cookies be distributed? (The leftover cookie will be given to thedog.)

(A) 5040 (B) 50 (C) 25 (D) 15 (E) 12

44. Sammy has x flavors of candies with which to make goody bags for Frank's birthday party. Sammy tosses outy flavors, because he doesn't like them. How many different 10-flavor bags can Sammy make from theremaining flavors? (It doesn't matter how many candies are in a bag, only how many flavors).

(1) If Sammy had thrown away 2 additional flavors of candy, he could have made exactly 3,003 different 10-flavor bags.

(2) x = y + 17

45. How many different combinations of outcomes can you make by rolling three standard (6-sided) dice if theorder of the dice does not matter?

(A) 24 (B) 30 (C) 56 (D) 120 (E) 216

46. A certain league has four divisions. The respective divisions had 9, 10, 11, and 12 teams qualify for theplayoffs. Each division held its own double-elimination tournament -- where a team is eliminated from thetournament upon losing two games -- in order to determine its champion. The four division champions thenplayed in a single-elimination tournament -- where a team is eliminated upon losing one game -- in order to

QA - IPM

Page 116: QUESTION BANK - edumentor.co.inedumentor.co.in/Downloads/2018/Extra Content... · of India's oldest and most important uranium mine, ... Punjab banned e-cigarettes last year and Maharashtra

determine the overall league champion. Assuming that there were no ties and no forfeits, what is the maximum number of games that could have been played in order to determine the overall league champion?

(A) 79 (B) 83 (C) 85 (D) 87 (E) 88

47. You have a bag of 9 letters: 3 Xs, 3 Ys, and 3 Zs. You are given a box divided into 3 rows and 3 columns for atotal of 9 areas. How many different ways can you place one letter into each area such that there are no rowsor columns with 2 or more of the same letter? (Note: One such way is shown below.)

X Y Z Y Z X Z X Y

(A) 5 (B) 6 (C) 9 (D) 12 (E) 18

48. Eight women of eight different heights are to pose for a photo in two rows of four. Each woman in the secondrow must stand directly behind a shorter woman in the first row. In addition, all of the women in each rowmust be arranged in order of increasing height from left to right. Assuming that these restrictions are fullyadhered to, in how many different ways can the women pose?

(A) 2 (B) 14 (C) 15 (D) 16 (E) 18

49. Company X has 6 regional offices. Each regional office must recommend two candidates, one male and onefemale, to serve on the corporate auditing committee. If each of the offices must be represented by exactlyone member on the auditing committee and if the committee must consist of an equal number of male andfemale employees, how many different committees can be formed?

50. You have a six-sided cube and six cans of paint, each a different color. You may not mix colors of paint. Howmany distinct ways can you paint the cube using a different color for each side? (If you can reorient a cube tolook like another cube, then the two cubes are not distinct.)

(A) 24 (B) 30 (C) 48 (D) 60 (E) 120

51. A group of four women and three men have tickets for seven adjacent seats in one row of a theatre. If thethree men will not sit in three adjacent seats, how many possible different seating arrangements are there forthese 7 theatre-goers?

(A) 7! – 2!3!2! (B) 7! – 4!3! (C) 7! – 5!3! (D) 7 × 2!3!2! (E) 2!3!2!

52. Anthony and Michael sit on the six-member board of directors for company X. If the board is to be split upinto 2 three-person subcommittees, what percent of all the possible subcommittees that include Michael alsoinclude Anthony?20% 30% 40% 50% 60%

53. A family consisting of one mother, one father, two daughters and a son is taking a road trip in a sedan. Thesedan has two front seats and three back seats. If one of the parents must drive and the two daughtersrefuse to sit next to each other, how many possible seating arrangements are there?28 32 48 60 120

54. Six mobsters have arrived at the theater for the premiere of the film “Goodbuddies.” One of the mobsters,Frankie, is an informer, and he's afraid that another member of his crew, Joey, is on to him. Frankie, wantingto keep Joey in his sights, insists upon standing behind Joey in line at the concession stand. How many wayscan the six arrange themselves in line such that Frankie’s requirement is satisfied?6 24 120 360 720

55. A college admissions committee will grant a certain number of $10,000 scholarships, $5,000 scholarships, and$1,000 scholarships. If no student can receive more than one scholarship, how many different ways can thecommittee dole out the scholarships among the pool of 10 applicants?

(1) In total, six scholarships will be granted. (2) An equal number of scholarships will be granted at each scholarship level.

56. A certain panel is to be composed of exactly three women and exactly two men, chosen from x women and y men. How many different panels can be formed with these constraints?(1) If two more women were available for selection, exactly 56 different groups of three women could be

selected. (2) x = y + 1

QA - IPM

Page 117: QUESTION BANK - edumentor.co.inedumentor.co.in/Downloads/2018/Extra Content... · of India's oldest and most important uranium mine, ... Punjab banned e-cigarettes last year and Maharashtra

57. A student committee that must consist of 5 members is to be formed from a pool of 8 candidates. How manydifferent committees are possible?5 8 40 56 336

58. How many ways are there to award a gold, silver and bronze medal to 10 contending teams?10 × 9 × 8 10! / 3! 7! 10! / 3! 360 300

59. From a drawer containing black, blue and gray solid-color socks, including at least three socks of each color,how many matched pairs can be removed?(1) The drawer contains 11 socks.(2) The drawer contains an equal number of black and gray socks.

60. On Tuesday, Kramer purchases exactly 3 new shirts, 2 new sweaters, and 4 new hats, On the following dayand each subsequent day thereafter, Kramer wears one of his new shirts together with one of his newsweaters and one of his new hats. Kramer avoids wearing the exact same combination of shirt, sweater, andhat for as long as possible. On which day is this no longer possible?Tuesday Wednesday Thursday Friday Saturday

61. A certain stock exchange designates each stock with a one- , two-, or three-letter code, where each letter isselected from the 26 letters of the alphabet. If the letter may be repeated and if the same letters used in adifferent order constitute a different code, how many different stocks is it possible to uniquely designate withthese codes?2951 8125 15600 15302 18278

62. A certain law firm consists of 4 senior partners and 6 junior partners. How many different groups of 3partners can be formed in which at least one member of the group is a senior partner? (Two groups areconsidered different if at least one group member is different.)48 100 120 288 600

63. A company plans to assign identification numbers to its employees. Each number is to consist of four differentdigits from 0 to 9, inclusive, except that the first digit cannot be 0. How many different identification numbersare possible?

64. Pat will walk from intersection X to intersection y along route that is confined to the square grid of fourstreets and three avenues shown in the map above. How many routes from X to Y can Pat take that have theminimum possible length?

6 8 10 14 16

65. Tanya prepared four different letters to be sent to four different addresses. For each letter, she prepared anenvelope with its correct address. If the 4 letters are to be put into the envelopes at random, what is theprobability that only one letter will be put into the envelope with its correct address?

66. 5 people are to be seated around a circular table. Two seating arrangement are considered different onlywhen the positions of the people are different relative to each other. What is the total number of differentpossible seating arrangements for the group?

QA - IPM

Page 118: QUESTION BANK - edumentor.co.inedumentor.co.in/Downloads/2018/Extra Content... · of India's oldest and most important uranium mine, ... Punjab banned e-cigarettes last year and Maharashtra

Probability

67. A fair coin is flipped three times. What is the probability that the coin lands on heads exactly twice?(A) 1/8 (B) 3/8 (C) 1/2 (D) 5/8 (E) 7/8

68. Is the probability that Patty will answer all of the questions on her chemistry exam correctly greater than50%?(1) For each question on the chemistry exam, Patty has a 90% chance of answering the question

correctly. (2) There are fewer than 10 questions on Patty's chemistry exam.

69. There are 10 women and 3 men in room A. One person is picked at random from room A and moved to roomB, where there are already 3 women and 5 men. If a single person is then to be picked from room B, what isthe probability that a woman will be picked?(A) 13/21 (B) 49/117 (C) 15/52 (D) 5/18 (E) 40/117

70. If the probability of rain on any given day in Chicago during the summer is 50%, independent of whathappens on any other day, what is the probability of having exactly 3 rainy days from July 4 through July 8,inclusive?(A) 1/32 (B) 2/25 (C) 5/16 (D) 8/25 (E) ¾

71. In a shipment of 20 cars, 3 are found to be defective. If four cars are selected at random, what is theprobability that exactly one of the four will be defective?(A) 170/1615 (B) 3/20 (C) 8/19 (D) 3/5 (E) 4/5

72. A certain bag of gemstones is composed of two-thirds diamonds and one-third rubies. If the probability ofrandomly selecting two diamonds from the bag, without replacement, is 5/12, what is the probability ofselecting two rubies from the bag, without replacement?(A) 5/36 (B) 5/24 (C) 1/12 (D) 1/6 (E) ¼

73. Triplets Adam, Bruce, and Charlie enter a triathlon. If there are 9 competitors in the triathlon and medals areawarded for first, second, and third place, what is the probability that at least two of the triplets will win amedal?(A) 3/14 (B) 19/84 (C) 11/42 (D) 15/28 (E) ¾

74. Set S is the set of all prime integers between 0 and 20. If three numbers are chosen randomly from set S andeach number can be chosen only once, what is the positive difference between the probability that theproduct of these three numbers is a number less than 31 and the probability that the sum of these threenumbers is odd?(A) 1/336 (B) ½ (C) 17/28 (D) ¾ (E) 301/336

75. A random 10-letter code is to be formed using the letters A, B, C, D, E, F, G, H, I and I (only the “I” will beused twice). What is the probability that a code that has the two I’s adjacent to one another will be formed?(A) 1/10 (B) 1/8 (C) 1/5 (D) ¼ (E) ½

76. If p2 – 13p + 40 = q, and p is a positive integer between 1 and 10, inclusive, what is the probability that q <0?(A) 1/10 (B) 1/5 (C) 2/5 (D) 3/5 (E) 3/10

77. A box contains three pairs of blue gloves and two pairs of green gloves. Each pair consists of a left-handglove and a right-hand glove. Each of the gloves is separate from its mate and thoroughly mixed togetherwith the others in the box. If three gloves are randomly selected from the box, what is the probability that amatched set (i.e., a left- and right-hand glove of the same color) will be among the three gloves selected?(A) 3/10 (B) 23/60 (C) 7/12 (D) 41/60 (E) 5/6

78. A football team has 99 players. Each player has a uniform number from 1 to 99 and no two players share thesame number. When football practice ends, all the players run off the field one-by-one in a completely

QA - IPM

Page 119: QUESTION BANK - edumentor.co.inedumentor.co.in/Downloads/2018/Extra Content... · of India's oldest and most important uranium mine, ... Punjab banned e-cigarettes last year and Maharashtra

random manner. What is the probability that the first four players off the field will leave in order of increasing uniform numbers (e.g., #2, then #6, then #67, then #72, etc)? (A) 1/64 (B) 1/48 (C) 1/36 (D) 1/24 (E) 1/16

79. What is the probability thatw

v

u

and z

y

x

are reciprocal fractions?

(1) v, w, y, and z are each randomly chosen from the first 100 positive integers. (2) The product (u)(x) is the median of 100 consecutive integers.

80. Bill and Jane play a simple game involving two fair dice, each of which has six sides numbered from 1 to 6(with an equal chance of landing on any side). Bill rolls the dice and his score is the total of the two dice. Janethen rolls the dice and her score is the total of her two dice. If Jane’s score is higher than Bill’s, she wins thegame. What is the probability the Jane will win the game?(A) 15/36 (B) 175/432 (C) 575/1296 (D) 583/1296 (E) ½

81. Kate and Danny each have $10. Together, they flip a fair coin 5 times. Every time the coin lands on heads,Kate gives Danny $1. Every time the coin lands on tails, Danny gives Kate $1. After the five coin flips, what isthe probability that Kate has more than $10 but less than $15?(A) 5/16 (B) ½ (C) 12/30 (D) 15/32 (E) 3/8

82. There is a 10% chance that it won’t snow all winter long. There is a 20% chance that schools will not beclosed all winter long. What is the greatest possible probability that it will snow and schools will be closedduring the winter?(A) 55% (B) 60% (C) 70% (D) 72% (E) 80%

83. There are y different travelers who each have a choice of vacationing at one of n different destinations. Whatis the probability that all y travelers will end up vacationing at the same destination?(A) 1/n! (B) n/n! (C) 1/ny (D) 1/ny -1 (E) n/yn

84. A small, experimental plane has three engines, one of which is redundant. That is, as long as two of theengines are working, the plane will stay in the air. Over the course of a typical flight, there is a 1/3 chancethat engine one will fail. There is a 75% probability that engine two will work. The third engine works onlyhalf the time. What is the probability that the plane will crash in any given flight?(A) 7/12 (B) ¼ (C) ½ (D) 7/24 (E) 17/24

85. In the game of Funball, each batter can either hit a home run, hit a single, or strikeout, and the likelihood ofeach outcome is completely determined by the opposing pitcher. A Funball batter scores a point for theirteam by advancing sequentially through each of four "bases", according to the following rules:Home run: The batter and any players already on a base advance through all four bases.Single: The batter advances to first base, and any players already on a base advance one base each.Strikeout: No one advances any bases, and the batter loses his/her turn.If the batting team has a runner on first base, which pitcher (Roger or Greg) is more likely to allow a pointbefore recording a strikeout?(1) Greg is twice as likely as Roger to allow a single, and four times as likely as Roger to record a

strikeout. (2) Greg is twice as likely as Roger to allow a single, and one fourth as likely as Roger to allow a home

run.

86. Ms. Barton has four children. You are told correctly that she has at least two girls but you are not told whichtwo of her four children are those girls. What is the probability that she also has two boys? (Assume that theprobability of having a boy is the same as the probability of having a girl.)(A) ¼ (B) 3/8 (C) 5/11 (D) ½ (E) 6/11

87. Mike recently won a contest in which he will have the opportunity to shoot free throws in order to win$10,000. In order to win the money Mike can either shoot 1 free throw and make it, or shoot 3 free throwsand make at least 2 of them. Mike occasionally makes shots and occasionally misses shots. He knows that hisprobability of making a single free throw is p, and that this probability doesn't change. Would Mike have abetter chance of winning if he chose to attempt 3 free throws?(1) p < 0.7 (2) p > 0.6

QA - IPM

Page 120: QUESTION BANK - edumentor.co.inedumentor.co.in/Downloads/2018/Extra Content... · of India's oldest and most important uranium mine, ... Punjab banned e-cigarettes last year and Maharashtra

88. Laura has a deck of standard playing cards with 13 of the 52 cards designated as a "heart." If Laura shufflesthe deck thoroughly and then deals 10 cards off the top of the deck, what is the probability that the 10thcard dealt is a heart?(A) ¼ (B) 1/5 (C) 5/26 (D) 12/42 (E) 13/42

89. A license plate in the country Kerrania consists of four digits followed by two letters. The letters A, B, and Care used only by government vehicles while the letters D through Z are used by non-government vehicles.Kerrania's intelligence agency has recently captured a message from the country Gonzalia indicating that anelectronic transmitter has been installed in a Kerrania government vehicle with a license plate starting with79. If it takes the police 10 minutes to inspect each vehicle, what is the probability that the police will find thetransmitter within three hours? (A) 18/79 (B) 1/6 (C) 1/25 (D) 1/50 (E) 1/900

90. A grid of light bulbs measures x bulbs by x bulbs, where x > 2. If 4 light bulbs are illuminated at random,what is the probability, in terms of x, that the 4 bulbs form a 2 bulb by 2 square?

91. Baseball's World Series matches 2 teams against each other in a best-of-seven series. The first team to winfour games wins the series and no subsequent games are played. If you have no special information abouteither of the teams, what is the probability that the World Series will consist of fewer than 7 games?(A) 12.5% (B) 25% (C) 31.25% (D) 68.75% (E) 75%

92. Harriet and Tran each have $10. Together, they flip a fair coin 5 times. Every time the coin lands on heads,Tran gives Harriet $1. Every time the coin lands on tails, Harriet gives Tran $1. After the five coin flips, whatis the probability that Harriet has more than $10 but less than $15?(A) 5/16 (B) 1/2 (C) 12/30 (D) 15/32 (E) 3/8

93. If 40 percent of all students at College X have brown hair and 70 percent of all students at College X haveblue eyes, what is the difference between the minimum and the maximum probability of picking a studentfrom College X who has neither brown hair nor blue eyes?(A) 0.2 (B) 0.3 (C) 0.4 (D) 0.6 (E) 0.7

94. In a room filled with 7 people, 4 people have exactly 1 friend in the room and 3 people have exactly 2 friendsin the room (Assuming that friendship is a mutual relationship, i.e. if John is Peter's friend, Peter is John'sfriend). If two individuals are selected from the room at random, what is the probability that those twoindividuals are NOT friends?5/21 3/7 4/7 5/7 16/21

95. Bill has a small deck of 12 playing cards made up of only 2 suits of 6 cards each. Each of the 6 cards within asuit has a different value from 1 to 6; thus, there are 2 cards in the deck that have the same value. Bill likesto play a game in which he shuffles the deck, turns over 4 cards, and looks for pairs of cards that have thesame value. What is the chance that Bill finds at least one pair of cards that have the same value?8/33 62/165 17/33 103/165 25/33

96. If a jury of 12 people is to be selected randomly from a pool of 15 potential jurors, and the jury pool consistsof 2/3 men and 1/3 women, what is the probability that the jury will comprise at least 2/3 men?24/91 45/91 2/3 67/91 84/91

97. John and Peter are among the nine players a basketball coach can choose from to field a five-player team. Ifall five players are chosen at random, what is the probability of choosing a team that includes John andPeter?1/9 1/6 2/9 5/18 1/3

98. A small company employs 3 men and 5 women. If a team of 4 employees is to be randomly selected toorganize the company retreat, what is the probability that the team will have exactly 2 women?1/14 1/7 2/7 3/7 ½

99. A box contains one dozen donuts. Four of the donuts are chocolate, four are glazed, and four are jelly. If twodonuts are randomly selected from the box, one after the other, what is the probability that both will be jellydonuts?1/11 1/9 1/3 2/3 8/9

QA - IPM

Page 121: QUESTION BANK - edumentor.co.inedumentor.co.in/Downloads/2018/Extra Content... · of India's oldest and most important uranium mine, ... Punjab banned e-cigarettes last year and Maharashtra

Geometry Part 1: Lines and Angles

1. What is the value of a + b?

30 50 55 65 90

2. If l1 is parallel to l2, what is x + 2y?

90 120 180 270 360

3. What is the value of x?

(1) l1 is parallel to l2 (2) y = 70

4. What is the value of a + b + c + d + e + f?

180 270 300 360 720

QA - IPM

Page 122: QUESTION BANK - edumentor.co.inedumentor.co.in/Downloads/2018/Extra Content... · of India's oldest and most important uranium mine, ... Punjab banned e-cigarettes last year and Maharashtra

5. If x – q = s – y, what is the value of z?

1) xq + sy + sx + yq = zr 2) zq – ry = rx – zs

6. In the figure, point D divides side BC of triangle ABC into segments BD and DC of lengths 1 and 2 unitsrespectively. Given that ∠ADC = 60º and ∠ABD = 45º, what is the measure of angle x in degrees? (Note:Figure is not drawn to scale.)

(A) 55 (B) 60 (C) 70 (D) 75 (E) 90

7. What is the degree measure of angle a?

(1) b + c = 287 degrees (2) d + e = 269 degrees

Topic 2: Triangles

1. In triangle ABC, if BC = 3 and AC = 4, then what is the length of segment CD?

3 15/4 5 16/3 20/3

QA - IPM

Page 123: QUESTION BANK - edumentor.co.inedumentor.co.in/Downloads/2018/Extra Content... · of India's oldest and most important uranium mine, ... Punjab banned e-cigarettes last year and Maharashtra

2. The figure is comprised of three squares and a triangle. If the areas marked X, Y and Z are 25, 144, and 169,respectively, what is the area of the triangle marked T?

25 30 50 60 97

3. If angle BAD is a right angle, what is the length of side BD?

(1) AC is perpendicular to BD (2) BC = CD

4. What is the length of segment BC?

(1) Angle ABC is 90 degrees. (2) The area of the triangle is 30.

5. What is the perimeter of isosceles triangle ABC?(1) The length of side AB is 9 (2) The length of side BC is 4

6. The figure is made up of a series of inscribed equilateral triangles. If the pattern continues until the length ofa side of the largest triangle (i.e. the entire figure) is exactly 128 times that of the smallest triangle, whatfraction of the total figure will be shaded?

QA - IPM

Page 124: QUESTION BANK - edumentor.co.inedumentor.co.in/Downloads/2018/Extra Content... · of India's oldest and most important uranium mine, ... Punjab banned e-cigarettes last year and Maharashtra

7. Given that ABCD is a rectangle, is the area of triangle ABE > 25?(Note: Figure above is not drawn to scale).

(1) AB = 6 (2) AE = 10

8. In the figure, AC = 3, CE = x, and BC is parallel to DE. If the area of

triangle ABC is 1/12 of the area of triangle ADE, then x = ?

9. Triangle A has one side of length x. If √(x8) = 81, what is the perimeter of Triangle A?1) Triangle A has sides whose lengths are consecutive integers2) Triangle A is NOT a right triangle

10. If AD is 6√3, and ADC is a right angle, what is the area of triangular region ABC?

(1) Angle ABD = 60° (2) AC = 12

QA - IPM

Page 125: QUESTION BANK - edumentor.co.inedumentor.co.in/Downloads/2018/Extra Content... · of India's oldest and most important uranium mine, ... Punjab banned e-cigarettes last year and Maharashtra

11. If BE || CD, and BC = AB = 3, AE = 4 and CD = 10, what is the area of trapezoid BEDC?

12 18 24 30 48

12. If the length of side AB is 17, is triangle ABC a right triangle?(1) The length of side BC is 144. (2) The length of side AC is 145.

13. In the figure, if point C is the center of the circle and DB = 7, what is the length of DE?

(1) x = 60° (2) DE || CA

14. The area of the right triangle ABC is 4 times greater than the area of the right triangle KLM. If the hypotenuseKL is 10 inches, what is the length of the hypotenuse AB?

(1) Angles ABC and KLM are each equal to 55 degrees. (2) LM is 6 inches.

15. In the diagram, triangle PQR has a right angle at Q and a perimeter of 60. Line segment QS is perpendicularto PR and has a length of 12. PQ > QR. What is the ratio of the area of triangle PQS to the area of triangleRQS?

3/2 7/4 15/8 16/9 2

QA - IPM

Page 126: QUESTION BANK - edumentor.co.inedumentor.co.in/Downloads/2018/Extra Content... · of India's oldest and most important uranium mine, ... Punjab banned e-cigarettes last year and Maharashtra

Circles

1. If P, Q and R are the centers of circles P, Q, and R and the points P, Q, R and T all lie on the same line, whatportion of circle P is shaded?

2. If 1/a2 + a2 represents the diameter of circle O and 1/a + a = 3, which of the following best approximates thecircumference of circle O?28 22 20 16 12

3. A car is being driven on a road. Assuming that the car's wheels turn without slipping, how many full 360°rotations does each tire on the car make in 10 minutes?(1) The car is traveling at 50 miles per hour.(2) Each tire has a radius of 20 inches.

4. Two circular road signs are to be painted yellow. If the radius of the larger sign is twice that of the smallersign, how many times more paint is needed to paint the larger sign (assuming that a given amount of paintcovers the same area on both signs)?2 3 π 4 3π/2

5. The figure represents five concentric quarter-circles. The length of the radius of the largest quarter-circle is x.The length of the radius of each successively smaller quarter-circle is one less than that of the next largerquarter-circle. What is the combined area of the shaded regions (black), in terms of x?

6. In the diagram (not drawn to scale), Sector PQ is a quarter-circle. The distance from A to P is half thedistance from P to B. The distance from C to Q is 2/7 of the distance from Q to B. If the length of AC is 100,what is the length of the radius of the circle with center B?

QA - IPM

Page 127: QUESTION BANK - edumentor.co.inedumentor.co.in/Downloads/2018/Extra Content... · of India's oldest and most important uranium mine, ... Punjab banned e-cigarettes last year and Maharashtra

7. A circular gear with a diameter of 24 centimeters is mounted directly on another circular gear with a diameterof 96 centimeters. Both gears turn on the same axle at their exact centers and each gear has a single notch,at the 12 o'clock position. At the same moment, the gears begin to turn at the same rate, with the larger gearmoving clockwise and the smaller gear counterclockwise. How far, in centimeters, will the notch on the largergear have traveled the second time the notches pass each other?

8. In the diagram, points A, B, and C are on the diameter of the circle with center B. Additionally, all arcspictured are semicircles. Suppose angle YXA = 105 degrees. What is the ratio of the area of the shadedregion above the line YB to the area of the shaded region below the line YB? (Note: Diagram is not drawn toscale and angles drawn are not accurate.)

(A) ¾ (B) 5/6 (C) 1 (D) 7/5 (E) 9/7

9. For a circle with center point P, cord XY is the perpendicular bisector of radius AP (A is a point on the edge ofthe circle). What is the length of cord XY?(1) The circumference of circle P is twice the area of circle P. (2) The length of Arc XAY = 2π/3.

10. ABCD is a square inscribed in a circle and arc ADC has a length of π√x. If a dart is thrown and landssomewhere in the circle, what is the probability that it will not fall within the inscribed square? (Assume thatthe point in the circle where the dart lands is completely random.)

11. Figure ABCD is a square with sides of length x. Arcs AB, AD, BC, and DC are all semicircles. What is the areaof the black region, in terms of x?

QA - IPM

Page 128: QUESTION BANK - edumentor.co.inedumentor.co.in/Downloads/2018/Extra Content... · of India's oldest and most important uranium mine, ... Punjab banned e-cigarettes last year and Maharashtra

12. In the figure, a small circle with radius r intersects a larger circle with radius R (where R > r). If k > 0, whatis the difference in the areas of the non-overlapping parts of the two circles?

(1) R = r + 3k (2) (kR) / (kr – 6) = -1

13. If CD is the diameter of the circle, does x equal 30?

(1) The length of CD is twice the length of BD. (2) y = 60

14. Two circles share a center at point C, as shown. Segment AC is broken up into two shorter segments, AB andBC, with dimensions shown. What is the ratio of the area of the large circle to the area of the small circle?

25/4 5/2 3/2 2/5 4/25

15. The length of minor arc AB is twice the length of minor arc BC and the length of minor arc AC is three timesthe length of minor arc AB. What is the measure of angle BCA?

20 40 60 80 120

QA - IPM

Page 129: QUESTION BANK - edumentor.co.inedumentor.co.in/Downloads/2018/Extra Content... · of India's oldest and most important uranium mine, ... Punjab banned e-cigarettes last year and Maharashtra

16. What is the radius of the circle shown?

(1) The measure of arc PQ is 4 . (2) The center of the circle is at point O.

17. A cylindrical tank has a base with a circumference of meters and an equilateral triangle painted onthe interior side of the base. A grain of sand is dropped into the tank, and has an equal probability of landingon any particular point on the base. If the probability of the grain of sand landing on the portion of the baseoutside the triangle is 3/4, what is the length of a side of the triangle?

18. In the figure, circle O has center O, diameter AB and a radius of 5. Line CD is parallel to the diameter. What isthe perimeter of the shaded region?

19. The figure shows the top side of a circular medallion made of a circular piece of colored glass surrounded bya metal frame, represented by the shaded region.

If the radius of the medallion is r centimeter and width of the metal frame is s centimeter, then, in terms of r and s, what is the area of the metal frame, in square centimeter?

20. A thin piece of wire 40 meters long is cut into two pieces. One piece is used to form a circle with radius r, andthe other is used to form a square. No wire is left over. Which of the following represents the total area, insquare meters, of the circular and the square regions in terms of r?

(5/3)π + 5√3 (5/3) π + 10√3 (10/3) π + 5√3 (10/3) π + 10√3 (10/3) π + 20√3

QA - IPM

Page 130: QUESTION BANK - edumentor.co.inedumentor.co.in/Downloads/2018/Extra Content... · of India's oldest and most important uranium mine, ... Punjab banned e-cigarettes last year and Maharashtra

Co-ordinate Geometry

1. If ab ≠ 0 and points (–a,b) and (–b,a) are in the same quadrant of the xy–plane, is point (–x,y) in this samequadrant? (1) xy > 0 (2) ax > 0

2. In the xy–plane, at what two points does the graph of y = (x+a)(x+b) intersect the x–axis?(1) a + b = –1 (2) The graph intersects the y–axis at (0, –6).

3. For any triangle T in the xy–coordinate plan, the center of T is defined to be the point whose x–coordinate isthe average (arithmetic mean) of the x–coordinates of the vertices of T and whose y–coordinate is theaverage of the y–coordinates of the vertices of T. If a certain triangle has vertices at the points (0,0) and(6,0) and center at the point (3,2), what are the coordinates of the remaining vertex?A. (3,4) B. (3,6) C. (4,9) D. (6,4) E. (9,6)

4. Circle C and line k lie in the xy–plane. if circle C is centered at the origin and has radius 1, does line kintersect circle C?(1) the x–intercept of line k is greater than 1(2) the slope of line k is –1/10

5. In the rectangular coordinate system, are the points (r,s) and (u,v) equidistant from the origin?(1) r + s = 1 (2) u = 1 – r and v = 1 – s

6. In the x–y plane, what is the y–intercept of the line l?(1) The slope of the line l is 3 times its y intercept.(2) The x–intercept of line l is –1/3

7. In the figure shown, point P (–√3, 1) and Q (s, t) lie on the circle with center O.

What is value of s?

8. In the xy–plane, line k has positive slope and x–intercept 4. If the area of the triangle formed by line k andthe two axes is 12, what of the y– intercept of line?

9. Line l is defined by the equation y – 5x = 4 and line w is defined by the equation 10y + 2x + 20 = 0. If line k does not intersect line l, what is the degree measure of the angle formed by line k and line w?0 30 60 90 It cannot be determined from the information given.

QA - IPM

Page 131: QUESTION BANK - edumentor.co.inedumentor.co.in/Downloads/2018/Extra Content... · of India's oldest and most important uranium mine, ... Punjab banned e-cigarettes last year and Maharashtra

10. In the rectangular coordinate plane points X and Z lie on the same line through the origin and points W and Y lie on the same line through the origin. If a2 + b2 = c2 + d2 and e2 + f2 = g2 + h2, what is the value of lengthXZ – length WY?

-2 -1 0 1 2

11. In the xy-coordinate system, what is the slope of the line that goes through the origin and is equidistant fromthe two points P = (1, 11) and Q = (7, 7)?2 2.25 2.50 2.75 3

12. What is the slope of the line represented by the equation x + 2y = 1?-3/2 -1 -1/2 0 ½

13. A certain square is to be drawn on a coordinate plane. One of the vertices must be on the origin, and thesquare is to have an area of 100. If all coordinates of the vertices must be integers, how many different wayscan this square be drawn?4 6 8 10 12

14. Does the equation y = (x – p)(x – q) intercept the x-axis at the point (2,0)?(1) pq = -8 (2) -2 – p = q

15. Does line S intersect line segment QR?

(1) The equation of line S is y = -x + 4. (2) The slope of line S is -1.

16. Line L contains the points (2,3) and (p,q). If q = 2, which of the following could be the equation of line m,which is perpendicular to line L?(A) 2x + y = px + 7 (B) 2x + y = –px (C) x + 2y = px + 7 (D) y – 7 = x ÷ (p – 2) (E) 2x + y = 7 – px

17. Point K = (A,0), Point G = ( )92,42 ++ AA . Is the distance between point K and G prime?

(1) A2 – 5A – 6 = 0 (2) A > 2

18. The (x, y) coordinates of points P and Q are (-2, 9) and (-7, -3), respectively. The height of equilateraltriangle XYZ is the same as the length of line segment PQ. What is the area of triangle XYZ?169/√3 84.5 75√3 169√3 /4 225√3 /4

19. If points A and B are on the y-axis in the figure, what is the area of equilateral triangle ABC?

QA - IPM

Page 132: QUESTION BANK - edumentor.co.inedumentor.co.in/Downloads/2018/Extra Content... · of India's oldest and most important uranium mine, ... Punjab banned e-cigarettes last year and Maharashtra

(1) The coordinates of point B are (0, 5√3). (2) The coordinates of point C are (6, 3√3).

20. The line 3x + 4y = 8 passes through all of the quadrants in the coordinate plane except:I II III IV II and IV.

21. If p and q are nonzero numbers, and p is not equal to q, in which quadrant of the coordinate system doespoint (p, p – q) lie?(1) (p, q) lies in quadrant IV. (2) (q, -p) lies in quadrant III.

22. The coordinates of points A and C are (0, -3) and (3, 3), respectively. If point B lies on line AC between pointsA and C, and if AB = 2BC, which of the following represents the coordinates of point B?(1, -√5) (1, -1) (2, 1) (1.5, 0) (√5, √5)

23. In the xy-coordinate system, rectangle ABCD is inscribed within a circle having the equation x2 + y2 = 25.Line segment AC is a diagonal of the rectangle and lies on the x-axis. Vertex B lies in quadrant II and vertexD lies in quadrant IV. If side BC lies on line y = 3x + 15, what is the area of rectangle ABCD?(A) 15 (B) 30 (C) 40 (D) 45 (E) 50

24. The line represented by the equation y = 4 – 2x is the perpendicular bisector of line segment RP. If R has thecoordinates (4, 1), what are the coordinates of point P?(A) (–4, 1) (B) (–2, 2) (C) (0, 1) (D) (0, –1) (E) (2, 0)

25. A certain computer program randomly generates equations of lines in the form y = mx + b. If point Pis a point on a line generated by this program, what is the probability that the line does NOT pass throughfigure ABCD?

(A) ¾ (B) 3/5 (C) ½ (D) 2/5 (E) ¼

QA - IPM